Sei sulla pagina 1di 241

Saturday, November 10, 2007

StuffedNurse : diannemaydee's 300 items NLE reviewer

Situation – Richard has a nursing diagnosis of ineffective airway clearance related to


excessive secretions and is at risk for infection because of retained secretions. Part of Nurse
Mario’s nursing care plan is to loosen and remove excessive secretions in the airway.

Mario listens to Richard’s bilateral sounds and finds that congestion is in the upper lobes of
the lungs. The appropriate position to drain the anterior and posterior apical segments of the
lungs when Mario does percussion would be:

A. Client lying on his back then flat on his abdomen on Trendelenburg position
B. Client seated upright in bed or on a chair then leaning forward in sitting position then flat on
his back and on his abdomen
C. Client lying flat on his back and then flat on his abdomen
D. Client lying on his right then left side on Trendelenburg position

When documenting outcome of Richard’s treatment Mario should include the following in his
recording EXCEPT:

A. Color, amount and consistency of sputum


B. Character of breath sounds and respiratory rate before and after procedure
C. Amount of fluid intake of client before and after the procedure
D. Significant changes in vital signs

When assessing Richard for chest percussion or chest vibration and postural drainage, Mario
would focus on the following EXCEPT:

A. Amount of food and fluid taken during the last meal before treatment
B. Respiratory rate, breath sounds and location of congestion
C. Teaching the client’s relatives to perform the procedure
D. Doctor’s order regarding position restrictions and client’s tolerance for lying flat

Mario prepares Richard for postural drainage and percussion. Which of the following is a
special consideration when doing the procedure?

A. Respiratory rate of 16 to 20 per minute


B. Client can tolerate sitting and lying positions
C. Client has no signs of infection
D. Time of last food and fluid intake of the client
The purpose of chest percussion and vibration is to loosen secretions in the lungs. The
difference between the procedures is:

A. Percussion uses only one hand while vibration uses both hands
B. Percussion delivers cushioned blows to the chest with cupped palms while vibration gently
shakes secretion loose on the exhalation cycle
C. In both percussion and vibration the hands are on top of each other and hand action is in tune
with client’s breath rhythm
D. Percussion slaps the chest to loosen secretions while vibration shakes the secretions along with
the inhalation of air

Situation – A 61 year old man, Mr. Regalado, is admitted to the private ward for
observation after complaints of severe chest pain. You are assigned to take care of the client.

When doing an initial assessment, the best way for you to identify the client’s priority problem
is to:

A. Interview the client for chief complaints and other symptoms


B. Talk to the relatives to gather data about history of illness
C. Do auscultation to check for chest congestion
D. Do a physical examination while asking the client relevant questions

Nancy blames God for her situation. She is easily provoked to tears and wants to be left alone,
refusing to eat or talk to her family. A religious person before, she now refuses to pray or go to
church stating that God has abandoned her. The nurse understands that Nancy is grieving for
her self and is in the stage of:

A. bargaining
B. denial
C. anger
D. acceptance

Which of the following ethical principles refers to the duty to do good?

A. Beneficence
B. Fidelity
C. Veracity
D. Nonmaleficence
During which step of the nursing process does the nurse analyze data related to the patient's
health status?

A. Assessment
B. Implementation
C. Diagnosis
D. Evaluation

The basic difference between nursing diagnoses and collaborative problems is that

A. nurses manage collaborative problems using physician-prescribed interventions.


B. collaborative problems can be managed by independent nursing interventions.
C. nursing diagnoses incorporate physician-prescribed interventions.
D. nursing diagnoses incorporate physiologic complications that nurses monitor to detect change
in status.

Situation – Mrs. Seva, 52 years old, asks you about possible problems regarding her
elimination now that she is in the menopausal stage.

Instruction on health promotion regarding urinary elimination is important. Which would you
include?

A. Hold urine as long as she can before emptying the bladder to strengthen her sphincter muscles
B. If burning sensation is experienced while voiding, drink pineapple juice
C. After urination, wipe from anal area up towards the pubis
D. Tell client to empty the bladder at each voiding

Mrs. Seva also tells the nurse that she is often constipated. Because she is aging, what physical
changes predispose her to constipation?

A. inhibition of the parasympathetic reflex


B. weakness of sphincter muscles of anus
C. loss of tone of the smooth muscles of the colon
D. decreased ability to absorb fluids in the lower intestines

The nurse understands that one of these factors contributes to constipation:

A. excessive exercise
B. high fiber diet
C. no regular time for defecation daily
D. prolonged use of laxative

You will do nasopharyngeal suctioning on Mr. Abad. Your guide for the length of insertion of
the tubing for an adult would be:

A. tip of the nose to the base of the neck


B. the distance from the tip of the nose to the middle of the neck
C. the distance from the tip of the nose to the tip of the ear lobe
D. eight to ten inches

Situation– Mr. Dizon, 84 years old, brought to the Emergency Room for complaint of
hypertension, flushed face, severe headache, and nausea. You are doing the initial
assessment of vital signs.

You are to measure the client’s initial blood pressure reading by doing all of the following
EXCEPT:

A. Take the blood pressure reading on both arms for comparison


B. Listen to and identify the phases of Korotkoff’s sound
C. Pump the cuff to around 50 mmHg above the point where the pulse is obliterated
D. Observe procedures for infection control

A pulse oximeter is attached to Mr. Dizon’s finger to:

A. Determine if the client’s hemoglobin level is low and if he needs blood transfusion
B. Check level of client’s tissue perfusion
C. Measure the efficacy of the client’s anti-hypertensive medications
D. Detect oxygen saturation of arterial blood before symptoms of hypoxemia develops

In which type of shock does the patient experiences a mismatch of blood flow to the cells?

A. Distributive
B. Cardiogenic
C. Hypovolemic
D. Septic

The preferred route of administration of medication in the most acute care situations is which
of the following routes?

A. Intravenous
B. Epidural
C. Subcutaneous
D. Intramuscular

After a few hours in the Emergency Room, Mr. Dizon is admitted to the ward with an order of
hourly monitoring of blood pressure. The nurse finds that the cuff is too narrow and this will
cause the blood pressure reading to be:

A. inconsistent
B. low systolic and high diastolic
C. higher than what the reading should be
D. lower than what the reading should be
Through the client’s health history, you gather that Mr. Dizon smokes and drinks coffee. When
taking the blood pressure of a client who recently smoked or drank coffee, how long should the
nurse wait before taking the client’s blood pressure for accurate reading?

A. 15 minutes
B. 30 minutes
C. 1 hour
D. 5 minutes

While the client has pulse oximeter on his fingertip, you notice that the sunlight is shining on
the area where the oximeter is. Your action will be to:

A. Set and turn on the alarm of the oximeter


B. Do nothing since there is no identified problem
C. Cover the fingertip sensor with a towel or bedsheet
D. Change the location of the sensor every four hours

When taking blood pressure reading the cuff should be:

A. deflated fully then immediately start second reading for same client
B. deflated quickly after inflating up to 180 mmHg
C. large enough to wrap around upper arm of the adult client 1 cm above brachial artery
D. inflated to 30 mmHg above the estimated systolic BP based on palpation of radial or bronchial
artery

To ensure client safety before starting blood transfusions the following are needed before the
procedure can be done EXCEPT:

A. take baseline vital signs


B. blood should be warmed to room temperature for 30 minutes before blood transfusions is
administered
C. have two nurses verify client identification, blood type, unit number and expiration date of
blood
D. get consent signed for blood transfusion

Mr. Bruno asks what the “normal” allowable salt intake is. Your best response to Mr. Bruno is:

A. 1 tsp of salt/day with iodine and sprinkle of MSG


B. 5 gms per day or 1 tsp of table salt/day
C. 1 tbsp of salt/day with some patis and toyo
D. 1 tsp of salt/day but no patis and toyo

Which of the following methods is the best method for determining nasogastric tube placement
in the stomach?
A. X-ray
B. Observation of gastric aspirate
C. Testing of pH of gastric aspirate
D. Placement of external end of tube under water

Which of the following is the most important risk factor for development of Chronic
Obstructive Pulmonary Disease?

A. Cigarette smoking
B. Occupational exposure
C. Air pollution
D. Genetic abnormalities

When performing endotracheal suctioning, the nurse applies suctioning while withdrawing
and gently rotating the catheter 360 degrees for which of the following time periods?

A. 10-15 seconds
B. 30-35 seconds
C. 20-25 seconds
D. 0-5 seconds

The nurse auscultates the apex beat at which of the following anatomical locations?

A. Fifth intercostal space, midclavicular line


B. Mid-sternum
C. 2” to the left of the lower end of the sternum
D. 1” to the left of the xiphoid process

Which of the following terms describes the amount of blood ejected per heartbeat?

A. Stroke volume
B. Cardiac output
C. Ejection fraction
D. Afterload

You are to apply a transdermal patch of nitoglycerin to your client. The following are
important guidelines to observe EXCEPT:

A. Apply to hairless clean area of the skin not subject to much wrinkling
B. Patches may be applied to distal part of the extremities like forearm
C. Change application and site regularly to prevent irritation of the skin
D. Wear gloves to avoid any medication on your hand

The GAUGE size in ET tubes determines:


A. The external circumference of the tube
B. The internal diameter of the tube
C. The length of the tube
D. The tube’s volumetric capacity

The nurse is correct in performing suctioning when she applies the suction intermittently
during:

A. Insertion of the suction catheter


B. Withdrawing of the suction catheter
C. both insertion and withdrawing of the suction catheter
D. When the suction catheter tip reaches the bifurcation of the trachea

The purpose of the cuff in Tracheostomy tube is to:

A. Separate the upper and lower airway


B. Separate trachea from the esophagus
C. Separate the larynx from the nasopharynx
D. Secure the placement of the tube

Which priority nursing diagnosis is applicable for a patient with indwelling urinary catheter?

A. Self esteem disturbance


B. Impaired urinary elimination
C. Impaired skin integrity
D. Risk for infection

An incontinent elderly client frequently wets his bed and eventually develop redness and skin
excoriation at the perianal area. The best nursing goal for this client is to:

A. Make sure that the bed linen is always dry


B. Frequently check the bed for wetness and always keep it dry
C. Place a rubber sheet under the client’s buttocks
D. Keep the patient clean and dry

As a Nurse Manager, DMLM enjoys her staff of talented and self motivated individuals. She
knew that the leadership style to suit the needs of this kind of people is called:

A. Autocratic
B. Participative
C. Democratic
D. Laissez Faire

A fire has broken in the unit of DMLM R.N. The best leadership style suited in cases of
emergencies like this is:
A. Autocratic
B. Participative
C. Democratic
E. Laissez Faire

Which step of the management process is concerned with Policy making and Stating the goals
and objective of the institution?

A. Planning
B. Organizing
C. Directing
D. Controlling

In the management process, the periodic checking of the results of action to make sure that it
coincides with the goal of the institution is termed as:

A. Planning
B. Evaluating
C. Directing
D. Organizing

The Vision of a certain agency is usually based on their beliefs, Ideals and Values that directs
the organization. It gives the organization a sense of purpose. The belief, Ideals and Values of
this Agency is called:

A. Philosophy
B. Mission
C. Vision
D. Goals and Objectives

Mr. CKK is unconscious and was brought to the E.R. Who among the following can give
consent for CKK’s Operation?

A. Doctor
B. Nurse
C. Next of Kin
D. The Patient

Mang Carlos has been terminally ill for 5 years. He asked his wife to decide for him when he is
no longer capable to do so. As a Nurse, You know that this is called:

A. Last will and testament


B. DNR
C. Living will
D. Durable Power of Attorney
Mang Carlos has a standing DNR order. He then suddenly stopped breathing and you are at
his bedside. You would:

A. Give extraordinary measures to save Mang Carlos


B. Stay with Mang Carlos and Do nothing
C. Call the physician
D. Activate Code Blue

It is not a legally binding document but nevertheless, Very important in caring for the patients.

A. BON Resolution No. 220 Series of 2002


B. Patient’s Bill of Rights
C. Nurse’s Code of Ethics
D. Philippine Nursing Act of 2002

In monitoring the patient in PACU, the nurse correctly identify that checking the patient’s vital
signs is done every:

A. 1 hour
B. 5 minutes
C. 15 minutes
D. 30 minutes

diannemayde R.N is conducting a research on her unit about the effects of effective nurse-
patient communication in decreasing anxiety of post operative patients. Which of the following
step in nursing research should she do next?

A. Review of related literature


B. Ask permission from the hospital administrator
C. Determine the research problem
D. Formulate ways on collecting the data

Before diannemaydee perform the formal research study, what do you call the pre testing,
small scale trial run to determine the effectiveness of data collection and methodological
problem that might be encountered?

A. Sampling
B. Pre testing
C. Pre Study
E. Pilot Study

On the study “effects of effective nurse-patient communication in decreasing anxiety of post


operative patients” What is the Independent variable?
A. Effective Nurse-patient communication
B. Communication
C. Decreasing Anxiety
D. Post operative patient

On the study “effects of effective nurse-patient communication in decreasing anxiety of post


operative patients” What is the Dependent variable?

A. Effective Nurse-patient communication


B. Communication
C. Anxiety level
D. Post operative patient

In the recent technological innovations, which of the following describe researches that are
made to improve and make human life easier?

A. Pure research
B. Basic research
C. Applied research
D. Experimental research

Which of the following is not true about a Pure Experimental research?

A. There is a control group


B. There is an experimental group
C. Selection of subjects in the control group is randomized
D. There is a careful selection of subjects in the experimental group

When Mrs. Guevarra, a nurse, delegates aspects of the clients care to the nurse-aide who is an
unlicensed staff, Mrs. Guevarra

A. makes the assignment to teach the staff member


B. is assigning the responsibility to the aide but not the accountability for those tasks
C. does not have to supervise or evaluate the aide
D. most know how to perform task delegated

Process of formal negotiations of working conditions between a group of registered nurses and
employer is

A. grievance
B. arbitration
C. collective bargaining
D. strike

You are attending a certification on cardiopulmonary resuscitation (CPR) offered and required
by the hospital employing you. This is
A. professional course towards credits
B. inservice education
C. advance training
D. continuing education

The law which regulated the practice of nursing profession in the Philippines is:

A. R.A 9173
B. LOI 949
C. Patient’s Bill of Rights
E. Code of Ethics for Nurses

This quality is being demonstrated by a Nurse who raise the side rails of a confuse and
disoriented patient?

A. Autonomy
B. Responsibility
C. Prudence
D. Resourcefulness

Nurse Joel and Ana is helping a 16 year old Nursing Student in a case filed against the
student. The case was frustrated homicide. Nurse Joel and Ana are aware of the different
circumstances of crimes. They are correct in identifying which of the following
Circumstances that will be best applied in this case?

A. Justifying
B. Aggravating
C. Mitigating
D. Exempting

In signing the consent form, the nurse is aware that what is being observed as an ethical
consideration is the patient’s

A. Autonomy
B. Justice
C. Accountability
D. Beneficence

Why is there an ethical dilemma?

A. Because the law do not clearly state what is right from what is wrong
B. Because morality is subjective and it differs from each individual
C. Because the patient’s right coincide with the nurse’s responsibility
D. Because the nurse lacks ethical knowledge to determine what action is correct and what action
is unethical
Who among the following can work as a practicing nurse in the Philippines without taking the
Licensure examination?

A. Internationally well known experts which services are for a fee


B. Those that are hired by local hospitals in the country
C. Expert nurse clinicians hired by prestigious hospitals
D. Those involved in medical mission who’s services are for free

In signing the consent form, the nurse is aware that what is being observed as an ethical
consideration is the patient’s

A. Autonomy
B. Justice
C. Accountability
D. Beneficence

Nurse Buddy gave Inapsine instead of Insulin to a patient in severe hyperglycemia. He


reported the incident as soon as he knew there was an error. A nurse that is always ready to
answer for all his actions and decision is said to be:

A. Accountable
B. Responsible
C. Critical thinker
D. Assertive

Which of the following best describes Primary Nursing?

A. Is a form of assigning a nurse to lead a team of registered nurses in care of patient from
admission to discharge
B. A nurse is responsible in doing certain tasks for the patient
C. A registered nurse is responsible for a group of patients from admission to discharge
D. A registered nurse provides care for the patient with the assistant of nursing aides

The best and most effective method in times of staff and financial shortage is:

A. Functional Method
B. Primary Nursing
C. Team Nursing
E. Modular Method

You are doing bed bath to the client when suddenly, The nursing assistant rushed to the
room and tell you that the client from the other room was in Pain. The best intervention in
such case is:

A. Raise the side rails, cover the client and put the call bell within reach and then attend to the
client in pain to give the PRN medication
B. Tell the nursing assistant to give the pain medication to the client complaining of pain
C. Tell the nursing assistant to go the client’s room and tell the client to wait
D. Finish the bed bath quickly then rush to the client in Pain

Angie is a disoriented client who frequently falls from the bed. As her nurse, which of the
following is the best nursing intervention to prevent future falls?

A. Tell Angie not to get up from bed unassisted


B. Put the call bell within her reach
C. Put bedside commode at the bedside to prevent Angie from getting up
D. Put the bed in the lowest position ever

When injecting subcutaneous injection in an obese patient, It should be angled at around:

A. 45 °
B. 90 °
C. 180 °
D. Parallel to the skin

The following statements are all true about Z-Track technique except:

A. Z track injection prevent irritation of the subcutaneous tissues


B. The technique involve creating a Zig Zag like pattern of medication
C. It forces the medication to be contained at the subcutaneous tissues
D. It is used when administering Parenteral Iron

Communication is best undertaken if barriers are first removed. Considering this statement,
which of the following is considered as deterrent factor in communication?

A. Not universally accepted abbreviations


B. Wrong Grammar
C. Poor Penmanship
D. Old age of the client

Nurse DMLM is correct in identifying the correct sequence of events during abdominal
assessment if she identifies which of the following?

A. Inspection, Auscultation, Percussion, Palpation


B. Inspection, Percussion, Palpation, Auscultation
C. Inspection, Palpation, Percussion, Auscultation
D. Inspection, Auscultation, Palpation, Percussion

. To prevent injury and strain on the muscles, the nurse should observe proper body
mechanics. Among the following, which is a principle of proper body mechanics?
A. Broaden the space between the feet
B. Push instead of pull
C. Move the object away from the body when lifting
D. Bend at the waist, not on the knees

In taking the client’s blood pressure, the nurse should position the client’s arm:

A. At the level of the heart


B. Slightly above the level of the heart
C. At the 5th intercostals space midclavicular line
D. Below the level of the heart

What principle is used when the client with fever loses heat through giving cooling bed bath to
lower body temperature?

A. Radiation
B. Convection
C. Evaporation
D. Conduction

The most effective way in limiting the number of microorganism in the hospital is:

A. Using strict aseptic technique in all procedures


B. Wearing mask and gown in care of all patients with communicable diseases
C. Sterilization of all instruments
D. Handwashing

The immunoglobulin of the mother that crosses the placenta to protect the child is an example
of:

A. Natural active immunity


B. Natural passive immunity
C. Artificial active immunity
D. Artificial passive immunity

Richard is a subject of a research lead by his doctor. The nurse knows that all of the following
is a correct understanding as his right as a research subject except:

A. I can withdraw with this research even after the research has been started
B. My confidentiality will not be compromised in this research
C. I must choose another doctor if I withdrew from this research
D. I can withdraw with this research before the research has been started

Which of the following is a normal finding during assessment of a Chest tube in a 3 way bottle
system?
A. There is a continuous bubbling in the drainage bottle
B. There is an intermittent bubbling in the suction control bottle
C. The water fluctuates during inhalation of the patient
D. There is 3 cm of water left in the water seal bottle

In obtaining a urine specimen for culture and sensitivity on a catheterized patient, the nurse is
correct if:

A. Clamp the catheter for 30 minutes, Alcoholize the tube above the clamp site, Obtain a sterile
syringe and draw the specimen on the tube above the clamp

B. Alcoholize the self sealing port, obtain a sterile syringe and draw the specimen on the self
sealing port

C. Disconnect the drainage bag, obtain a sterile syringe and draw the specimen from the drainage
bag

D. Disconnect the tube, obtain a sterile syringe and draw the specimen from the tube

Which of the following is an example of secondary prevention?

A. Teaching the diabetic client on obtaining his blood sugar level using a glucometer
B. Screening patients for hypertension
C. Immunizing infants with BCG
D. Providing PPD on a construction site

Which of the following is a form of primary prevention?

A. Regular Check ups


B. Regular Screening
C. Self Medication
D. Immunization

An abnormal condition in which a person must sit, stand or use multiple pillows when lying
down is:

A. Orthopnea
B. Dyspnea
C. Eupnea
D. Apnea

As a nurse assigned for care for geriatric patients, you need to frequently assess your
patient using the nursing process. Which of the following needs be considered with the
highest priority?
A. Patients own feeling about his illness
B. Safety of the client especially those elderly clients who frequently falls
C. Nutritional status of the elderly client
D. Physiologic needs that are life threatening

The component that should receive the highest priority before physical examination is the:

A. Psychological preparation of the client


B. Physical Preparation of the client
C. Preparation of the Environment
D. Preparation of the Equipments

Legally, Patients chart are:

A. Owned by the government since it is a legal document


B. Owned by the doctor in charge and should be kept from the administrator for whatever reason
C. Owned by the hospital and should not be given to anyone who request it other than the doctor
in charge
D. Owned by the patient and should be given by the nurse to the client as requested

Which of the following categories identifies the focus of community/public health nursing
practice?

A. Promoting and maintaining the health of populations and preventing and minimizing the
progress of disease
B. Rehabilitation and restorative services
C. Adaptation of hospital care to the home environment
D. Hospice care delivery

A major goal for home care nurses is

A. restoring maximum health function.


B. promoting the health of populations.
C. minimizing the progress of disease.
D. maintaining the health of populations.

A written nursing care plan is a tool that:

A. Check whether nursing care goals were achieved


B. Gives quality nursing care
C. Select the appropriate nursing intervention
D. Make a nursing diagnosis

Gina, A client in prolong labor said she cannot go on anymore. The health care team decided
that both the child and the mother cannot anymore endure the process. The baby is premature
and has a little chance of surviving. Caesarian section is not possible because Gina already lost
enough blood during labor and additional losses would tend to be fatal. The husband decided
that Gina should survive and gave his consent to terminate the fetus. The principle that will be
used by the health care team is:

A. Beneficence
B. Non malfeasance
C. Justice
D. Double effect

Situation – There are various developments in health education that the nurse should know
about:

The provision of health information in the rural areas nationwide through television and radio
programs and video conferencing is referred to as:

A. Community health program


B. Telehealth program
C. Wellness program
D. Red Cross program

In teaching the sister of a diabetic client about the proper use of a glucometer in determining
the blood sugar level of the client, The nurse is focusing in which domain of learning
according to bloom?

A. Cognitive
B. Affective
C. Psychomotor
D. Affiliative

A nearby community provides blood pressure screening, height and weight measurement,
smoking cessation classes and aerobics class services. This type of program is referred to as

A. outreach program
B. hospital extension program
C. barangay health program
D. wellness program

After cleaning the abrasions and applying antiseptic, the nurse applies cold compress to the
swollen ankle as ordered by the physician. This statement shows that the nurse has correct
understanding of the use of cold compress:

A. Cold compress reduces blood viscosity in the affected area


B. It is safer to apply than hot compress
C. Cold compress prevents edema and reduces pain
D. It eliminates toxic waste products due to vasodilation
After receiving prescription for pain medication, Ronnie is instructed to continue applying 30
minute cold at home and start 30 minute hot compress the next day. You explain that the use of
hot compress:

A. Produces anesthetic effect


B. Increases nutrition in the blood to promote wound healing
C. Increase oxygenation to the injured tissues for better healing
D. Induces vasoconstriction to prevent infection

Situation – A nursing professor assigns a group of students to do data gathering by


interviewing their classmates as subjects.

She instructed the interviewees not to tell the interviewees that the data gathered are for her
own research project for publication. This teacher has violated the student’s right to:

A. Not be harmed
B. Disclosure
C. Privacy
D. Self-determination

Before the nurse researcher starts her study, she analyzes how much time, money, materials
and people she will need to complete the research project. This analysis prior to beginning the
study is called:

A. Validity
B. Feasibility
C. Reliability
D. Researchability

Data analysis is to be done and the nurse researcher wants to include variability. These include
the following EXCEPT:

A. Variance
B. Range
C. Standards of Deviation
D. Mean

Nurse Minette needs to schedule a first home visit to OB client Leah. When is a first home-
care visit typically made?

A. Within 4 days after discharge


B. Within 24 hours after discharge
C. Within 1 hour after discharge
D. Within 1 week of discharge
By force of law, therefore, the PRC-Board of Nursing released Resolution No. 14 Series of
1999 entitled: “Adoption of a Nursing Specialty Certification Program and Creation of
Nursing Specialty Certification Council.” This rule-making power is called:

A. Quasi-Judicial Power
B. Regulatory Power
C. Quasi-Legislative Power
D. Executive/Promulgating Power

Anita is performing BSE and she stands in front of the Mirror. The rationale for standing in
front of the mirror is to check for:

A. Unusual discharges coming out from the breast


B. Any obvious malignancy
C. The Size and Contour of the breast
D. Thickness and lumps in the breast

An emerging technique in screening for Breast Cancer in developing countries like the
Philippines is:

A. Mammography once a year starting at the age of 50


B. Clinical BSE Once a year
C. BSE Once a month
D. Pap smear starting at the age of 18 or earlier if sexually active

Transmission of HIV from an infected individual to another person occurs:

A. Most frequently in nurses with needlesticks


B. Only if there is a large viral load in the blood
C. Most commonly as a result of sexual contact
D. In all infants born to women with HIV infection

After a vaginal examination, the nurse determines that the client’s fetus is in an occiput
posterior position. The nurse would anticipate that the client will have:

A. A precipitous birth
B. Intense back pain
C. Frequent leg cramps
D. Nausea and vomiting

The rationales for using a prostaglandin gel for a client prior to the induction of labor is to:

A. Soften and efface the cervix


B. Numb cervical pain receptors
C. Prevent cervical lacerations
D. Stimulate uterine contractions
Nurse Lorena is a Family Planning and Infertility Nurse Specialist and currently attends to
FAMILY PLANNING CLIENTS AND INFERTILE COUPLES. The following conditions
pertain to meeting the nursing needs of this particular population group.

Dina, 17 years old, asks you how a tubal ligation prevents pregnancy. Which would be the best
answer?

A. Prostaglandins released from the cut fallopian tubes can kill sperm
B. Sperm can not enter the uterus because the cervical entrance is blocked.
C. Sperm can no longer reach the ova, because the fallopian tubes are blocked
D. The ovary no longer releases ova as there is no where for them to go.

The Dators are a couple undergoing testing for infertility. Infertility is said to exist when:

A. a woman has no uterus


B. a woman has no children
C. a couple has been trying to conceive for 1 year
D. a couple has wanted a child for 6 months

The correct temperature to store vaccines in a refrigerator is:

A. between -4 deg C and +8 deg C


B. between 2 deg C and +8 deg C
C. between -8 deg C and 0 deg C
D. between -8 deg C and +4 deg C

Which of the following vaccines is not done by intramuscular (IM) injection?

A. Measles vaccine
B. DPT
C. Hepa-B vaccine
D. Tetanus toxoids

This vaccine content is derived from RNA recombinants.

A. Measles
B. Tetanus toxoids
C. Hepatitis B vaccines
D. DPT
This special form is used when the patient is admitted to the unit. The nurse completes the
information in this record particularly his/her basic personal data, current illness, previous
health history, health history of the family, emotional profile, environmental history as well as
physical assessment together with nursing diagnosis on admission. What do you call this
record?
A. Nursing Kardex
B. Nursing Health History and Assessment Worksheet
C. Medicine and Treatment Record
D. Discharge Summary

These are sheets/forms which provide an efficient and time saving way to record information
that must be obtained repeatedly at regular and/or short intervals of time. This does not replace
the progress notes; instead this record of information on vital signs, intake and output,
treatment, postoperative care, post partum care, and diabetic regimen, etc. This is used
whenever specific measurements or observations are needed to be documented repeatedly.
What is this?

A. Nursing Kardex
B. Graphic Flow Sheets
C. Discharge Summary
D. Medicine and Treatment Record

These records show all medications and treatment provided on a repeated basis. What do you
call this record?

A. Nursing Health History and Assessment Worksheet


B. Discharge Summary
C. Nursing Kardex
D. Medicine and Treatment Record

This flip-over card is usually kept in a portable file at the Nurse’s Station. It has 2-parts: the
activity and treatment section and a nursing care plan section. This carries information about
basic demographic data, primary medical diagnosis, current orders of the physician to be
carried out by the nurse, written nursing care plan, nursing orders, scheduled tests and
procedures, safety precautions in patient care and factors related to daily living activities. This
record is used in the charge-of-shift reports or during the bedside rounds or walking rounds.
What record is this?

A. Discharge Summary
B. Medicine and Treatment Record
C. Nursing Health History and Assessment Worksheet
D. Nursing Kardex

Most nurses regard this conventional recording of the date, time, and mode by which the
patient leaves a healthcare unit but this record includes importantly, directs of planning for
discharge that starts soon after the person is admitted to a healthcare institution. It is accepted
that collaboration or multidisciplinary involvement (of all members of the health team) in
discharge results in comprehensive care. What do you call this?

A. Discharge Summary
B. Nursing Kardex
C. Medicine and Treatment Record
D. Nursing Health History and Assessment Worksheet

Based on the Code of Ethics for Filipino Nurses, what is regarded as the hallmark of
nursing responsibility and accountability?

A. Human rights of clients, regardless of creed and gender


B. The privilege of being a registered professional nurses
C. Health, being a fundamental right of every individual
D. Accurate documentation of actions and outcomes

A nurse should be cognizant that professional programs for specialty certification by the Board
of Nursing accredited through the:

A. Professional Regulation Commission


B. Nursing Specialty Certification Council
C. Association of Deans of Philippine Colleges of Nursing
D. Philippine Nurse Association

Integrated management for childhood illness is the universal protocol of care endorsed by
WHO and is use by different countries of the world including the Philippines. In any case that
the nurse classifies the child and categorized the signs and symptoms in PINK category, You
know that this means:

A. Urgent referral
B. Antibiotic Management
C. Home treatment
D. Out patient treatment facility is needed

You know that fast breathing of a child age 13 months is observed if the RR is more than:

A. 40
B. 50
C. 60
D. 30

Angelo, An 8 month old child is brought to the health care facility with sunken eyes. You pinch
his skin and it goes back very slowly. In what classification of dehydration will you categorize
Angelo?

A. No Dehydration
B. Some Dehydration
C. Severe Dehydration
D. Diarrhea
In responding to the care concerns of children with severe disease, referral to the hospital is of
the essence especially if the child manifests which of the following?

A. Wheezing
B. Stop feeding well
C. Fast breathing
D. Difficulty to awaken

A child with ear problem should be assessed for the following, EXCEPT:

A. is there any fever?


B. Ear discharge
C. If discharge is present for how long?
D. Ear pain

If the child does not have ear problem, using IMCI, what should you as the nurse do?

A. Check for ear discharge


B. Check for tender swellings behind the ear
C. Check for ear pain
D. Go to the next question, check for malnutrition

All of the following are treatment for a child classified with no dehydration except:

A. 1,000 ml to 1,400 ml be given within 4 hours


B. Continue feeding
C. Have the child takes as much fluid as he wants
D. Return the child to the doctor if condition worsens

An ear infection that persists but still less than 14 days is classified as:

A. Mastoiditis
B. Chronic Ear Infection
C. Acute Ear Infection
D. Otitis Media

If a child has two or more pink signs, you would classify the child as having:

A. No disease
B. Mild form of disease
C. Urgent Referral
D. Very severe disease

The nurse knows that the most common complication of Measles is:

A Pneumonia and larynigotracheitis


B. Encephalitis
C. Otitis Media
D. Bronchiectasis

A client scheduled for hysterosalpingography needs health teaching before the procedure. The
nurse is correct in telling the patient that:

A. She needs to void prior to the procedure


B. A full bladder is needed prior to the procedure
C. Painful sensation is felt as the needle is inserted
D. Flushing sensation is felt as the dye in injected

In a population of 9,500. What is your estimate of the population of pregnant woman needing
tetanus toxoid vaccination?

A. 632.5
B. 512.5
C. 450.5
D. 332.5

All of the following are seen in a child with measles. Which one is not?

A. Reddened eyes
B. Coryza
C. Pustule
D. Cough

Mobilizing the people to become aware of their own problem and to do actions to solve it is
called:

A. Community Organizing
B. Family Nursing Care Plan
C. Nursing Intervention
D. Nursing Process

Prevention of work related accidents in factories and industries are responsibilities of which
field of nursing?

A. School health nursing


B. Private duty nursing
C. Occupational health nursing
D. Institutional nursing

In one of your home visit to Mr. JUN, you found out that his son is sick with cholera. There is
a great possibility that other member of the family will also get cholera. This possibility is a/an:
A. Foreseeable crisis
B. Health threat
C. Health deficit
D. Crisis

Why is bleeding in the leg of a pregnant woman considered as an emergency?

A. Blood volume is greater in pregnant woman; therefore, blood loss is increased


B. There is an increase blood pressure during pregnancy increasing the likelihood of hemorrhage
C. Pregnant woman are anemic, all forms of blood loss should be considered as an emergency
especially if it is in the lower extremity
D. The pressure of the gravid uterus will exert additional force thus, increasing the blood loss in
the lower extremities

Aling Maria is nearing menopause. She is habitually taking cola and coffee for the past 20
years. You should tell Aling Maria to avoid taking caffeinated beverages because:

A. It is stimulating
B. It will cause nervousness and insomnia
C. It will contribute to additional bone demineralization
D. It will cause tachycardia and arrhythmias

All of the following are contraindication when giving Immunization except:

A. BCG Vaccines can be given to a child with AIDS


B. BCG Vaccine can be given to a child with Hepatitis B
C. DPT Can be given to a child that had convulsion 3 days after being given the first DPT Dose
D. DPT Can be given to a child with active convulsion or other neurological disease

Theresa, a mother with a 2 year old daughter asks, “at what age can I be able to take the blood
pressure of my daughter as a routine procedure since hypertension is common in the family?”
Your answer to this is:

A. At 2 years you may


B. As early as 1 year old
C. When she’s 3 years old
D. When she’s 6 years old

Baby John develops hyperbilirubinemia. What is a method used to treat hyperbilirubinemia


in a newborn?

A. Keeping infants in a warm and dark environment


B. Administration of cardiovascular stimulant
C. Gentle exercise to stop muscle breakdown
D. Early feeding to speed passage of meconium
The community/Public Health Bag is:

A. a requirement for home visits


B. an essential and indispensable equipment of the community health nurse
C. contains basic medications and articles used by the community health nurse
D. a tool used by the Community health nurse is rendering effective nursing procedures during a
home visit

What is the rationale in the use of bag technique during home visits?

A. It helps render effective nursing care to clients or other members of the family
B. It saves time and effort of the nurse in the performance of nursing procedures
C. It should minimize or prevent the spread of infection from individuals to families
D. It should not overshadow concerns for the patient

In consideration of the steps in applying the bag technique, which side of the paper lining of
the CHN bag is considered clean to make a non-contaminated work area?

A. The lower lip


B. The outer surface
C. The upper tip
D. The inside surface

How many words does a typical 12-month-old infant use?

A. About 12 words
B. Twenty or more words
C. About 50 words
D. Two, plus “mama” and “papa”

. During operation, The OR suite’s lighting, noise, temperature and other factors that affects the
operation are managed by whom?

A. Nurse Supervisor
B. Surgeon
C. Circulating nurse
D. Scrub nurse

Before and after the operation, the operating suite is managed by the:

A. Surgeon
B. Nurse Supervisor
C. Nurse Manager
D. Chief Nurse
The counting of sponges is done by the Surgeon together with the:

A. Circulating nurse
B. Scrub nurse
C. Assistant surgeon
D. Nurse supervisor

The OR team performs distinct roles for one surgical procedure to be accomplished within a
prescribed time frame and deliver a standard patient outcome. While the surgeon performs the
surgical procedure, who monitors the status of the client like urine output, blood loss?

A. Scrub Nurse
B. Surgeon
C. Anaesthesiologist
D. Circulating Nurse

Surgery schedules are communicated to the OR usually a day prior to the procedure by the
nurse of the floor or ward where the patient is confined. For orthopedic cases, what
department is usually informed to be present in the OR?

A. Rehabilitation department
B. Laboratory department
C. Maintenance department
D. Radiology department

In some hip surgeries, an epidural catheter for Fentanyl epidural analgesia is given. What is
your nursing priority care in such a case?

A. Instruct client to observe strict bed rest


B. Check for epidural catheter drainage
C. Administer analgesia through epidural catheter as prescribed
D. Assess respiratory rate carefully

The patient’s medical record can work as a double edged sword. When can the medical record
become the doctor’s/nurse’s worst enemy?

A. When the record is voluminous


B. When a medical record is subpoenaed in court
C. When it is missing
D. When the medical record is inaccurate, incomplete, and inadequate

Disposal of medical records in government hospitals/institutions must be done in close


coordination with what agency?

A. Department of Interior and Local Government (DILG)


B. Metro Manila Development Authority (MMDA)
C. Records Management Archives Office (RMAO)
D. Department of Health (DOH)

In the hospital, when you need the medical record of a discharged patient for research you will
request permission through:

A. Doctor in charge
B. The hospital director
C. The nursing service
D. Medical records section

You will give health instructions to Carlo, a case of bronchial asthma. The health instruction
will include the following, EXCEPT:

A. Avoid emotional stress and extreme temperature


B. Avoid pollution like smoking
C. Avoid pollens, dust, seafood
D. Practice respiratory isolation

As the head nurse in the OR, how can you improve the effectiveness of clinical alarm systems?

A. Limit suppliers to a few so that quality is maintained


B. Implement a regular inventory of supplies and equipment
C. Adherence to manufacturer’s recommendation
D. Implement a regular maintenance and testing of alarm systems

Overdosage of medication or anesthetic can happen even with the aid of technology like infusion
pumps, sphygmomanometer and similar devices/machines. As a staff, how can you improve the
safety of using infusion pumps?

A. Check the functionality of the pump before use


B. Select your brand of infusion pump like you do with your cellphone
C. Allow the technician to set the infusion pump before use
D. Verify the flow rate against your computation

While team effort is needed in the OR for efficient and quality patient care delivery, we should
limit the number of people in the room for infection control. Who comprise this team?

A. Surgeon, anesthesiologist, scrub nurse, radiologist, orderly


B. Surgeon, assistants, scrub nurse, circulating nurse, anesthesiologist
C. Surgeon, assistant surgeon, anesthesiologist, scrub nurse, pathologist
D. Surgeon, assistant surgeon, anesthesiologist, intern, scrub nurse

When surgery is on-going, who coordinates the activities outside, including the family?
A. Orderly/clerk
B. Nurse Supervisor
C. Circulating Nurse
D. Anesthesiologist

The breakdown in teamwork is often times a failure in:

A. Electricity
B. Inadequate supply
C. Leg work
D. Communication

To prevent recurrent attacks on client with glomerulonephritis, the nurse instructs the client to:

A. Take a shower instead of tub baths


B. Avoid situations that involve physical activity
C. Continue the same restriction on fluid intake
D. Seek early treatment for respiratory infection

When administering Tapazole, The nurse should monitor the client for which of the following
adverse effect?

A. Hyperthyroidism
B. Hypothyroidism
C. Drowsiness
D. Seizure

Post bronchoscopy, the nurse priority is to check which of the following before feeding?

A. Gag reflex
B. Wearing off of anesthesia
C. Swallowing reflex
D. Peristalsis

Changes normally occur in the elderly. Among the following, which is a normal change in an
elderly client?

A. Increased sense of taste


B. Increased appetite
C. Urinary frequency
D. Thinning of the lens

Colostomy is a surgically created anus. It can be temporary or permanent, depending on the


disease condition.
Skin care around the stoma is critical. Which of the following is not indicated as a skin care
barriers?

A. Apply liberal amount of mineral oil to the area


B. Use karaya paste and rings around the stoma
C. Clean the area daily with soap and water before applying bag
D. Apply talcum powder twice a day

What health instruction will enhance regulation of a colostomy (defecation) of clients?

A. Irrigate after lunch everyday


B. Eat fruits and vegetables in all three meals
C. Eat balanced meals at regular intervals
D. Restrict exercise to walking only

After ileostomy, which of the following condition is NOT expected?

A. Increased weight
B. Irritation of skin around the stoma
C. Liquid stool
D. Establishment of regular bowel movement

The following are appropriate nursing interventions during colostomy irrigation, EXCEPT:

A. Increase the irrigating solution flow rate when abdominal cramps is felt
B. Insert 2-4 inches of an adequately lubricated catheter to the stoma
C. Position client in semi-Fowler
D. Hang the solution 18 inches above the stoma

What sensation is used as a gauge so that patients with ileostomy can determine how often their
pouch should be drained?

A. Sensation of taste
B. Sensation of pressure
C. Sensation of smell
D. Urge to defecate

In performing a cleansing enema, the nurse performs the procedure by positioning the client in:

A. Right lateral position


B. Left lateral position
C. Right sim’s position
D. Left sim’s position

Mang Caloy is scheduled to have a hemorrhoidectomy, after the operation, you would expect that
the client’s position post operatively will be:

A. Knee chest position


B. Side lying position
C. Sims position
D. Genopectoral position

You would expect that after an abdominal perineal resection, the type of colostomy that will be
use is?

A. Double barrel colostomy


B. Temporary colostomy
C. Permanent colostomy
D. An Ileostomy

You are an ostomy nurse and you know that colostomy is defined as:

A. It is an incision into the colon to create an artificial opening to the exterior of the abdomen
B. It is end to end anastomosis of the gastric stump to the duodenum
C. It is end to end anastomosis of the gastric stump to the jejunum
D. It is an incision into the ileum to create an artificial opening to the exterior of the abdomen

Larry, 55 years old, who is suspected of having colorectal cancer, is admitted to the CI. After
taking the history and vital signs the physician does which test as a screening test for colorectal
cancer.

A. Barium enema
B. Carcinoembryonic antigen
C. Annual digital rectal examination
D. Proctosigmoidoscopy

Symptoms associated with cancer of the colon include:

A. constipation, ascites and mucus in the stool


B. diarrhea, heart burn and eructation
C. blood in the stools, anemia, and “pencil shaped” stools
D. anorexia, hematemesis, and increased peristalasis

24 Hours after creation of colostomy, Nurse Violy is correct if she identify that the normal
appearance of the stoma is :

A. Pink, moist and slightly protruding from the abdomen


B. Gray, moist and slightly protruding from the the abdomen
C. Pink, dry and slightly protruding from the abdomen
D. Red, moist and slightly protruding from the abdomen

In cleaning the stoma, the nurse would use which of the following cleaning mediums?

A. Hydrogen Peroxide, water and mild soap


B. Providone Iodine, water and mild soap
C. Alcohol, water and mild soap
D. Mild soap and water

When observing a return demonstration of a colostomy irrigation, you know that more teaching is
required if pt:

A. Lubricates the tip of the catheter prior to inserting into the stoma
B. Hangs the irrigating bag on the bathroom door cloth hook during fluid insertion
C. Discontinues the insertion of fluid after only 500 ml of fluid has been instilled
D. Clamps of the flow of fluid when felling uncomfortable
What does a sample group represent?

A. Control group
B. Study subjects
C. General population
D. Universe

As a nurse, you can help improve the effectiveness of communication among healthcare givers
by:

A. Use of reminders of ‘what to do’


B. Using standardized list of abbreviations, acronyms, and symbols
C. One-on-one oral endorsement
D. Text messaging and e-mail

Myxedema coma is a life threatening complication of long standing and untreated


hypothyroidism with one of the following characteristics.

A. Hyperglycemia
B. Hypothermia
C. Hyperthermia
D. Hypoglycemia

Mang Edgardo has a chest tube inserted in place after a Lobectomy. The nurse knows that that
Chest tube after this procedure will:

A. Prevents mediastinal shift


B. Promote chest expansion of the remaining lung
C. Drain fluids and blood accumulated post operatively
D. Remove the air in the lungs to promote lung expansion

Mrs. Pichay who is for thoracentesis is assigned by the nurse to any of the following positions,
EXCEPT:

A. straddling a chair with arms and head resting on the back of the chair
B. lying on the unaffected side with the bed elevated 30-40 degrees
C. lying prone with the head of the bed lowered 15-30 degrees
D. sitting on the edge of the bed with her feet supported and arms and head on a padded overhead
table

Chest x-ray was ordered after thoracentesis. When your client asks what is the reason for another
chest x-ray, you will explain:

A. to rule out pneumothorax


B. to rule out any possible perforation
C. to decongest
D. to rule out any foreign body

The RR nurse should monitor for the most common postoperative complication of:

A. hemorrhage
B. endotracheal tube perforation
C. osopharyngeal edema
D. epiglottis

The PACU nurse will maintain postoperative T and A client in what position?

A. Supine with neck hyperextended and supported with pillow


B. Prone with the head on pillow and turned to the side
C. Semi-fowler’s with neck flexed
D. Reverse trendelenburg with extended neck

Tony is to be discharged in the afternoon of the same day after tonsillectomy and adenoidectomy.
You as the RN will make sure that the family knows to:

A. offer osterized feeding


B. offer soft foods for a week to minimize discomfort while swallowing
C. supplement his diet with Vitamin C rich juices to enhance healing
D. offer clear liquid for 3 days to prevent irritation

Situation – Rudy was diagnosed to have chronic renal failure. Hemodialysis is ordered so that an
A-V shunt was surgically created.

Which of the following action would be of highest priority with regards to the external shunt?
A. Avoid taking BP or blood sample from the arm with the shunt
B. Instruct the client not to exercise the arm with the shunt
C. Heparinize the shunt daily
D. Change dressing of the shunt daily

Diet therapy for Rudy, who has acute renal failure is low-protein, low potassium and low sodium.
The nutrition instructions should include:

A. Recommend protein of high biologic value like eggs, poultry and lean meats
B. Encourage client to include raw cucumbers, carrot, cabbage, and tomatoes
C. Allowing the client cheese, canned foods and other processed food
D. Bananas, cantaloupe, orange and other fresh fruits can be included in the diet

The most common causative agent of Pyelonephritis in hospitalized patient attributed to


prolonged catheterization is said to be:

A. E. Coli
B. Klebsiella
C. Pseudomonas
D. Staphylococcus

The IVP reveals that Fe has small renal calculus that can be passed out spontaneously. To increase
the chance of passing the stones, you instructed her to force fluids and do which of the following?

A. Balanced diet
B. Ambulate more
C. Strain all urine
D. Bed rest

Sergio is brought to Emergency Room after the barbecue grill accident. Based on the assessment
of the physician, Sergio sustained superficial partial thickness burns on his trunk, right upper
extremities and right lower extremities. His wife asks what that means? Your most accurate
response would be:

A. Structures beneath the skin are damage


B. Dermis is partially damaged
C. Epidermis and dermis are both damaged
D. Epidermis is damaged

During the first 24 hours after the thermal injury, you should asses Sergio for:

A. hypokalemia and hypernatremia


B. hypokalemia and hyponatremia
C. hyperkalemia and hyponatremia
D. hyperkalemia and hypernatremia

All of the following are instruction for proper foot care to be given to a client with peripheral
vascular disease caused by Diabetes. Which is not?

A. Trim nail using nail clipper


B. Apply cornstarch to the foot
C. Always check for the temperature of the water before bathing
D. Use Canvas shoes

You are on morning duty in the medical ward. You have 10 patients assigned to you. During your
endorsement rounds, you found out that one of your patients was not in bed. The patient next to
him informed you that he went home without notifying the nurses. Which among the following
will you do first?

A. Make an incident report


B. Call security to report the incident
C. Wait for 2 hours before reporting
D. Report the incident to your supervisor

You are on duty in the medical ward. You were asked to check the narcotics cabinet. You found
out that what is on record does not tally with the drugs used. Which among the following will you
do first?

A. Write an incident report and refer the matter to the nursing director
B. Keep your findings to yourself
C. Report the matter to your supervisor
D. Find out from the endorsement any patient who might have been given narcotics

You are on duty in the medical ward. The mother of your patient who is also a nurse, came
running to the nurses station and informed you that Fiolo went into cardiopulmonary arrest.

A. Start basic life support measures


B. Call for the Code
C. Bring the crash cart to the room
D. Go to see Fiolo and assess for airway patency and breathing problems

When observing a return demonstration of a colostomy irrigation, you know that more teaching is
required if pt:

E. Lubricates the tip of the catheter prior to inserting into the stoma
F. Hangs the irrigating bag on the bathroom door cloth hook during fluid insertion
G. Discontinues the insertion of fluid after only 500 ml of fluid has been instilled
H. Clamps of the flow of fluid when felling uncomfortable

Which of the four phases of emergency management is defined as “sustained action that reduces
or eliminates long-term risk to people and property from natural hazards and their effects.”?

A. Recovery
B. Mitigation
C. Response
D. Preparedness

Which of the following terms refer to a process by which the individual receives education about
recognition of stress reaction and management strategies for handling stress which may be
instituted after a disaster?

A. Clinical incident stress management


B. Follow-up
C. Defriefing
D. Defusion

Fires are approached using the mnemonic RACE, in which, R stands for:

A. Run
B. Race
C. Rescue
D. Remove

You are caring for Conrad who has a brained tumor and increased Intracranial Pressure (ICP).
Which intervention should you include in your plan to reduce ICP?

A. Administer bowel softener


B. Position Conrad with his head turned toward the side of the tumor
C. Provide sensory stimulation
D. Encourage coughing and deep breathing

Keeping Conrad’s head and neck alignment results in:

A. increased inthrathoracic pressure


B. increased venous outflow
C. decreased venous outflow
D. increased intrabdominal pressure

Earliest sign of skin reaction to radiation therapy is:

A. desquamation
B. erythema
C. atrophy
D. pigmentation

A guideline that is utilized in determining priorities is to asses the status of the following,
EXCEPT:

A. perfusion C. respiration
B. locomotion D. mentation

Miss Kate is a bread vendor and you are buying a bread from her. You noticed that she receives
and changes money and then hold the bread without washing her hand. As a nurse, What will you
say to Miss Kate?

A. Miss, Don’t touch the bread I’ll be the one to pick it up


B. Miss, Please wash your hands before you pick up those breads
C. Miss, Use a pick up forceps when picking up those breads
D. Miss, Your hands are dirty I guess I’ll try another bread shop

In administering blood transfusion, what needle gauge is used?

A. 18
B. 22
C. 23
D. 24

Before administration of blood and blood products, the nurse should first:

A. Check with another R.N the client’s name, Identification number, ABO and RH type.
B. Explain the procedure to the client
C. Assess baseline vital signs of the client
D. Check for the BT order

The only IV fluid compatible with blood products is:

A. D5LR
B. D5NSS
C. NSS
D. Plain LR

In any event of an adverse hemolytic reaction during blood transfusion, Nursing intervention
should focus on:

A. Slow the infusion, Call the physician and assess the patient
B. Stop the infusion, Assess the client, Send the remaining blood to the laboratory and call the
physician
C. Stop the infusion, Call the physician and assess the client
D. Slow the confusion and keep a patent IV line open for administration of medication

The nurse knows that after receiving the blood from the blood bank, it should be administered
within:

A. 1 hour
B. 2 hours
C. 4 hours
D. 6 hours

During blood administration, the nurse should carefully monitor adverse reaction. To monitor
this, it is essential for the nurse to:

A. Stay with the client for the first 15 minutes of blood administration
B. Stay with the client for the entire period of blood administration
C. Run the infusion at a faster rate during the first 15 minutes
D. Tell the client to notify the staff immediately for any adverse reaction

As Leda’s nurse, you plan to set up an emergency equipment at her beside following
thyroidectomy. You should include:

A. An airway and rebreathing tube


B. A tracheostomy set and oxygen
C. A crush cart with bed board
D. Two ampules of sodium bicarbonate

Which of the following nursing interventions is appropriate after a total thyroidectomy?

A. Place pillows under your patient’s shoulders.


B. Raise the knee-gatch to 30 degrees
C. Keep you patient in a high-fowler’s position.
D. Support the patient’s head and neck with pillows and sandbags.

If there is an accidental injury to the parathyroid gland during a thyroidectomy which of the
following might Leda develop postoperatively?
A. Cardiac arrest C. Respiratory failure
B. Dyspnea D. Tetany

After surgery Leda develops peripheral numbness, tingling and muscle twitching and spasm.
What would you anticipate to administer?

A. Magnesium sulfate C. Potassium iodide


B. Calcium gluconate D. Potassium chloride

NURSES are involved in maintaining a safe and healthy environment. This is part of quality care
management.

The first step in decontamination is:


A. to immediately apply a chemical decontamination foam to the area of contamination
B. a through soap and water wash and rinse of the patient
C. to immediately apply personal protective equipment
D. removal of the patients clothing and jewelry and then rinsing the patient with water
E.
For a patient experiencing pruritus, you recommend which type of bath.

A. water C. saline
B. colloidal (oatmeal) D. sodium bicarbonate
Induction of vomiting is indicated for the accidental poisoning patient who has ingested.

A. rust remover C. toilet bowl cleaner


B. gasoline D. aspirin

Because severe burn can affect the person’s totality it is important that-you apply interventions
focusing on the various dimensions of man. You also have to understand the rationale of the
treatment.

A client was rushed in the E.R showing a whitish, leathery and painless burned area on his skin.
The nurse is correct in classifying this burn as:

A. First degree burn


B. Second degree burn
C. Third degree burn
E. Partial thickness burn

During the first 24 hours of burn, nursing measures should focus on which of the following?

A. I and O hourly
B. Strict aseptic technique
C. Forced oral fluids
D. Isolate the patient

During the Acute phase of burn, the priority nursing intervention in caring for this client is:

A. Prevention of infection
B. Pain management
C. Prevention of Bleeding
D. Fluid Resuscitation

The nurse knows that the most fatal electrolyte imbalance in burned client during the Emergent
phase of burn is:

A. Hypokalemia
B. Hyperkalemia
C. Hypernatremia
D. Hyponatremia

Hypokalemia is reflected in the ECG by which of the following?

A. Tall T waves
B. Widening QRS Complex
C. Pathologic Q wave
D. U wave

Pain medications given to the burn clients are best given via what route?

A. IV
B. IM
C. Oral
D. SQ

What type of debridement involves proteolytic enzymes?

A. Interventional C. Surgical
B. Mechanical D. Chemical

Which topical antimicrobial is most frequently used in burn wound care?


A. Neosporin
B. Silver nitrate
C. Silver sulfadiazine
D. Sulfamylon
Hypertrophic burn scars are caused by:

A. exaggerated contraction
B. random layering of collagen
C. wound ischemia
D. delayed epithelialization

This study which is an in depth study of one boy is a:

A. case study
B. longitudinal study
C. cross-sectional study
D. evaluative study

The process recording was the principal tool for data collection. Which of the following is NOT a
part of a process recording?

A. Non verbal narrative account


B. Analysis and interpretation
C. Audio-visual recording
D. Verbal narrative account

the most significant factor that might affect the nurse’s care for the psychiatric patient is:

A. Nurse’s own beliefs and attitude about the mentally ill


B. Amount of experience he has with psychiatric clients
C. Her abilities and skill to care for the psychiatric clients
D. Her knowledge in dealing with the psychiatric clients

In order to establish a therapeutic relationship with the client, the nurse must first have:

A. Self awareness
B. Self understanding
C. Self acceptance
D. Self motivation

Nurse Edna thinks that the patient is somewhat like his father. She then identifies positive feeling
for the patient that affects the objectivity of her nursing care. This emotional reaction is called:

A. Transference
B. Counter Transference
C. Reaction formation
D. Sympathy

the most important quality of a nurse during a Nurse-Patient interaction is:


A. Understanding
B. Acceptance
C. Listening
D. Teaching

Selective inattention is seen in what level of anxiety?

A. Mild
B. Moderate
C. Severe
D. Panic

Obsessive compulsive disorder is characterized by:

A. Uncontrollable impulse to perform an act or ritual repeatedly


B. Persistent thoughts and behavior
C. Recurring unwanted and disturbing thoughts
D. Pathological persistence of unwilled thoughts

Ms. Maria Salvacion says that she is the incarnation of the holy Virgin Mary. She said that she is
the child of the covenant that would save this world from the evil forces of Satan. One morning,
while caring for her, she stood in front of you and said “Bow down before me! I am the holy
mother of Christ! I am the blessed Virgin Mary!” The best response by the Nurse is:

A. Tell me more about being the Virgin Mary


B. So, You are the Virgin Mary?
C. Excuse me but, you are not anymore a Virgin so you cannot be the Blessed Virgin Mary.
D. You are Maria Salvacion

Maria’s statement “Bow down before me! I am the holy mother of Christ! I am the blessed Virgin
Mary!” is an example of:

A. Delusion of grandeur
B. Visual Hallucination
C. Religious delusion
D. Auditory Hallcucination

The nurse interprets the statement “Bow down before me! I am the holy mother of Christ! I am
the blessed Virgin Mary!” as important in documenting in which of the following areas of mental
status examination?

A. Thought content
B. Mood
C. Affect
D. Attitude
Mang David, A 27 year old psychiatric client was admitted with a diagnosis of schizophrenia.
During the morning assessment, Mang David shouted “Did you know that I am the top salesman
in the world? Different companies want me!” As a nurse, you know that this is an example of:

A. Hallucination
B. Delusion
C. Confabulation
D. Flight of Ideas

The recommended treatment modality in clients with obsessive compulsive disorder is:

A. Psychotherapy
B. Behavior therapy
C. Aversion therapy
D. Psychoanalysis

A state of disequilibrium wherein a person cannot readily solve a problem or situation even by
using his usual coping mechanisms is called:

A. Mental illness
B. Mental health
C. Crisis
D. Stress

Obsessive compulsive disorder is classified under:

A. Psychotic disorders
B. Neurotic disorders
C. Major depressive disorder
D. Bipolar disorder

Which nursing diagnosis is a priority for clients with Borderline personality disorder?

A. Risk for injury


B. Ineffective individual coping
C. Altered thought process
D. Sensory perceptual alteration

An appropriate nursing diagnosis for clients in the acute manic phase of bipolar disorder is:

A. Risk for injury directed to self


B. Risk for injury directed to others
C. Impaired nutrition less than body requirements
D. Ineffective individual coping
A paranoid client refuses to eat telling you that you poisoned his food. The best intervention to
this client is:

A. Taste the food in front of him and tell him that the food is not poisoned
B. Offer other types of food until the client eats
C. Simply state that the food is not poisoned
D. Offer sealed foods

Toilet training occurs in the anal stage of Freud’s psychosexual task. This is equivalent to
Erikson’s:

A. Trust vs. Mistrust


B. Autonomy vs. Shame and Doubt
C. Initiative vs. Guilt
D. Industry vs. Inferiority

During the phallic stage, the child must identify with the parent of:

A. The same sex


B. The opposite sex
C. The mother or the primary caregiver
D. Both sexes

Ms. ANA had a car accident where he lost her boyfriend. As a result, she became passive and
submissive. The nurse knows that the type of crisis Ms. ANA is experiencing is:

A. Developmental crisis
B. Maturational crisis
C. Situational crisis
D. Social Crisis

Persons experiencing crisis becomes passive and submissive. As a nurse, you know that the best
approach in crisis intervention is to be:

A. Active and Directive


B. Passive friendliness
C. Active friendliness
D. Firm kindness

The psychosocial task of a 55 year old adult client is:

A. Industry vs. Inferiority


B. Intimacy vs. Isolation
C. Integrity vs. Despair
D. Generativity vs. Stagnation
The stages of grieving identified by Elizabeth Kubler-Ross are:

A. Numbness, anger, resolution and reorganization


B. Denial, anger, identification, depression and acceptance
C. Anger, loneliness, depression and resolution
D. Denial, anger, bargaining, depression and acceptance

Which physiologic effect should the nurse expect in a client addicted to hallucinogens?

A. Dilated pupils
B. Constricted pupils
C. Bradycardia
D. Bradypnea

Miss CEE is admitted for treatment of major depression. She is withdrawn, disheveled and states
“Nobody wants me” The nurse most likely expects that Miss CEE is to be placed on:

A. Neuroleptics medication
B. Special diet
C. Suicide precaution
D. Anxiolytics medication

In alcoholic patient, the nurse knows that the vitamin deficient to these types of clients that leads
to psychoses is:

A. Thiamine
B. Vitamin C
C. Niacin
D. Vitamin A

Which of the following terms refers to weakness of both legs and the lower part of the trunk?

A. Paraparesis
B. Hemiplegia
C. Quadriparesis
D. Paraplegia

Of the following neurotransmitters, which demonstrates inhibitory action, helps control mood and
sleep, and inhibits pain pathways?

A. Serotonin
B. Enkephalin
C. Norepinephrine
D. Acetylcholine
The lobe of the brain that contains the auditory receptive areas is the ____________ lobe.

A. temporal
B. frontal
C. parietal
D. occipital

In preparation for ECT, the nurse knows that it is almost similar to that of:

A. ECG
B. General Anesthesia
C. EEG
D. MRI

The expected side effect after ECT is commonly associated with:

A. Transient loss of memory, confusion and disorientation


B. Nausea and vomiting
C. Fractures
D. Hypertension and increased in cardiac rate

The purpose of ECT in clients with depression is to:

A. Stimulation in the brain to increase brain conduction and counteract depression


B. Mainly Biologic, increasing the norepinephrine and serotonin level
C. Creates a temporary brain damage that will increase blood flow to the brain
D. Involves the conduction of electrical current to the brain to charge the neurons and combat
depression

The priority nursing diagnosis for a client with major depression is:

A. Altered nutrition
B. Altered thought process
C. Self care deficit
D. Risk for injury

A patient tells the nurse “I am depressed to talk to you, leave me alone” Which of the following
response by the nurse is most therapeutic?

A. I’ll be back in an hour


B. Why are you so depressed?
C. I’ll seat with you for a moment
D. Call me when you feel like talking to me
One of the following statements is true with regards to the care of clients with depression:

A. Only mentally ill persons commit suicide


B. All depressed clients are considered potentially suicidal
C. Most suicidal person gives no warning
D. The chance of suicide lessens as depression lessens

An adolescent client has bloodshot eyes, a voracious appetite and dry mouth. Which drug abuse
would the nurse most likely suspect?

A. Marijuana
B. Amphetamines
C. Barbiturates
D. Anxiolytics

During which phase of therapeutic relationship should the nurse inform the patient for
termination of therapy?

A. Pre orientation
B. Orientation
C. Working
D. Termination

A client says to the nurse “I am worthless person, I should be dead” The nurse best replies:

A. “Don’t say you are worthless, you are not a worthless person”
B. “We are going to help you with your feelings”
C. “What makes you feel you’re worthless?”
D. “What you say is not true”

The nurse’s most unique tool in working with the emotionally ill client is his/her

A. theoretical knowledge
B. personality make up
C. emotional reactions
D. communication skills

The mentally ill person responds positively to the nurse who is warm and caring. This is a
demonstration of the nurse’s role as:

A. counselor
B. mother surrogate
C. therapist
D. socializing agent
The past history of Camila would most probably reveal that her premorbid personality is:

A. schizoid
B. extrovert
C. ambivert
D. cycloid

In an extreme situation and when no other resident or intern is available, should a nurse receive
telephone orders, the order has to be correctly written and signed by the physician within:

A. 24 hours
B. 36 hours
C. 48 hours
D. 12 hours

. If it is established that the child is physically abused by a parent, the most important goal the
nurse could formulate with the family is that:

A. Child and any siblings will live in a safe environment


B. Family will feel comfortable in their relationship with the counselor
C. Family will gain understanding of their abusive behavior patterns
D. Mother will be able to use verbal discipline with her children

Cocaine is derived from the leaves of coca plant; the nurse knows that cocaine is classified as:

A. Narcotic
B. Stimulant
C. Barbiturate
D. Hallucinogen

To successfully complete the tasks of older adulthood, an 85 year old who has been a widow for
25 years should be encouraged to:

A. Invest her creative energies in promoting social welfare


B. Redefine her role in the society and offer something and offer something of value
C. Feel a sense of satisfaction in reflecting on her productive life
D. Look to recapture the opportunities that were never started or completed

In a therapeutic relationship, the nurse must understand own values, beliefs, feelings, prejudices
& how these affect others. This is called:

A. Therapeutic use of self


B. Psychotherapy
C. Therapeutic communication
D. Self awareness

While on Bryant’s traction, which of these observations of Graciela and her traction apparatus
would indicate a decrease in the effectiveness of her traction?

A. Graciela’s buttocks are resting on the bed.


B. The traction weights are hanging 10 inches above the floor.
C. Graciela’s legs are suspended at a 90 degree angle to her trunk.
D. The traction ropes move freely through the pulley.

The nurse notes that the fall might also cause a possible head injury. She will be observed for
signs of increased intracranial pressure which include:

A. Narrowing of the pulse pressure


B. Vomiting
C. Periorbital edema
D. A positive Kernig’s sign

This is a tricyclic antidepressant drug:

A. Venlafaxine (Effexor)
B. Flouxetine (Prozac)
C. Sertraline (Zoloft)
D. Imipramine (Tofranil)

The working phase in a therapy group is usually characterized by which of the following?

A. Caution
B. Cohesiveness
C. Confusion
D. Competition

Substance abuse is different from substance dependence in that, substance dependence:

A. includes characteristics of adverse consequences and repeated use


B. requires long term treatment in a hospital based program
C. produces less severe symptoms than that of abuse
D. includes characteristics of tolerance and withdrawal

Ricky’s IQ falls within the range of 50-55. he can be expected to:

A. Profit from vocational training with moderate supervision


B. Live successfully in the community
C. Perform simple tasks in closely supervised settings
D. Acquire academic skills of 6th grade level

The mother of a drug dependent would never consider referring her son to a drug rehabilitation
agency because she fears her son might just become worse while relating with other drug users.
The mother’s behavior can be described as:

A. Unhelpful
B. Codependent
C. Caretaking
D. Supportive

You teach your clients the difference between, Type I (IDDM) and Type II (NDDM) diabetes.
Which of the following is true?

A. both types diabetes mellitus clients are all prone to developing ketosis
B. Type II (NIDDM) is more common and is also preventable compared to Type I (IDDM)
diabetes which is genetic in etiology
C. Type I (IIDM) is characterized by fasting hyperglycemia
D. Type II (NIDDM) is characterized by abnormal immune response

Lifestyle-related diseases in general share areas common risk factors. These are the following
except:

A. physical activity
B. smoking
C. genetics
D. nutrition

The following mechanisms can be utilized as part of the quality assurance program of your
hospital EXCEPT:

A. Patient satisfaction surveys


B. Peer review to assess care provided
C. Review of clinical records of care of client
D. Use of Nursing Interventions Classification

The use of the Standards of Nursing Practice is important in the hospital. Which of the following
statements best describes what it is?

A. These are statements that describe the maximum or highest level of acceptable performance in
nursing practice
B. It refers to the scope of nursing practice as defined in Republic Act 9173
C. It is a license issued by the Professional Regulation Commission to protect the public from
substandard nursing practice
D. The Standards of Care includes the various steps of the nursing process and the standards of
professional performance

you are taking care of critically ill client and the doctor in charge calls to order a DNR (do not
resuscitate) for the client. Which of the following is the appropriate action when getting DNR
order over the phone?

A. Have the registered nurse, family spokesperson, nurse supervisor and doctor sign
B. Have 2 nurse validate the phone order, both nurses sign the order and the doctor should sign
his order within 24 hours
C. Have the registered nurse, family and doctor sign the order
D. Have 1 nurse take the order and sign it and have the doctor sign it within 24 hours

Under the PRC-Board of Nursing Resolution promulgating the adoption of a Nursing Specialty
Certification Program and Council, which two (2) of the following serves as the strongest for its
enforcement?

(a) Advances made in Science and Technology have provided the climate for specialization in
almost all aspects of human endeavor; and
(b) As necessary consequence, there has emerged a new concept known as globalization which
seeks to remove barriers in trade, industry and services imposed by the national laws of countries
all over the world; and
(c) Awareness of this development should impel the nursing sector to prepare our people in the
services sector to meet the above challenge; and
(d) Current trends of specialization in nursing practice recognized by the International Council of
Nurses (ICN) of which the Philippines is a member for the benefit of the Filipino in terms of
deepening and refining nursing practice and enhancing the quality of nursing care.

A. b & c are strong justifications


B. a & b are strong justifications
C. a & c are strong justifications
D. a & d are strong justifications

Knowing that for a comatose patient hearing is the last sense to be lost, as Judy’s nurse, what
should you do?

A. Tell her family that probably she can’t hear them


B. Talk loudly so that Wendy can hear you
C. Tell her family who are in the room not to talk
D. Speak softly then hold her hands gently

Which among the following interventions should you consider as the highest priority when caring
for June who has hemiparesis secondary to stroke?

A. Place June on an upright lateral position


B. Perform range of motion exercises
C. Apply antiembolic stockings
D. Use hand rolls or pillows for support

Salome was fitted a hearing aid. She understood the proper use and wear of this device when she
says that the battery should be functional, the device is turned on and adjusted to a:

A. therapeutic level
B. comfortable level
C. prescribed level
D. audible level

Membership dropout generally occurs in group therapy after a member:

A. Accomplishes his goal in joining the group


B. Discovers that his feelings are shared by the group members
C. Experiences feelings of frustration in the group
D. Discusses personal concerns with group members

Which of the following questions illustrates the group role of encourager?

A. What were you saying?


B. Who wants to respond next?
C. Where do you go from here?
D. Why haven’t we heard from you?

The goal of remotivation therapy is to facilitate:

A. Insight
B. Productivity
C. Socialization
D. Intimacy

Being in contact with reality and the environment is a function of the:

A. conscience
B. ego
C. id
D. super ego

Substance abuse is different from substance dependence in that, substance dependence:

E. includes characteristics of adverse consequences and repeated use


F. requires long term treatment in a hospital based program
G. produces less severe symptoms than that of abuse
H. includes characteristics of tolerance and withdrawal
During the detoxification stage, it is a priority for the nurse to:

A. teach skills to recognize and respond to health threatening situations


B. increase the client’s awareness of unsatisfactory protective behaviors
C. implement behavior modification
D. promote homeostasis and minimize the client’s withdrawal symptoms

Commonly known as “shabu” is:

A. Cannabis Sativa
B. Lysergic acid diethylamide
C. Methylenedioxy methamphetamine
D. Methampetamine hydrochloride
Wednesday, November 21, 2007

diannemaydee : ANSWER KEY 300 items reviewer

Situation – Richard has a nursing diagnosis of ineffective airway clearance related to excessive
secretions and is at risk for infection because of retained secretions. Part of Nurse Mario’s nursing
care plan is to loosen and remove excessive secretions in the airway.

Mario listens to Richard’s bilateral sounds and finds that congestion is in the upper lobes of the
lungs. The appropriate position to drain the anterior and posterior apical segments of the lungs
when Mario does percussion would be:

B. Client seated upright in bed or on a chair then leaning forward in sitting position then flat on
his back and on his abdomen

When documenting outcome of Richard’s treatment Mario should include the following in his
recording EXCEPT:

C. Amount of fluid intake of client before and after the procedure

When assessing Richard for chest percussion or chest vibration and postural drainage, Mario
would focus on the following EXCEPT:

C. Teaching the client’s relatives to perform the procedure

Mario prepares Richard for postural drainage and percussion. Which of the following is a special
consideration when doing the procedure?

B. Client can tolerate sitting and lying positions

The purpose of chest percussion and vibration is to loosen secretions in the lungs. The difference
between the procedures is:

B. Percussion delivers cushioned blows to the chest with cupped palms while vibration gently
shakes secretion loose on the exhalation cycle

Situation – A 61 year old man, Mr. Regalado, is admitted to the private ward for observation after
complaints of severe chest pain. You are assigned to take care of the client.

When doing an initial assessment, the best way for you to identify the client’s priority problem is
to:

D. Do a physical examination while asking the client relevant questions

Nancy blames God for her situation. She is easily provoked to tears and wants to be left alone,
refusing to eat or talk to her family. A religious person before, she now refuses to pray or go to
church stating that God has abandoned her. The nurse understands that Nancy is grieving for her
self and is in the stage of:

C. anger

Which of the following ethical principles refers to the duty to do good?


A. Beneficence

During which step of the nursing process does the nurse analyze data related to the patient's
health status?

A. Assessment

The basic difference between nursing diagnoses and collaborative problems is that

A. nurses manage collaborative problems using physician-prescribed interventions.

Situation – Mrs. Seva, 52 years old, asks you about possible problems regarding her elimination
now that she is in the menopausal stage.

Instruction on health promotion regarding urinary elimination is important. Which would you
include?

A. Hold urine as long as she can before emptying the bladder to strengthen her sphincter muscles

Mrs. Seva also tells the nurse that she is often constipated. Because she is aging, what physical
changes predispose her to constipation?

C. loss of tone of the smooth muscles of the colon

The nurse understands that one of these factors contributes to constipation:

D. prolonged use of laxatives

You will do nasopharyngeal suctioning on Mr. Abad. Your guide for the length of insertion of the
tubing for an adult would be:

C. the distance from the tip of the nose to the tip of the ear lobe

situation– Mr. Dizon, 84 years old, brought to the Emergency Room for complaint of
hypertension, flushed face, severe headache, and nausea. You are doing the initial assessment of
vital signs.

You are to measure the client’s initial blood pressure reading by doing all of the following
EXCEPT:

C. Pump the cuff to around 50 mmHg above the point where the pulse is obliterated

A pulse oximeter is attached to Mr. Dizon’s finger to:

D. Detect oxygen saturation of arterial blood before symptoms of hypoxemia develops


In which type of shock does the patient experiences a mismatch of blood flow to the cells?

A. Distributive

The preferred route of administration of medication in the most acute care situations is which of
the following routes?

A. Intravenous

After a few hours in the Emergency Room, Mr. Dizon is admitted to the ward with an order of
hourly monitoring of blood pressure. The nurse finds that the cuff is too narrow and this will
cause the blood pressure reading to be:

C. higher than what the reading should be

Through the client’s health history, you gather that Mr. Dizon smokes and drinks coffee. When
taking the blood pressure of a client who recently smoked or drank coffee, how long should the
nurse wait before taking the client’s blood pressure for accurate reading?

B. 30 minutes

While the client has pulse oximeter on his fingertip, you notice that the sunlight is shining on the
area where the oximeter is. Your action will be to:

C. Cover the fingertip sensor with a towel or bedsheet

When taking blood pressure reading the cuff should be:

D. inflated to 30 mmHg above the estimated systolic BP based on palpation of radial or bronchial
artery

-this is a typo error of brachial artery

To ensure client safety before starting blood transfusions the following are needed before the
procedure can be done EXCEPT:

B. blood should be warmed to room temperature for 30 minutes before blood transfusions is
administered

Mr. Bruno asks what the “normal” allowable salt intake is. Your best response to Mr. Bruno is:
D. 1 tsp of salt/day but no patis and toyo

Which of the following methods is the best method for determining nasogastric tube placement in
the stomach?

A. X-ray

Which of the following is the most important risk factor for development of Chronic Obstructive
Pulmonary Disease?

A. Cigarette smoking

When performing endotracheal suctioning, the nurse applies suctioning while withdrawing and
gently rotating the catheter 360 degrees for which of the following time periods?

A. 10-15 seconds

The nurse auscultates the apex beat at which of the following anatomical locations?

A. Fifth intercostal space, midclavicular line

Which of the following terms describes the amount of blood ejected per heartbeat?

A. Stroke volume

You are to apply a transdermal patch of nitoglycerin to your client. The following are important
guidelines to observe EXCEPT:

B. Patches may be applied to distal part of the extremities like forearm

The GAUGE size in ET tubes determines:

B. The internal diameter of the tube

The nurse is correct in performing suctioning when she applies the suction intermittently during:

B. Withdrawing of the suction catheter

The purpose of the cuff in Tracheostomy tube is to:

A. Separate the upper and lower airway


the placement of the tube
Which priority nursing diagnosis is applicable for a patient with indwelling urinary catheter?

D. Risk for infection

An incontinent elderly client frequently wets his bed and eventually develop redness and skin
excoriation at the perianal area. The best nursing goal for this client is to:

D. Keep the patient clean and dry

As a Nurse Manager, DMLM enjoys her staff of talented and self motivated individuals. She
knew that the leadership style to suit the needs of this kind of people is called:

D. Laissez Faire

A fire has broken in the unit of DMLM R.N. The best leadership style suited in cases of
emergencies like this is:

A. Autocratic

Which step of the management process is concerned with Policy making and Stating the goals
and objective of the institution?

A. Planning

In the management process, the periodic checking of the results of action to make sure that it
coincides with the goal of the institution is termed as:

B. Evaluating

The Vision of a certain agency is usually based on their beliefs, Ideals and Values that directs the
organization. It gives the organization a sense of purpose. The belief, Ideals and Values of this
Agency is called:

A. Philosophy

Mr. CKK is unconscious and was brought to the E.R. Who among the following can give consent
for CKK’s Operation?

C. Next of Kin

Mang Carlos has been terminally ill for 5 years. He asked his wife to decide for him when he is
no longer capable to do so. As a Nurse, You know that this is called:

D. Durable Power of Attorney

Mang Carlos has a standing DNR order. He then suddenly stopped breathing and you are at his
bedside. You would:

B. Stay with Mang Carlos and Do nothing

It is not a legally binding document but nevertheless, Very important in caring for the patients.

B. Patient’s Bill of Rights

In monitoring the patient in PACU, the nurse correctly identify that checking the patient’s vital
signs is done every:

C. 15 minutes

diannemayde R.N is conducting a research on her unit about the effects of effective nurse-patient
communication in decreasing anxiety of post operative patients. Which of the following step in
nursing research should she do next?

A. Review of related literature

Before diannemaydee perform the formal research study, what do you call the pre testing, small
scale trial run to determine the effectiveness of data collection and methodological problem that
might be encountered?

E. Pilot Study

On the study “effects of effective nurse-patient communication in decreasing anxiety of post


operative patients” What is the Independent variable?

A. Effective Nurse-patient communication

On the study “effects of effective nurse-patient communication in decreasing anxiety of post


operative patients” What is the Dependent variable?

C. Anxiety level

In the recent technological innovations, which of the following describe researches that are made
to improve and make human life easier?
C. Applied research

Which of the following is not true about a Pure Experimental research?

D. There is a careful selection of subjects in the experimental group

When Mrs. Guevarra, a nurse, delegates aspects of the clients care to the nurse-aide who is an
unlicensed staff, Mrs. Guevarra

B. is assigning the responsibility to the aide but not the accountability for those tasks

Process of formal negotiations of working conditions between a group of registered nurses and
employer is

C. collective bargaining

You are attending a certification on cardiopulmonary resuscitation (CPR) offered and required by
the hospital employing you. This is

B. inservice education

The law which regulated the practice of nursing profession in the Philippines is:

A. R.A 9173

This quality is being demonstrated by a Nurse who raise the side rails of a confuse and
disoriented patient?

C. Prudence

Nurse Joel and Ana is helping a 16 year old Nursing Student in a case filed against the student.
The case was frustrated homicide. Nurse Joel and Ana are aware of the different circumstances of
crimes. They are correct in identifying which of the following Circumstances that will be best
applied in this case?

C. Mitigating

In signing the consent form, the nurse is aware that what is being observed as an ethical
consideration is the patient’s

A. Autonomy

Why is there an ethical dilemma?


C. Because the patient’s right coincide with the nurse’s responsibility

Who among the following can work as a practicing nurse in the Philippines without taking the
Licensure examination?

D. Those involved in medical mission who’s services are for free

In signing the consent form, the nurse is aware that what is being observed as an ethical
consideration is the patient’s

A. Autonomy

Nurse Buddy gave Inapsine instead of Insulin to a patient in severe hyperglycemia. He reported
the incident as soon as he knew there was an error. A nurse that is always ready to answer for all
his actions and decision is said to be:

A. Accountable

Which of the following best describes Primary Nursing?

C. A registered nurse is responsible for a group of patients from admission to discharge

The best and most effective method in times of staff and financial shortage is:

A. Functional Method

You are doing bed bath to the client when suddenly, The nursing assistant rushed to the room and
tell you that the client from the other room was in Pain. The best intervention in such case is:

A. Raise the side rails, cover the client and put the call bell within reach and then attend to the
client in pain to give the PRN medication

Angie is a disoriented client who frequently falls from the bed. As her nurse, which of the
following is the best nursing intervention to prevent future falls?

D. Put the bed in the lowest position ever

When injecting subcutaneous injection in an obese patient, It should be angled at around:

B. 90 °

The following statements are all true about Z-Track technique except:
C. It forces the medication to be contained at the subcutaneous tissues

Communication is best undertaken if barriers are first removed. Considering this statement,
which of the following is considered as deterrent factor in communication?

A. Not universally accepted abbreviations

Nurse DMLM is correct in identifying the correct sequence of events during abdominal
assessment if she identifies which of the following?

A. Inspection, Auscultation, Percussion, Palpation

To prevent injury and strain on the muscles, the nurse should observe proper body mechanics.
Among the following, which is a principle of proper body mechanics?

A. Broaden the space between the feet

In taking the client’s blood pressure, the nurse should position the client’s arm:

A. At the level of the heart

What principle is used when the client with fever loses heat through giving cooling bed bath to
lower body temperature?

D. Conduction

The most effective way in limiting the number of microorganism in the hospital is:

D. Handwashing

The immunoglobulin of the mother that crosses the placenta to protect the child is an example of:

B. Natural passive immunity

Richard is a subject of a research lead by his doctor. The nurse knows that all of the following is a
correct understanding as his right as a research subject except:

C. I must choose another doctor if I withdrew from this research

Which of the following is a normal finding during assessment of a Chest tube in a 3 way bottle
system?

C. The water fluctuates during inhalation of the patient


In obtaining a urine specimen for culture and sensitivity on a catheterized patient, the nurse is
correct if:

B. Alcoholize the self sealing port, obtain a sterile syringe and draw the specimen on the self
sealing port

Which of the following is an example of secondary prevention?

B. Screening patients for hypertension

Which of the following is a form of primary prevention?

D. Immunization

An abnormal condition in which a person must sit, stand or use multiple pillows when lying down
is:

A. Orthopnea

As a nurse assigned for care for geriatric patients, you need to frequently assess your patient
using the nursing process. Which of the following needs be considered with the highest priority?

D. Physiologic needs that are life threatening

The component that should receive the highest priority before physical examination is the:

A. Psychological preparation of the client

Legally, Patients chart are:

C. Owned by the hospital and should not be given to anyone who request it other than the doctor
in charge

Which of the following categories identifies the focus of community/public health nursing
practice?

A. Promoting and maintaining the health of populations and preventing and minimizing the
progress of disease

A major goal for home care nurses is

A. restoring maximum health function.


A written nursing care plan is a tool that:

B. Gives quality nursing care

Gina, A client in prolong labor said she cannot go on anymore. The health care team decided that
both the child and the mother cannot anymore endure the process. The baby is premature and has
a little chance of surviving. Caesarian section is not possible because Gina already lost enough
blood during labor and additional losses would tend to be fatal. The husband decided that Gina
should survive and gave his consent to terminate the fetus. The principle that will be used by the
health care team is:

D. Double effect

Situation – There are various developments in health education that the nurse should know about:

The provision of health information in the rural areas nationwide through television and radio
programs and video conferencing is referred to as:

B. Telehealth program

In teaching the sister of a diabetic client about the proper use of a glucometer in determining the
blood sugar level of the client, The nurse is focusing in which domain of learning according to
bloom?

C. Psychomotor

A nearby community provides blood pressure screening, height and weight measurement,
smoking cessation classes and aerobics class services. This type of program is referred to as

D. wellness program

After cleaning the abrasions and applying antiseptic, the nurse applies cold compress to the
swollen ankle as ordered by the physician. This statement shows that the nurse has correct
understanding of the use of cold compress:

C. Cold compress prevents edema and reduces pain

After receiving prescription for pain medication, Ronnie is instructed to continue applying 30
minute cold at home and start 30 minute hot compress the next day. You explain that the use of
hot compress:

C. Increase oxygenation to the injured tissues for better healing

Situation – A nursing professor assigns a group of students to do data gathering by interviewing


their classmates as subjects.
She instructed the interviewees not to tell the interviewees that the data gathered are for her own
research project for publication. This teacher has violated the student’s right to:

B. Disclosure

Before the nurse researcher starts her study, she analyzes how much time, money, materials and
people she will need to complete the research project. This analysis prior to beginning the study is
called:

B. Feasibility

Data analysis is to be done and the nurse researcher wants to include variability. These include
the following EXCEPT:

D. Mean

Nurse Minette needs to schedule a first home visit to OB client Leah. When is a first home-care
visit typically made?

B. Within 24 hours after discharge

By force of law, therefore, the PRC-Board of Nursing released Resolution No. 14 Series of 1999
entitled: “Adoption of a Nursing Specialty Certification Program and Creation of Nursing
Specialty Certification Council.” This rule-making power is called:

C. Quasi-Legislative Power

Anita is performing BSE and she stands in front of the Mirror. The rationale for standing in front
of the mirror is to check for:

C. The Size and Contour of the breast

An emerging technique in screening for Breast Cancer in developing countries like the
Philippines is:

C. BSE Once a month

Transmission of HIV from an infected individual to another person occurs:

A. Most frequently in nurses with needlesticks

After a vaginal examination, the nurse determines that the client’s fetus is in an occiput posterior
position. The nurse would anticipate that the client will have:
B. Intense back pain

The rationales for using a prostaglandin gel for a client prior to the induction of labor is to:

A. Soften and efface the cervix

Nurse Lorena is a Family Planning and Infertility Nurse Specialist and currently attends to
FAMILY PLANNING CLIENTS AND INFERTILE COUPLES. The following conditions pertain
to meeting the nursing needs of this particular population group.

Dina, 17 years old, asks you how a tubal ligation prevents pregnancy. Which would be the best
answer?

C. Sperm can no longer reach the ova, because the fallopian tubes are blocked

The Dators are a couple undergoing testing for infertility. Infertility is said to exist when:

C. a couple has been trying to conceive for 1 year

The correct temperature to store vaccines in a refrigerator is:

B. between 2 deg C and +8 deg C

Which of the following vaccines is not done by intramuscular (IM) injection?

A. Measles vaccine

This vaccine content is derived from RNA recombinants.

C. Hepatitis B vaccines

This special form is used when the patient is admitted to the unit. The nurse completes the
information in this record particularly his/her basic personal data, current illness, previous health
history, health history of the family, emotional profile, environmental history as well as physical
assessment together with nursing diagnosis on admission. What do you call this record?

B. Nursing Health History and Assessment Worksheet

These are sheets/forms which provide an efficient and time saving way to record information that
must be obtained repeatedly at regular and/or short intervals of time. This does not replace the
progress notes; instead this record of information on vital signs, intake and output, treatment,
postoperative care, post partum care, and diabetic regimen, etc. This is used whenever specific
measurements or observations are needed to be documented repeatedly. What is this?

B. Graphic Flow Sheets

These records show all medications and treatment provided on a repeated basis. What do you call
this record?

D. Medicine and Treatment Record

This flip-over card is usually kept in a portable file at the Nurse’s Station. It has 2-parts: the
activity and treatment section and a nursing care plan section. This carries information about
basic demographic data, primary medical diagnosis, current orders of the physician to be carried
out by the nurse, written nursing care plan, nursing orders, scheduled tests and procedures, safety
precautions in patient care and factors related to daily living activities. This record is used in the
charge-of-shift reports or during the bedside rounds or walking rounds. What record is this?

D. Nursing Kardex

Most nurses regard this conventional recording of the date, time, and mode by which the patient
leaves a healthcare unit but this record includes importantly, directs of planning for discharge that
starts soon after the person is admitted to a healthcare institution. It is accepted that collaboration
or multidisciplinary involvement (of all members of the health team) in discharge results in
comprehensive care. What do you call this?

A. Discharge Summary

Based on the Code of Ethics for Filipino Nurses, what is regarded as the hallmark of nursing
responsibility and accountability?

D. Accurate documentation of actions and outcomes

A nurse should be cognizant that professional programs for specialty certification by the Board of
Nursing accredited through the:

B. Nursing Specialty Certification Council

Integrated management for childhood illness is the universal protocol of care endorsed by WHO
and is use by different countries of the world including the Philippines. In any case that the nurse
classifies the child and categorized the signs and symptoms in PINK category, You know that this
means:

A. Urgent referral

You know that fast breathing of a child age 13 months is observed if the RR is more than:
A. 40

Angelo, An 8 month old child is brought to the health care facility with sunken eyes. You pinch
his skin and it goes back very slowly. In what classification of dehydration will you categorize
Angelo?

C. Severe Dehydration

In responding to the care concerns of children with severe disease, referral to the hospital is of the
essence especially if the child manifests which of the following?

D. Difficulty to awaken

A child with ear problem should be assessed for the following, EXCEPT:

A. is there any fever?

If the child does not have ear problem, using IMCI, what should you as the nurse do?

D. Go to the next question, check for malnutrition

All of the following are treatment for a child classified with no dehydration except:

A. 1,000 ml to 1,400 ml be given within 4 hours

An ear infection that persists but still less than 14 days is classified as:

C. Acute Ear Infection

If a child has two or more pink signs, you would classify the child as having:

very severe disease

The nurse knows that the most common complication of Measles is:

A Pneumonia and larynigotracheitis

A client scheduled for hysterosalpingography needs health teaching before the procedure. The
nurse is correct in telling the patient that:

D. Flushing sensation is felt as the dye in injected


In a population of 9,500. What is your estimate of the population of pregnant woman needing
tetanus toxoid vaccination?

D. 332.5

All of the following are seen in a child with measles. Which one is not?

C. Pustule

Mobilizing the people to become aware of their own problem and to do actions to solve it is
called:

A. Community Organizing

Prevention of work related accidents in factories and industries are responsibilities of which field
of nursing?

C. Occupational health nursing

In one of your home visit to Mr. JUN, you found out that his son is sick with cholera. There is a
great possibility that other member of the family will also get cholera. This possibility is a/an:

B. Health threat

Why is bleeding in the leg of a pregnant woman considered as an emergency?

D. The pressure of the gravid uterus will exert additional force thus, increasing the blood loss in
the lower extremities

Aling Maria is nearing menopause. She is habitually taking cola and coffee for the past 20 years.
You should tell Aling Maria to avoid taking caffeinated beverages because:

C. It will contribute to additional bone demineralization

All of the following are contraindication when giving Immunization except:

B. BCG Vaccine can be given to a child with Hepatitis B

Theresa, a mother with a 2 year old daughter asks, “at what age can I be able to take the blood
pressure of my daughter as a routine procedure since hypertension is common in the family?”
Your answer to this is:

C. When she’s 3 years old

Baby John develops hyperbilirubinemia. What is a method used to treat hyperbilirubinemia in a


newborn?

D. Early feeding to speed passage of meconium

The community/Public Health Bag is:

B. an essential and indispensable equipment of the community health nurse

What is the rationale in the use of bag technique during home visits?

B. It saves time and effort of the nurse in the performance of nursing procedures

In consideration of the steps in applying the bag technique, which side of the paper lining of the
CHN bag is considered clean to make a non-contaminated work area?

D. The inside surface

How many words does a typical 12-month-old infant use?

D. Two, plus “mama” and “papa”

During operation, The OR suite’s lighting, noise, temperature and other factors that affects the
operation are managed by whom?

C. Circulating nurse

Before and after the operation, the operating suite is managed by the:

C. Nurse Manager

The counting of sponges is done by the Surgeon together with the:

B. Scrub nurse

The OR team performs distinct roles for one surgical procedure to be accomplished within a
prescribed time frame and deliver a standard patient outcome. While the surgeon performs the
surgical procedure, who monitors the status of the client like urine output, blood loss?

C. Anaesthesiologist

Surgery schedules are communicated to the OR usually a day prior to the procedure by the nurse
of the floor or ward where the patient is confined. For orthopedic cases, what department is
usually informed to be present in the OR?

D. Radiology department

In some hip surgeries, an epidural catheter for Fentanyl epidural analgesia is given. What is your
nursing priority care in such a case?

D. Assess respiratory rate carefully

The patient’s medical record can work as a double edged sword. When can the medical record
become the doctor’s/nurse’s worst enemy?

D. When the medical record is inaccurate, incomplete, and inadequate

Disposal of medical records in government hospitals/institutions must be done in close


coordination with what agency?

D. Department of Health (DOH)

In the hospital, when you need the medical record of a discharged patient for research you will
request permission through:

D. Medical records section

You will give health instructions to Carlo, a case of bronchial asthma. The health instruction will
include the following, EXCEPT:

D. Practice respiratory isolation

As the head nurse in the OR, how can you improve the effectiveness of clinical alarm systems?

D. Implement a regular maintenance and testing of alarm systems

Overdosage of medication or anesthetic can happen even with the aid of technology like infusion
pumps, sphygmomanometer and similar devices/machines. As a staff, how can you improve the
safety of using infusion pumps?

A. Check the functionality of the pump before use


While team effort is needed in the OR for efficient and quality patient care delivery, we should
limit the number of people in the room for infection control. Who comprise this team?

B. Surgeon, assistants, scrub nurse, circulating nurse, anesthesiologist

When surgery is on-going, who coordinates the activities outside, including the family?

B. Nurse Supervisor

The breakdown in teamwork is often times a failure in:

D. Communication

To prevent recurrent attacks on client with glomerulonephritis, the nurse instructs the client to:

D. Seek early treatment for respiratory infection

When administering Tapazole, The nurse should monitor the client for which of the following
adverse effect?

B. Hypothyroidism

Post bronchoscopy, the nurse priority is to check which of the following before feeding?

A. Gag reflex

Changes normally occur in the elderly. Among the following, which is a normal change in an
elderly client?

C. Urinary frequency

Colostomy is a surgically created anus. It can be temporary or permanent, depending on the


disease condition.

Skin care around the stoma is critical. Which of the following is not indicated as a skin care
barriers?

A. Apply liberal amount of mineral oil to the area

What health instruction will enhance regulation of a colostomy (defecation) of clients?

C. Eat balanced meals at regular intervals


After ileostomy, which of the following condition is NOT expected?

A. Increased weight

The following are appropriate nursing interventions during colostomy irrigation, EXCEPT:

A. Increase the irrigating solution flow rate when abdominal cramps is felt

What sensation is used as a gauge so that patients with ileostomy can determine how often their
pouch should be drained?

B. Sensation of pressure

In performing a cleansing enema, the nurse performs the procedure by positioning the client in:

D. Left sim’s position

Mang Caloy is scheduled to have a hemorrhoidectomy, after the operation, you would expect that
the client’s position post operatively will be:

B. Side lying position

You would expect that after an abdominal perineal resection, the type of colostomy that will be
use is?

C. Permanent colostomy

You are an ostomy nurse and you know that colostomy is defined as:

A. It is an incision into the colon to create an artificial opening to the exterior of the abdomen

Larry, 55 years old, who is suspected of having colorectal cancer, is admitted to the CI. After
taking the history and vital signs the physician does which test as a screening test for colorectal
cancer.

D. Proctosigmoidoscopy

Symptoms associated with cancer of the colon include:


C. blood in the stools, anemia, and “pencil shaped” stools

24 Hours after creation of colostomy, Nurse Violy is correct if she identify that the normal
appearance of the stoma is :

D. Red, moist and slightly protruding from the abdomen

In cleaning the stoma, the nurse would use which of the following cleaning mediums?

D. Mild soap and water

When observing a return demonstration of a colostomy irrigation, you know that more teaching is
required if pt:

B. Hangs the irrigating bag on the bathroom door cloth hook during fluid insertion

What does a sample group represent?

C. General population

As a nurse, you can help improve the effectiveness of communication among healthcare givers
by:

B. Using standardized list of abbreviations, acronyms, and symbols

Myxedema coma is a life threatening complication of long standing and untreated


hypothyroidism with one of the following characteristics.

B. Hypothermia

Mang Edgardo has a chest tube inserted in place after a Lobectomy. The nurse knows that that
Chest tube after this procedure will:

C. Drain fluids and blood accumulated post operatively

Mrs. Pichay who is for thoracentesis is assigned by the nurse to any of the following positions,
EXCEPT:

C. lying prone with the head of the bed lowered 15-30 degrees
Chest x-ray was ordered after thoracentesis. When your client asks what is the reason for another
chest x-ray, you will explain:

A. to rule out pneumothorax

The RR nurse should monitor for the most common postoperative complication of:

A. hemorrhage

The PACU nurse will maintain postoperative T and A client in what position?

B. Prone with the head on pillow and turned to the side

Tony is to be discharged in the afternoon of the same day after tonsillectomy and adenoidectomy.
You as the RN will make sure that the family knows to:

B. offer soft foods for a week to minimize discomfort while swallowing

Situation – Rudy was diagnosed to have chronic renal failure. Hemodialysis is ordered so that an
A-V shunt was surgically created.

Which of the following action would be of highest priority with regards to the external shunt?

A. Avoid taking BP or blood sample from the arm with the shunt

Diet therapy for Rudy, who has acute renal failure is low-protein, low potassium and low sodium.
The nutrition instructions should include:

A. Recommend protein of high biologic value like eggs, poultry and lean meats

The most common causative agent of Pyelonephritis in hospitalized patient attributed to


prolonged catheterization is said to be:

A. E. Coli

The IVP reveals that Fe has small renal calculus that can be passed out spontaneously. To increase
the chance of passing the stones, you instructed her to force fluids and do which of the following?

B. ambulate more

Sergio is brought to Emergency Room after the barbecue grill accident. Based on the assessment
of the physician, Sergio sustained superficial partial thickness burns on his trunk, right upper
extremities and right lower extremities. His wife asks what that means? Your most accurate
response would be:

C. Epidermis and dermis are both damaged

During the first 24 hours after the thermal injury, you should asses Sergio for:

C. hyperkalemia and hyponatremia

All of the following are instruction for proper foot care to be given to a client with peripheral
vascular disease caused by Diabetes. Which is not?

D. Use Canvas shoes

You are on morning duty in the medical ward. You have 10 patients assigned to you. During your
endorsement rounds, you found out that one of your patients was not in bed. The patient next to
him informed you that he went home without notifying the nurses. Which among the following
will you do first?

D. Report the incident to your supervisor

You are on duty in the medical ward. You were asked to check the narcotics cabinet. You found
out that what is on record does not tally with the drugs used. Which among the following will you
do first?

C. Report the matter to your supervisor

You are on duty in the medical ward. The mother of your patient who is also a nurse, came
running to the nurses station and informed you that Fiolo went into cardiopulmonary arrest.

C. Bring the crash cart to the room

When observing a return demonstration of a colostomy irrigation, you know that more teaching is
required if pt:

F. Hangs the irrigating bag on the bathroom door cloth hook during fluid insertion

Which of the four phases of emergency management is defined as “sustained action that reduces
or eliminates long-term risk to people and property from natural hazards and their effects.”?

B. Mitigation
Which of the following terms refer to a process by which the individual receives education about
recognition of stress reaction and management strategies for handling stress which may be
instituted after a disaster?

D. Defusion

Fires are approached using the mnemonic RACE, in which, R stands for:

C. Rescue

You are caring for Conrad who has a brained tumor and increased Intracranial Pressure (ICP).
Which intervention should you include in your plan to reduce ICP?

A. Administer bowel softener

Keeping Conrad’s head and neck alignment results in:

B. increased venous outflow

Earliest sign of skin reaction to radiation therapy is:

B. erythema

A guideline that is utilized in determining priorities is to asses the status of the following,
EXCEPT:

D. mentation

Miss Kate is a bread vendor and you are buying a bread from her. You noticed that she receives
and changes money and then hold the bread without washing her hand. As a nurse, What will you
say to Miss Kate?

C. Miss, Use a pick up forceps when picking up those breads

In administering blood transfusion, what needle gauge is used?

A. 18

Before administration of blood and blood products, the nurse should first:

A. Check with another R.N the client’s name, Identification number, ABO and RH type.

The only IV fluid compatible with blood products is:

C. NSS
In any event of an adverse hemolytic reaction during blood transfusion, Nursing intervention
should focus on:

B. Stop the infusion, Assess the client, Send the remaining blood to the laboratory and call the
physician

The nurse knows that after receiving the blood from the blood bank, it should be administered
within:

C. 4 hours

During blood administration, the nurse should carefully monitor adverse reaction. To monitor
this, it is essential for the nurse to:

A. Stay with the client for the first 15 minutes of blood administration

As Leda’s nurse, you plan to set up an emergency equipment at her beside following
thyroidectomy. You should include:

B. A tracheostomy set and oxygen

Which of the following nursing interventions is appropriate after a total thyroidectomy?

D. Support the patient’s head and neck with pillows and sandbags.

If there is an accidental injury to the parathyroid gland during a thyroidectomy which of the
following might Leda develop postoperatively?

D. Tetany

After surgery Leda develops peripheral numbness, tingling and muscle twitching and spasm.
What would you anticipate to administer?

B. Calcium gluconate

NURSES are involved in maintaining a safe and healthy environment. This is part of quality care
management.

The first step in decontamination is:


D. removal of the patients clothing and jewelry and then rinsing the patient with water

For a patient experiencing pruritus, you recommend which type of bath.

B. colloidal (oatmeal)

Induction of vomiting is indicated for the accidental poisoning patient who has ingested.

D. aspirin

Because severe burn can affect the person’s totality it is important that-you apply interventions
focusing on the various dimensions of man. You also have to understand the rationale of the
treatment.

A client was rushed in the E.R showing a whitish, leathery and painless burned area on his skin.
The nurse is correct in classifying this burn as:

C. Third degree burn

During the first 24 hours of burn, nursing measures should focus on which of the following?

A. I and O hourly

During the Acute phase of burn, the priority nursing intervention in caring for this client is:

D. Fluid Resuscitation

The nurse knows that the most fatal electrolyte imbalance in burned client during the Emergent
phase of burn is:

B. Hyperkalemia

Hypokalemia is reflected in the ECG by which of the following?

D. U wave

Pain medications given to the burn clients are best given via what route?

A. IV
What type of debridement involves proteolytic enzymes?

D. Chemical

Which topical antimicrobial is most frequently used in burn wound care?

D. Sulfamylon

Hypertrophic burn scars are caused by:

B. random layering of collagen

This study which is an in depth study of one boy is a:

A. case study

The process recording was the principal tool for data collection. Which of the following is NOT a
part of a process recording?

C. Audio-visual recording

the most significant factor that might affect the nurse’s care for the psychiatric patient is:

A. Nurse’s own beliefs and attitude about the mentally ill

In order to establish a therapeutic relationship with the client, the nurse must first have:

A. Self awareness

Nurse Edna thinks that the patient is somewhat like his father. She then identifies positive feeling
for the patient that affects the objectivity of her nursing care. This emotional reaction is called:

B. Counter Transference

the most important quality of a nurse during a Nurse-Patient interaction is:

C. Listening

Selective inattention is seen in what level of anxiety?

B. Moderate
Obsessive compulsive disorder is characterized by:

B. Persistent thoughts and behavior

Ms. Maria Salvacion says that she is the incarnation of the holy Virgin Mary. She said that she is
the child of the covenant that would save this world from the evil forces of Satan. One morning,
while caring for her, she stood in front of you and said “Bow down before me! I am the holy
mother of Christ! I am the blessed Virgin Mary!” The best response by the Nurse is:

D. You are Maria Salvacion

Maria’s statement “Bow down before me! I am the holy mother of Christ! I am the blessed Virgin
Mary!” is an example of:

C. Religious delusion

The nurse interprets the statement “Bow down before me! I am the holy mother of Christ! I am
the blessed Virgin Mary!” as important in documenting in which of the following areas of mental
status examination?

A. Thought content

Mang David, A 27 year old psychiatric client was admitted with a diagnosis of schizophrenia.
During the morning assessment, Mang David shouted “Did you know that I am the top salesman
in the world? Different companies want me!” As a nurse, you know that this is an example of:

B. Delusion

The recommended treatment modality in clients with obsessive compulsive disorder is:

A. Psychotherapy

A state of disequilibrium wherein a person cannot readily solve a problem or situation even by
using his usual coping mechanisms is called:

C. Crisis

Obsessive compulsive disorder is classified under:

B. Neurotic disorders
Which nursing diagnosis is a priority for clients with Borderline personality disorder?

A. Risk for injury

An appropriate nursing diagnosis for clients in the acute manic phase of bipolar disorder is:

B. Risk for injury directed to others

A paranoid client refuses to eat telling you that you poisoned his food. The best intervention to
this client is:

D. Offer sealed foods

Toilet training occurs in the anal stage of Freud’s psychosexual task. This is equivalent to
Erikson’s:

B. Autonomy vs. Shame and Doubt

During the phallic stage, the child must identify with the parent of:

A. The same sex

Ms. ANA had a car accident where he lost her boyfriend. As a result, she became passive and
submissive. The nurse knows that the type of crisis Ms. ANA is experiencing is:

C. Situational crisis

Persons experiencing crisis becomes passive and submissive. As a nurse, you know that the best
approach in crisis intervention is to be:

A. Active and Directive

The psychosocial task of a 55 year old adult client is:

D. Generativity vs. Stagnation

The stages of grieving identified by Elizabeth Kubler-Ross are:

A. Numbness, anger, resolution and reorganization

Which physiologic effect should the nurse expect in a client addicted to hallucinogens?
A. Dilated pupils
cardia
D. Bradypnea

Miss CEE is admitted for treatment of major depression. She is withdrawn, disheveled and states
“Nobody wants me” The nurse most likely expects that Miss CEE is to be placed on:

C. Suicide precaution

In alcoholic patient, the nurse knows that the vitamin deficient to these types of clients that leads
to psychoses is:

A. Thiamine

Which of the following terms refers to weakness of both legs and the lower part of the trunk?

A. Paraparesis

Of the following neurotransmitters, which demonstrates inhibitory action, helps control mood and
sleep, and inhibits pain pathways?

A. Serotonin

The lobe of the brain that contains the auditory receptive areas is the ____________ lobe.

A. temporal

In preparation for ECT, the nurse knows that it is almost similar to that of:

B. General Anesthesia

The expected side effect after ECT is commonly associated with:

A. Transient loss of memory, confusion and disorientation

The purpose of ECT in clients with depression is to:

B. Mainly Biologic, increasing the norepinephrine and serotonin level

The priority nursing diagnosis for a client with major depression is:

D. Risk for injury


A patient tells the nurse “I am depressed to talk to you, leave me alone” Which of the following
response by the nurse is most therapeutic?

A. I’ll be back in an hour

One of the following statements is true with regards to the care of clients with depression:

B. All depressed clients are considered potentially suicidal

An adolescent client has bloodshot eyes, a voracious appetite and dry mouth. Which drug abuse
would the nurse most likely suspect?

A. Marijuana

During which phase of therapeutic relationship should the nurse inform the patient for
termination of therapy?

B. Orientation

A client says to the nurse “I am worthless person, I should be dead” The nurse best replies:

C. “What makes you feel you’re worthless?”

The nurse’s most unique tool in working with the emotionally ill client is his/her

D. communication skills

The mentally ill person responds positively to the nurse who is warm and caring. This is a
demonstration of the nurse’s role as:

B. mother surrogate

The past history of Camila would most probably reveal that her premorbid personality is:

A. schizoid

In an extreme situation and when no other resident or intern is available, should a nurse receive
telephone orders, the order has to be correctly written and signed by the physician within:

A. 24 hours

If it is established that the child is physically abused by a parent, the most important goal the
nurse could formulate with the family is that:

A. Child and any siblings will live in a safe environment


Cocaine is derived from the leaves of coca plant; the nurse knows that cocaine is classified as:

A. Narcotic

To successfully complete the tasks of older adulthood, an 85 year old who has been a widow for
25 years should be encouraged to:

C. Feel a sense of satisfaction in reflecting on her productive life

In a therapeutic relationship, the nurse must understand own values, beliefs, feelings, prejudices
& how these affect others. This is called:

D. Self awareness

While on Bryant’s traction, which of these observations of Graciela and her traction apparatus
would indicate a decrease in the effectiveness of her traction?

A. Graciela’s buttocks are resting on the bed.

The nurse notes that the fall might also cause a possible head injury. She will be observed for
signs of increased intracranial pressure which include:

B. Vomiting

This is a tricyclic antidepressant drug:

D. Imipramine (Tofranil)

The working phase in a therapy group is usually characterized by which of the following?

B. Cohesiveness

Substance abuse is different from substance dependence in that, substance dependence:

D. includes characteristics of tolerance and withdrawal

Ricky’s IQ falls within the range of 50-55. he can be expected to:

D. Acquire academic skills of 6th grade level


The mother of a drug dependent would never consider referring her son to a drug rehabilitation
agency because she fears her son might just become worse while relating with other drug users.
The mother’s behavior can be described as:

B. Codependent

You teach your clients the difference between, Type I (IDDM) and Type II (NDDM) diabetes.
Which of the following is true?

B. Type II (NIDDM) is more common and is also preventable compared to Type I (IDDM)
diabetes which is genetic in etiology

Lifestyle-related diseases in general share areas common risk factors. These are the following
except:

C. genetics

The following mechanisms can be utilized as part of the quality assurance program of your
hospital EXCEPT:

D. Use of Nursing Interventions Classification

The use of the Standards of Nursing Practice is important in the hospital. Which of the following
statements best describes what it is?

D. The Standards of Care includes the various steps of the nursing process and the standards of
professional performance

you are taking care of critically ill client and the doctor in charge calls to order a DNR (do not
resuscitate) for the client. Which of the following is the appropriate action when getting DNR
order over the phone?

B. Have 2 nurse validate the phone order, both nurses sign the order and the doctor should sign
his order within 24 hours

Under the PRC-Board of Nursing Resolution promulgating the adoption of a Nursing Specialty
Certification Program and Council, which two (2) of the following serves as the strongest for its
enforcement?

(a) Advances made in Science and Technology have provided the climate for specialization in
almost all aspects of human endeavor; and
(b) As necessary consequence, there has emerged a new concept known as globalization which
seeks to remove barriers in trade, industry and services imposed by the national laws of countries
all over the world; and
(c) Awareness of this development should impel the nursing sector to prepare our people in the
services sector to meet the above challenge; and
(d) Current trends of specialization in nursing practice recognized by the International Council of
Nurses (ICN) of which the Philippines is a member for the benefit of the Filipino in terms of
deepening and refining nursing practice and enhancing the quality of nursing care.

B. a & b are strong justifications

Knowing that for a comatose patient hearing is the last sense to be lost, as Judy’s nurse, what
should you do?

D. Speak softly then hold her hands gently

Which among the following interventions should you consider as the highest priority when caring
for June who has hemiparesis secondary to stroke?

B. Perform range of motion exercises

Salome was fitted a hearing aid. She understood the proper use and wear of this device when she
says that the battery should be functional, the device is turned on and adjusted to a:

D. audible level

Membership dropout generally occurs in group therapy after a member:

C. Experiences feelings of frustration in the group

Which of the following questions illustrates the group role of encourager?

B. Who wants to respond next?

The goal of remotivation therapy is to facilitate:

A. Insight

Being in contact with reality and the environment is a function of the:

B. ego

Substance abuse is different from substance dependence in that, substance dependence:

H. includes characteristics of tolerance and withdrawal


During the detoxification stage, it is a priority for the nurse to:

D. promote homeostasis and minimize the client’s withdrawal symptoms

Commonly known as “shabu” is:

D. Methampetamine hydrochloride

Posted by diannemaydee at 8:13 PM 4 comments


read read read and study the following!!!

* all that has been discussed and stressed by sir Jungco, ung PA-lmr
lec nya, no need to sweat it out reading Venzon
* my 300 items reviewer, lalo na ung side lectures and notes q
during our rationalization
* the MS reviewer posted here, don't you ever skip reading the
rationales! lalo na if you don't have the CDs
* the fatima exclusive material and the last ten tips for last june
posted at http://pinoybsn.blogspot.com/
* all medication administration procedure na naka-table sa
Fundamentals of Nursing by Kozier
* Appendix C of the PHN book
* Sir V's OR nursing and "special lec"
* all that i stressed and wrote in bold letters during our
rationalization
* love ma'am Mallari promise!
* all notes na makukuha nyo during the final coaching! don't be
absent, wag nyo q gayahin pls lang!
* percussion and vibration Kozier
* Udan MS all practice test and end of the chapter questions
* Makasaysayang RESEARCH METHODOLOGY MANUAL of olfu
* Pilliteri pages 570-590
* mga appendix at glossary of Venzon PA
* new immunization table
* page 215-216 Venzon Management 3rd ed
* Ung likod ng Souvenir chuva namin
* Tables of child development sa pilliteri, tonsillectomy and
contraindications to immunization, ung 2 pages na fine print
* Operating Room technique Instructional Manual 1st ed pages 41-55
* My JCAHO, pain and nle CBQs handout
Thursday, October 18, 2007

StuffedNurse: MEDICAL SURGICAL BOARD EXAM REVIEWER

guys these are almost the same as what was in the june board and same concepts as the june 06
and dec 06.....

answers are the first ones after the questions

Source : Smeltzer and Bare/ Brunner and Suddarth CD

1.
According to Maslow, which of the following categories of needs represents the most basic?

Physiologic needs
Physiologic needs must be met before an individual is able to move toward psychological health
and well-being.
Self-actualization
Self-actualization is the highest level of need

Safety and security needs


Safety and security needs, while lower level, are not essential to physiologic survival.

Belongingness
Belongingness and affection needs are not essential to physiologic survival.

2.
Which of the following statements reflects the World Health Organization’s definition of health?

A state of complete physical, mental, and social well-being and not merely the absence of disease
and infirmity.
Such a definition, however, does not allow for any variations in the degrees of wellness or illness.

A condition of homeostatis and adaptation.


The WHO definition addresses physical, mental, and social dimensions of being.

An individual’s location along a wellness--illness continuum.


The concept of a health--illness continuum allows for a greater range in describing a person’s
health than the definition provided by the WHO.

A fluid, ever-changing balance reflected through physical, mental, and social behavior.
The WHO definition does not allow for any variations in the degrees of wellness and illness.

3.
Which of the following statements defines culture?

The learned patterns of behavior, beliefs, and values that can be attributed to a particular group of
people.
Included among characteristics that distinguish cultural groups are manner of dress, values,
artifacts, and health beliefs and practices.

A group of people distinguished by genetically transmitted material.


A group of people distinguished by genetically transmitted material describes the term race.

The status of belonging to a particular region by origin, birth, or naturalization.


The status of belonging to a particular region by origin, birth, or naturalization describes the term
nationality.

The classification of a group based upon certain distinctive characteristics.


The classification of a group based upon certain distinctive characteristics describes the term
ethnicity.

4.
The reason that case management has gained such prominence in health care can be traced to

decreased cost of care associated with inpatient stay.


The reasons case management has gained such prominence can be traced to the decreased cost of
care associated with decreased length of hospital stay, coupled with rapid and frequent inter-unit
transfers from specialty to standard care units.

increased length of hospital stay.


In general, length of hospital stay has decreased over the past 5 years.

discharge from specialty care units to home.


In general, patients are transferred from specialty care units to standard care units at least 24
hours prior to discharge.

limited availability for inter-unit hospital transfers.


In general, patients in acute care hospitals undergo frequent inter-unit transfers from specialty to
standard care units.

5.
A preferred provider organization is described as a

business arrangement between hospitals and physicians.


PPO’s usually contract to provide health care to subscribers, usually businesses, for a negotiated
fee that often is discounted.

prepaid group health practice system.


A prepaid group health practice system is termed a health maintenance organization.

limited insurance program.


Insurance is a cost payment system of shared risk, not a health care delivery system.

health care savings account program.


A health care savings account program is an incentive program to consumers, not a health care
delivery system.

6.
Which of the following categories identifies the focus of community/public health nursing
practice?

Promoting and maintaining the health of populations and preventing and minimizing the progress
of disease
Although nursing interventions used by public health nurses might involve individuals, families,
or small groups, the central focus remains promoting health and preventing disease in the entire
community.

Rehabilitation and restorative services


Rehabilitation and restorative services are the focus of extended care facilities and home care
nursing.

Adaptation of hospital care to the home environment


Adaptation of hospital care to the home environment is the focus of home nursing.

Hospice care delivery


Hospice care delivery refers to the delivery of services to the terminally ill.

7.
A major goal for home care nurses is

restoring maximum health function.


Tertiary preventive nursing care, focusing on rehabilitation and restoring maximum health
function, is a goal for home care nurses.

promoting the health of populations.


Promoting the health of populations is a focus of community/public health nursing.

minimizing the progress of disease.


Minimizing the progress of disease is a focus of community/public health nursing.

maintaining the health of populations.


Maintaining the health of populations is a focus of community/public health nursing.

8.
In the United States, nurses performing invasive procedures need to be up-to-date with their
immunizations, particularly

hepatitis B.
Hepatitis B is transmitted through contact with infected blood or plasma.

hepatitis E.
Hepatitis E is found mainly in underdeveloped countries with substandard sanitation and water
quality.

hepatitis A.
hepatitis A is transmitted through the oral route from the feces and saliva of an infected person.
hepatitis C.
At present, immunization against hepatitis C is not available.

9.
At what time during a patient’s hospital stay does discharge planning begin?

Admission
To prepare for early discharge and the possible need for follow-up in the home, discharge
planning begins with the patient’s admission.

Twenty-four hours prior to discharge


Discharge planning requires identification of patient needs and anticipatory guidance and is not
relegated to a specific time for beginning.

The shift prior to discharge


Discharge planning requires communication with and cooperation of the patient, family, and
health care team and is not relegated to a specific time for beginning.

By the third hospital day


Discharge planning may require involvement of personnel and agencies in the planning process
and is not relegated to a specific day of hospital stay.

10.
The leading health problems of elementary school children include

cancer.
The leading health problems of elementary school children are injuries, infections, malnutrition,
dental disease, and cancer.

alcohol and drug abuse.


Alcohol and drug abuse are leading health problems for high school students.

mental and emotional problems.


Mental and emotional problems are leading health problems for high school students.

homicide.
Homicide is a leading health problem for high school children.

11.
Which skill needed by the nurse to think critically involves identification of patient problems
indicated by data?
Analysis
Analysis is used to identify patient problems indicated by data.

Interpretation
Interpretation is used to determine the significance of data that is gathered.

Inferencing
Inferences are used by the nurse to draw conclusions.

Explanation
Explanation is the justification of actions or interventions used to address patient problems and to
help a patient move toward desired outcomes.

12.
The ethics theory that focuses on ends or consequences of actions is the

utilitarian theory.
Utilitarian theory is based on the concept of the greatest good for the greatest number.

formalist theory.
Formalist theory argues that moral standards exist independently of the ends or consequences.

deontological theory.
Deontological theory argues that moral standards exist independently of the ends or
consequences.

adaptation theory.
Adaptation theory is not an ethics theory.

13.
Which of the following ethical principles refers to the duty to do good?

Beneficence
Beneficence is the duty to do good and the active promotion of benevolent acts.

Fidelity
Fidelity refers to the duty to be faithful to one's commitments.

Veracity
Veracity is the obligation to tell the truth.

Nonmaleficence
Nonmaleficence is the duty not to inflict, as well as to prevent and remove, harm; it is more
binding than beneficence.
14.
During which step of the nursing process does the nurse analyze data related to the patient's
health status?

Assessment
Analysis of data is included as part of the assessment.

Implementation
Implementation is the actualization of the plan of care through nursing interventions.

Diagnosis
Diagnosis is the identification of patient problems.

Evaluation
Evaluation is the determination of the patient's responses to the nursing interventions and the
extent to which the outcomes have been achieved.

15.
The basic difference between nursing diagnoses and collaborative problems is that

nurses manage collaborative problems using physician-prescribed interventions.


Collaborative problems are physiologic complications that nurses monitor to detect onset or
changes and manage through the use of physician-prescribed and nursing-prescribed
interventions to minimize the complications of events.

collaborative problems can be managed by independent nursing interventions.


Collaborative problems require both nursing and physician-prescribed interventions.

nursing diagnoses incorporate physician-prescribed interventions.


Nursing diagnoses can be managed by independent nursing interventions.

nursing diagnoses incorporate physiologic complications that nurses monitor to detect change in
status.
Nursing diagnoses refer to actual or potential health problems that can be managed by
independent nursing interventions.

16.
Health education of the patient by the nurse

is an independent function of nursing practice.


Health education is an independent function of nursing practice and is included in all state nurse
practice acts.
requires a physician's order.
Teaching, as a function of nursing, is included in all state nurse practice acts.

must be approved by the physician.


Health education is a primary responsibility of the nursing profession.

must focus on wellness issues.


Health education by the nurse focuses on promoting, maintaining, and restoring health;
preventing illness; and assisting people to adapt to the residual effects of illness.

17.
Nonadherence to therapeutic regimens is a significant problem for which of the following age
groups?

Adults 65 and over


Elderly people frequently have one or more chronic illnesses that are managed with numerous
medications and complicated by periodic acute episodes, making adherence difficult.

Teenagers
Problems of teenagers, generally, are time limited and specific, and require promoting adherence
to treatment to return to health.

Children
In general, the compliance of children depends on the compliance of their parents.

Middle-aged adults
Middle-aged adults, in general, have fewer health problems, thus promoting adherence.

18.
Experiential readiness to learn refers to the patient's

past history with education and life experience.


Experiential readiness refers to past experiences that influence a person's ability to learn.

emotional status.
Emotional readiness refers to the patient's acceptance of an existing illness or the threat of an
illness and its influence on the ability to learn.

acceptance of an existing illness.


Emotional readiness refers to the patient's acceptance of an existing illness or the threat of an
illness and its influence on the ability to learn.

ability to focus attention.


Physical readiness refers to the patient's ability to cope with physical problems and focus
attention upon learning.

19.
Asking the patient questions to determine if the person understands the health teaching provided
would be included during which step of the nursing process?

Evaluation
Evaluation includes observing the person, asking questions, and comparing the patient's
behavioral responses with the expected outcomes.

Assessment
Assessment includes determining the patient's readiness regarding learning.

Planning and goals


Planning includes identification of teaching strategies and writing the teaching plan.

Implementation
Implementation is the step during which the teaching plan is put into action.

20.
Which of the following items is considered the single most important factor in assisting the health
professional in arriving at a diagnosis or determining the person's needs?

History of present illness


The history of the present illness is the single most important factor in assisting the health
professional in arriving at a diagnosis or determining the person's needs.

Physical examination
The physical examination is helpful but often only validates the information obtained from the
history.

Diagnostic test results


Diagnostic test results can be helpful, but they often only verify rather than establish the
diagnosis.

Biographical data
Biographical information puts the health history in context but does not focus the diagnosis.

21.
Of the following areas for assessing the patient profile, which should be addressed after the
others?
Body image
The patient is often less anxious when the interview progresses from information that is less
personal to information that is more personal.

Education
Educational level is relatively impersonal and readily revealed by the patient.

Occupation
Occupation is relatively impersonal and readily revealed by the patient.

Environment
Housing, religion, and language are relatively impersonal and readily revealed by the patient.

22.
Which of the following methods of physical examination refers to the translation of physical
force into sound?

Percussion
Percussion translates the application of physical force into sound.

Palpation
Palpation refers to examination by non-forceful touching.

Auscultation
Auscultation refers to the skill of listening to sounds produced within the body created by
movement of air or fluid.

Manipulation
Manipulation refers to the use of the hands to determine motion of a body part.

23.
In which range of body mass index (BMI) are patients considered to have increased risk for
problems associated with poor nutritional status?

Below 24
Additionally, higher mortality rates in hospitalized patients and community-dwelling elderly are
associated with individuals who have low BMI.

25-29
Those who have a BMI of 25 to 29 are considered overweight.

30 to 39
Those who have BMI of 30-39 are considered obese.
Over 40
Those who have BMI over 40 are considered extremely obese.

24.
To calculate the ideal body weight for a woman, the nurse allows

100 pounds for 5 feet of height.


To calculate the ideal body weight of a woman, the nurse allows 100 pounds for 5 feet of height
and adds 5 pounds for each additional inch over 5 feet

106 pounds for 5 feet of height.


The nurse allows 106 pounds for 5 feet of height in calculating the ideal body weight for a man.

6 pounds for each additional inch over 5 feet.


The nurse adds 6 pounds for each additional inch over 5 feet in calculating the ideal body weight
for a man.

80 pounds for 5 feet of height.


Eighty pounds for 5 feet of height is too little.

25.
A steady state within the body is termed

homeostasis.
When a change occurs that causes a body function to deviate from its stable range, processes are
initiated to restore and maintain the steady state or homeostasis.

constancy.
Constancy refers to the balanced internal state of the human body maintained by physiologic and
biochemical processes.

adaptation.
Adaptation refers to a constant, ongoing process that requires change in structure, function, or
behavior so that the person is better suited to the environment.

stress.
Stress refers to a state produced by a change in the environment that is perceived as challenging,
threatening, or damaging to the person's dynamic balance or equilibrium.

26.
Which of the following terms, according to Lazarus, refers to the process through which an event
is evaluated with respect to what is at stake and what might and can be done?
Cognitive appraisal
The outcome of cognitive appraisal is identification of the situation as either stressful or non-
stressful.

Coping
Coping consists of both cognitive and behavioral efforts made to manage the specific external or
internal demand that taxes a person's resources.

Hardiness
Hardiness is a personality characteristic that is composed of control, commitment, and challenge.

Adaptation
Lazarus believed adaptation was affected by emotion that subsumed stress and coping.

27.
An increase in the number of new cells in an organ or tissue that is reversible when the stimulus
for production of new cells is removed is termed

hyperplasia.
Hyperplasia occurs as cells multiply and are subjected to increased stimulation resulting in tissue
mass enlargement.

hypertrophy.
Hypertrophy is an increase in size and bulk of tissue that does not result from an increased
number of cells.

atrophy.
Atrophy refers to reduction in size of a structure after having come to full maturity.

neoplasia.
With neoplasia, the increase in the number of new cells in an organ or tissue continues after the
stimulus is removed.

28.
Which of the following types of cells have a latent ability to regenerate?

Stable
Stable cells have a latent ability to regenerate if they are damaged or destroyed and are found in
the kidney, liver, and pancreas, among other body organs.

Labile
Labile cells multiply constantly to replace cells worn out by normal physiologic processes.

Permanent
Permanent cells include neurons --- the nerve cell bodies, not their axons. Destruction of a neuron
causes permanent loss, but axons may regenerate.

Epithelial
Epithelial cells are a type of labile cell that multiply constantly to replace cells worn out by
normal physiologic processes.

29.
The relaxation techniques of progressive muscle relaxation, relaxation with guided imagery, and
the Benson Relaxation Response share which of the following elements?

A mental device (something on which to focus the attention)


Similar elements also include a quiet environment, a comfortable position, and a passive attitude.

Nutritional foundation
Relaxation techniques do not encompass specific nutritional guidelines.

Analgesic preparation
Relaxation techniques are used to reduce one's response to stress and do not require analgesia
prior to practicing the techniques.

Physician's order
A physician's order is not required to assist an individual to learn techniques to reduce one's
response to stress.

30.
Which of the following terms has been defined by the American Psychiatric Association as a
group of behavioral or psychological symptoms or a pattern that manifests itself in significant
distress, impaired functioning, or accentuated risk of enduring severe suffering or possible death?

Mental disorder
The definition was adopted by the American Psychiatric Association in 1994.

Emotional disorder
There is no universally accepted definition of what constitutes an emotional disorder.

Anxiety
Anxiety is defined as fear of the unknown.

Schizophrenia
Schizophrenia is a specific disorder characterized by psychosis.

31.
Establishing financial security has been identified as a developmental task of which of the
following groups?

Middle adult
The middle adult's tasks also include launching children, and refocusing on one's marital
relationship.

Older adult
The older adult's tasks include adapting to retirement and declining physical stamina.

Young adult
The young adult's tasks include establishing a lifestyle and independence.

Teenager
The teenager's primary developmental tasks include developing an identity and intimacy.

32.
When up to a 6-month period elapses between the experience of trauma and the onset of
symptoms of posttraumatic stress disorder (PTSD), the episode is termed

delayed.
In the case of delayed PTSD, there may be up to a 6-month period of time that elapses between
the trauma and the manifestation of symptoms.

acute.
Acute PTSD is defined as the experience of symptoms for less than a 3-month period.

chronic.
Chronic PTSD is defined as the experience of symptoms lasting longer than 3 months.

primary.
The concept of primary disease is not used in relation to PTSD.

33.
Which of the following statements accurately describes a risk factor for depression?

History of physical or sexual abuse


History of physical or sexual abuse and current substance abuse are risk factors for depression.

Male gender
A risk factor for depression is female gender.

Age over 50 years


A risk factor for depression is onset before 40 years.
Negative family history of depression
Family history of depression is a risk factor.

34.
Of the following stages of grieving as described by Kubler-Ross, which is the initial?

Denial
The stages include: denial, anger, bargaining, depression, and acceptance.

Anger
Anger is the second stage of the process.

Bargaining
Bargaining is the third stage of the process.

Depression
Depression is the fourth stage of the process.

35.
Which of the following terms refers to Leininger's description of the learned and transmitted
knowledge about values, beliefs, rules of behavior, and lifestyle practices that guide a designated
group in their thinking and actions in patterned ways?

Culture
Leininger was the founder of the specialty called transcultural nursing and advocated culturally
competent nursing care.

Minority
Minority refers to a group of people whose physical or cultural characteristics differ from the
majority of people in a society.

Race
Race refers to a group of people distinguished by genetically transmitted characteristics.

Subculture
Subculture refers to a group that functions within a culture.

36.
The inability of a person to recognize his or her own values, beliefs, and practices and those of
others because of strong ethnocentric tendencies is termed

cultural blindness.
Cultural blindness results in bias and stereotyping.

acculturation.
Acculturation is the process by which members of a culture adapt or learn how to take on the
behaviors of another group.

cultural imposition.
Cultural imposition is the tendency to impose one's cultural beliefs, values, and patterns of
behavior on a person from a different culture.

cultural taboo.
Cultural taboos are those activities governed by rules of behavior that are avoided, forbidden, or
prohibited by a particular cultural group.

37.
Which of the following groups of individuals may stare at the floor during conversations as a sign
of respect?

Native Americans
Some Native Americans stare at the floor during conversations, conveying respect and indicating
that the listener is paying close attention to the speaker.

Indo-Chinese
The Indo-Chinese may consider direct eye contact impolite or aggressive.

Arabs
Arabs may consider direct eye contact impolite or aggressive.

Asians
Asians may consider direct eye contact impolite or aggressive.

38.
For which of the following religious groups is all meat prohibited?

Hinduism
Hinduism prohibits consumption of all meats and animal shortening.

Seventh-Day Adventism
Seventh-Day Adventism prohibits consumption of pork.

Judaism
Judaism prohibits consumption of pork.

Islam
Islam prohibits the consumption of pork and animal shortening.

39.
The paradigm that explains the cause of illness as an imbalance in the forces of nature is the

holistic perspective.
The naturalist or holistic perspective believes that health exists when all aspects of a persona are
in perfect balance or harmony.

magico-religious view.
The magico-religious view holds that illness is caused by forces of evil.

biomedical view.
The biomedical view holds life events as cause and effect and incorporates the bacterial or viral
explanation of communicable disease.

scientific view.
The scientific view holds life events as cause and effect and incorporates the bacterial or viral
explanation of communicable disease.

40.
The aim of genomic medicine is

improving predictions about individuals’ susceptibility to diseases


Predictions regarding the time of their onset, their extent and eventual severity as well as which
treatments or medications are likely to be most effective or harmful are the focus of genomic
medicine.

reproduction
The focus of genomic medicine is broader than the reproduction of cells.

cure of disease
The focus of genomic medicine is broader than the cure of disease.

cloning
Genomic medicine is gene-based health care.

41.
Nondisjunction of a chromosome results in which of the following diagnoses?

Down Syndrome
When a pair of chromosomes fails to separate completely and creates a sperm or oocyte that
contains two copies of a particular chromosome (nondisjunction) Down syndrome results from
three number 21 chromosomes.
Huntingon Disease
Huntington disease is one example of a germ-line mutation.

Duchenne Muscular Dystrophy


Duchenne muscular dystrophy, an inherited form of muscular dystrophy, is an example of a
genetic caused by structural gene mutations.

Marphan Syndrome
Marphan Syndrome is a genetic condition that may occur in a single family member as a result of
spontaneous mutation.

42.
Which type of Mendelian inherited condition results in both genders being affected equally in a
vertical pattern?

Automosomal dominant inheritance


An individual who has an autosomal dominant inherited condition carries a gene mutation for that
condition on one chromosome of a pair.

Automosomal recessive inheritance


The pattern of inheritance in autosomal recessive inherited conditions is different from that of
autosomal dominant inherited conditions in that it is more horizontal than vertical, with relatives
of a single generation tending to have the condition.

X-linked inheritance
X-linked conditions may be inherited in families in recessive or dominant patterns. In both, the
gene mutation is located on the X-chromosome. All males inherit an X chromosome from their
mother with no counterpart; hence, all males express the gene mutation.

Multifactorial genetic inheritance


Neural tube defects, such as spina bifida and anencephaly, are examples of multifactorial genetic
conditions. The majority of neural tube defects are caused by both genetic and environmental
influences that combine during early embryonic development leading to incomplete closure of the
neural tube.

43.
A specific BRCA1 cancer-predisposing gene mutation seems to occur more frequently among
women of which descent?

Ashkanazi Jewish
Expression of the BRCA1 gene is an example of inheritance in the development of breast cancer.

Mediterranean
Glucose-6-phosphate dehydrogenase deficiency (G6PD) is a common enzyme abnormality that
affects millions of people throughout the world, especially those of Mediterranean, South East
Asian, African, Middle Eastern, and Near Eastern origin.

African American
Sickle cell anemia is associated with the African-American population.

Chinese and Japanese


Individuals of Chinese and Japanese descent who are rapid metabolizers of the enzyme N-
acetyltransferase and who are prescribed the drug isoniazid (as part of treatment for tuberculosis)
are at significantly increased risk for developing isoniazid-induced hepatitis.

44.
Which of the following statements describes accurate information related to chronic illness?

Most people with chronic conditions do not consider themselves sick or ill.
Although some people take on a sick role identity, most people with chronic conditions do not
consider themselves sick or ill and try to live as normal a life as is possible.

Most people with chronic conditions take on a sick role identity.


Research has demonstrated that some people with chronic conditions may take on a sick role
identity, but they are not the majority.

Chronic conditions do not result from injury.


Chronic conditions may be due to illness, genetic factors, or injury

Most chronic conditions are easily controlled.


Many chronic conditions require therapeutic regimens to keep them under control.

45.
In which phase of the trajectory model of chronic illness are the symptoms under control and
managed?

Stable
The stable phase indicates that the symptoms and disability are under control or managed.

Acute
The acute phase is characterized by sudden onset of severe or unrelieved symptoms or
complications that may necessitate hospitalization for their management.

Comeback
The comeback phase is the period in the trajectory marked by recovery after an acute period.

Downward
The downward phase occurs when symptoms worsen or the disability progresses despite attempts
to control the course through proper management.

46.
Which phase of the trajectory model of chronic illness is characterized by reactivation of the
illness?

Unstable
The unstable phase is characterized by development of complications or reactivation of the
illness.

Stable
The stable phase indicates that the symptoms and disability are under control or managed.

Acute
The acute phase is characterized by sudden onset of severe or unrelieved symptoms or
complications that may necessitate hospitalization for their management.

Comeback
The comeback phase is the period in the trajectory marked by recovery after an acute period.

47.
Which phase of the trajectory model of chronic illness is characterized by the gradual or rapid
decline in the trajectory despite efforts to halt the disorder?

Dying
The dying phase is characterized by stoppage of life-maintaining functions.

Unstable
The unstable phase is characterized by development of complications or reactivation of the
illness.

Acute
The acute phase is characterized by sudden onset of severe or unrelieved symptoms or
complications that may necessitate hospitalization for their management.

Downward
The downward phase occurs when symptoms worsen or the disability progresses despite attempts
to control the course through proper management.

48.
In order to help prevent the development of an external rotation deformity of the hip in a patient
who must remain in bed for any period of time, the most appropriate nursing action would be to
use

a trochanter roll extending from the crest of the ilium to the midthigh.
A trochanter roll, properly placed, provides resistance to the external rotation of the hip.

pillows under the lower legs.


Pillows under the legs will not prevent the hips from rotating externally.

a hip-abductor pillow.
A hip-abductor pillow is used for the patient after total hip replacement surgery.

a footboard.
A footboard will not prevent the hips from rotating externally.

49.
To prevent footdrop, the patient is positioned in:

Order to keep the feet at right angles to the leg


When the patient is supine in bed, padded splints or protective boots are used.

A semi-sitting position in bed


Semi-fowlers positioning is used to decrease the pressure of abdominal contents on the
diaphragm.

A sitting position with legs hanging off the side of the bed
In order to prevent footdrop, the feet must be supported.

A side-lying position
Side-lying positions do not provide support to prevent footdrop.

50.
Through which of the following activities does the patient learn to consciously contract excretory
sphincters and control voiding cues?
Biofeedback
Cognitively intact patients who have stress or urge incontinence may gain bladder control through
biofeedback.

Kegel exercises
Kegel exercises are pelvic floor exercises that strengthen the pubococcygeus muscle.

Habit training
Habit training is used to try to keep the patient dry by strictly adhering to a toileting schedule and
may be successful with stress, urge, or functional incontinence.
Bladder training
Habit training is a type of bladder training.

51.
During which stage of pressure ulcer development does the ulcer extend into the subcutaneous
tissue?

Stage III
Clinically, a deep crater with or without undermining of adjacent tissues is noted.

Stage IV
A stage IV pressure ulcer extends into the underlying structure, including the muscle and possibly
the bone.

Stage II
A stage II ulcer exhibits a break in the skin through the epidermis or dermis.

Stage I
A stage I pressure ulcer is an area of nonblanchable erythema, tissue swelling, and congestion,
and the patient complains of discomfort.

52.
During which stage of pressure ulcer development does the ulcer extend into the underlying
structures, including the muscle and possibly the bone?

Stage IV
A stage IV pressure ulcer extends into the underlying structure, including the muscle and possibly
the bone.

Stage III
A stage III ulcer extends into the subcutaneous tissue.

Stage II
A stage II ulcer exhibits a break in the skin through the epidermis or dermis.

Stage I
A stage I pressure ulcer is an area of nonblanchable erythema, tissue swelling, and congestion,
and the patient complains of discomfort.

53.
Which type of incontinence is associated with weakened perineal muscles that permit leakage of
urine when intra-abdominal pressure is increased?
Stress incontinence
Stress incontinence may occur with coughing or sneezing.

Urge incontinence
Urge incontinence is involuntary elimination of urine associated with a strong perceived need to
void.

Reflex (neurogenic) incontinence


Neurogenic incontinence is associated with a spinal cord lesion.

Functional incontinence
Functional incontinence refers to incontinence in patients with intact urinary physiology who
experience mobility impairment, environmental barriers, or cognitive problems.

54.
Ageism refers to

Bias against older people based solely on chronological age


Individuals demonstrating ageism base their beliefs and attitudes about older people based upon
chronological age without consideration of functional capacity.

fear of old age.


Fear of aging and the inability of many to confront their own aging process may trigger ageist
beliefs.

loss of memory.
Age-related loss of memory occurs more with short-term and recent memory.

benign senescent forgetfulness.


Benign senescent forgetfulness refers to the age-related loss of memory in the absence of a
pathologic process.

55.
When assessing the older adult, the nurse anticipates increase in which of the follow components
of respiratory status?

Residual lung volume


As a result, patient experience fatigue and breathlessness with sustained activity.

Vital capacity
The nurse anticipates decreased vital capacity.

Gas exchange and diffusing capacity


The nurse anticipates decreased gas exchange and diffusing capacity resulting in impaired healing
of tissues due to decreased oxygenation.

Cough efficiency
The nurse anticipates difficulty coughing up secretions due to decreased cough efficiency.

56.
According to the classification of hypertension diagnosed in the older adult, hypertension that can
be attributed to an underlying cause is termed

secondary.
Secondary hypertension may be caused by a tumor of the adrenal gland (e.g.,
pheochromacytoma).

primary.
Primary hypertension has no known underlying cause.

essential.
Essential hypertension has no known underlying cause.

isolated systolic.
Isolated systolic hypertension is demonstrated by readings in which the systolic pressure exceeds
140 mm Hg and the diastolic measurement is normal or near normal (less than 90 mm Hg).

57.
Which of the following terms refers to the decrease in lens flexibility that occurs with age,
resulting in the near point of focus getting farther away?

Presbyopia
Presbyopia usually begins in the fifth decade of life, when reading glasses are required to
magnify objects.

Presbycusis
Presbycusis refers to age-related hearing loss.

Cataract
Cataract is the development of opacity of the lens of the eye.

Glaucoma
Glaucoma is a disease characterized by increased intraocular pressure.

58.
Which of the following states is characterized by a decline in intellectual functioning?
Dementia
Dementia is an acquired syndrome in which progressive deterioration in global intellectual
abilities is of such severity that it interferes with the person's customary occupational and social
performance.

Depression
Depression is a mood disorder that disrupts quality of life.

Delirium
Delirium is often called acute confusional state.

Delusion
Delusion is a symptom of psychoses.

59.
When a person who has been taking opioids becomes less sensitive to their analgesic properties,
that person is said to have developed a (an)

tolerance.
Tolerance is characterized by the need for increasing dose requirements to maintain the same
level of pain relief.

addiction.
Addiction refers to a behavioral pattern of substance use characterized by a compulsion to take
the drug primarily to experience its psychic effects.

dependence.
Dependence occurs when a patient who has been taking opioids experiences a withdrawal
syndrome when the opioids are discontinued.

balanced analgesia.
Balanced analgesia occurs when the patient is using more than one form of analgesia
concurrently to obtain more pain relief with fewer side effects.

60.
Prostaglandins are chemical substances thought to

increase sensitivity of pain receptors.


Prostaglandins are believed to increase sensitivity to pain receptors by enhancing the pain-
provoking effect of bradykinin.

reduce the perception of pain.


Endorphins and enkephalins reduce or inhibit transmission or perception of pain.
inhibit the transmission of pain.
Endorphins and enkephalins reduce or inhibit transmission or perception of pain.

inhibit the transmission of noxious stimuli.


Morphine and other opioid medications inhibit the transmission of noxious stimuli by mimicking
enkephalin and endorphin.

61.
Which of the following principles or guidelines accurately informs the nurse regarding placebos?

Placebos should never be used to test the person's truthfulness about pain.
Perception of pain is highly individualized.

A placebo effect is an indication that the person does not have pain.
A placebo effect is a true physiologic response.

A placebo should be used as the first line of treatment for the patient.
A placebo should never be used as a first line of treatment.

A positive response to a placebo indicates that the person's pain is not real.
Reduction in pain as a response to placebo should never be interpreted as an indication that the
person's pain is not real.

62.
Regarding tolerance and addiction, the nurse understands that

although patients may need increasing levels of opioids, they are not addicted.
Physical tolerance usually occurs in the absence of addiction.

tolerance to opioids is uncommon.


Tolerance to opioids is common.

addiction to opioids commonly develops.


Addiction to opioids is rare.

the nurse must be primarily concerned about development of addiction by the patient in pain.
Addiction is rare and should never be the primary concern for a patient in pain.

63.
The preferred route of administration of medication in the most acute care situations is which of
the following routes?

Intravenous
The IV route is the preferred parenteral route in most acute care situations because it is much
more comfortable for the patient, and peak serum levels and pain relief occur more rapidly and
reliably.

Epidural
Epidural administration is used to control postoperative and chronic pain.

Subcutaneous
Subcutaneous administration results in slow absorption of medication.

Intramuscular
Intramuscular administration of medication is absorbed more slowly than intravenously
administered medication.

64.
Mu opioids have which of the following effects on respiratory rate:

Stimulation, then depression


Mu opioids also cause bradycardia, hypothermia, and constipation.

No change
Kappa opioids result in no change in respiratory rate.

Stimulation, only
Delta opioids result in stimulation of respiratory rate.

Depression, only
Neither mu, nor kappa, nor delta opoids depress respiratory rate as its only effect upon respiratory
rate.

65.
Which of the following electrolytes is a major cation in body fluid?

Potassium
Potassium is a major cation that affects cardiac muscle functioning.

Chloride
Chloride is an anion.

Bicarbonate
Bicarbonate is an anion.

Phosphate
Phosphate is an anion.
66.
Which of the following electrolytes is a major anion in body fluid?

Chloride
Chloride is a major anion found in extracellular fluid.

Potassium
Potassium is a cation.

Sodium
Sodium is a cation.

Calcium
Calcium is a cation.

67.
Oncotic pressure refers to

the osmotic pressure exerted by proteins.


Oncotic pressure is a pulling pressure exerted by proteins, such as albumin.

the number of dissolved particles contained in a unit of fluid.


Osmolality refers to the number of dissolved particles contained in a unit of fluid.

the excretion of substances such as glucose through increased urine output.


Osmotic diuresis occurs when the urine output increases due to excretion of substances such as
glucose.

the amount of pressure needed to stop flow of water by osmosis.


Osmotic pressure is the amount of pressure needed to stop the flow of water by osmosis.

68.
Which of the following solutions is hypotonic?

0.45% NaCl.
Half-strength saline is hypotonic

Lactated Ringer's solution.


Lactated Ringer's is isotonic.

0.9% NaCl.
Normal saline (0.9% NaCl) is isotonic.
5% NaCl.
A solution that is 5% NaCl is hypertonic.

69.
The normal serum value for potassium is

3.5-5.5 mEq/L.
Serum potassium must be within normal limits to prevent cardiac dysrhythmias.

135-145 mEq/L.
Normal serum sodium is 135-145 mEq/L.

96-106 mEq/L.
Normal serum chloride is 96-106 mEq/L.

8.5-10.5 mg/dL.
Normal total serum calcium is 8.5-10.5mg/dL.

70.
In which type of shock does the patient experiences a mismatch of blood flow to the cells?

Distributive
Distributive or vasogenic shock results from displacement of blood volume, creating a relative
hypovolemia.

Cardiogenic
Cardiogenic shock results from the failure of a heart as a pump.

Hypovolemic
In hypovolemic shock, there is a decrease in the intravascular volume.

Septic
In septic shock, overwhelming infection results in a relative hypovolemia.

71.
Which stage of shock is best described as that stage when the mechanisms that regulate blood
pressure fail to sustain a systolic pressure above 90 mm Hg?

Progressive
In the progressive stage of shock, the mechanisms that regulate blood pressure can no longer
compensate, and the mean arterial pressure falls below normal limits.
Refractory
The refractory or irreversible stage of shock represents the point at which organ damage is so
severe that the patient does not respond to treatment and cannot survive.

Compensatory
In the compensatory state, the patient's blood pressure remains within normal limits due to
vasoconstriction, increased heart rate, and increased contractility of the heart.

Irreversible
The refractory or irreversible stage of shock represents the point at which organ damage is so
severe that the patient does not respond to treatment and cannot survive.

72.
When the nurse observes that the patient's systolic blood pressure is less than 80--90 mm Hg,
respirations are rapid and shallow, heart rate is over 150 beats per minute, and urine output is less
than 30 cc per hour, the nurse recognizes that the patient is demonstrating which stage of shock?

Compensatory
In compensatory shock, the patient's blood pressure is normal, respirations are above 20, and
heart rate is above 100 but below 150.

Progressive
In progressive shock, the patient's skin appears mottled and mentation demonstrates lethargy.

Refractory
In refractory or irreversible shock, the patient requires complete mechanical and pharmacologic
support.

Irreversible
In refractory or irreversible shock, the patient requires complete mechanical and pharmacologic
support.

73.
Which of the following vasoactive drugs used in treating shock results in reduced preload and
afterload, reducing oxygen demand of the heart?

Nitroprusside (Nipride)
A disadvantage of nitroprusside is that it causes hypotension.

Dopamine (Intropin)
Dopamine improves contractility, increases stroke volume, and increases cardiac output.

Epinephrine (adrenaline)
Epinephrine improves contractility, increases stroke volume, and increases cardiac output.
Methoxamine (Vasoxyl)
Methoxamine increases blood pressure by vasoconstriction.

74.
The nurse anticipates that the immunosuppressed patient is at greatest risk for which type of
shock?

Septic
Septic shock is associated with immunosuppression, extremes of age, malnourishment, chronic
illness, and invasive procedures.

Neurogenic
Neurogenic shock is associated with spinal cord injury and anesthesia.

Cardiogenic
Cardiogenic shock is associated with disease of the heart.

Anaphylactic
Anaphylactic shock is associated with hypersensitivity reactions.

75.
Which of the following colloids is expensive but rapidly expands plasma volume?

Albumin
Albumin is a colloid that requires human donors, is limited in supply, and can cause congestive
heart failure.

Dextran
Dextran is a colloid, synthetic plasma expander that interferes with platelet aggregation and is not
recommended for hemorrhagic shock.

Lactated Ringers
Lactated ringers is a crystalloid, not a colloid.

Hypertonic Saline
Hypertonic saline is a crystalloid, not a colloid.

76.
Which of the following terms refers to cells that lack normal cellular characteristics and differ in
shape and organization with respect to their cells of origin?

Anaplasia
Usually, anaplastic cells are malignant.
Neoplasia
Neoplasia refers to uncontrolled cell growth that follows no physiologic demand.

Dysplasia
Dysplasia refers to bizarre cell growth resulting in cells that differ in size, shape, or arrangement
from other cells of the same type of tissue.

Hyperplasia
Hyperplasia refers to an increase in the number of cells of a tissue, most often associated with a
period of rapid body growth.

77.
Palliation refers to

relief of symptoms associated with cancer.


Palliation is the goal for care in terminal cancer patients.

hair loss.
Alopecia is the term that refers to hair loss.

the spread of cancer cells from the primary tumor to distant sites.
Metastasis is the term that refers to the spread of cancer cells from the primary tumor to distant
sites.

the lowest point of white blood cell depression after therapy that has toxic effects on the bone
marrow.
Nadir is the term that refers to the lowest point of white blood cell depression after therapy that
has toxic effects on the bone marrow.

78.
During which step of cellular carcinogenesis do cellular changes exhibit increased malignant
behavior?

Progression
During this third step, cells show a propensity to invade adjacent tissues and metastasize.

Promotion
During promotion, repeated exposure to promoting agents causes the expression of abnormal
genetic information even after long latency periods.

Initiation
During this first step, initiators such as chemicals, physical factors, and biologic agents escape
normal enzymatic mechanisms and alter the genetic structure of cellular DNA.
Prolongation
No stage of cellular carcinogenesis is termed prolongation.

79.
The drug, Interleukin-2, is an example of which type of biologic response modifier?

Cytokine
Other cytokines include interferon alfa and filgrastim.

Monoclonal antibodies
Monoclonal antibodies include rituximab, trastuzumab, and gemtuzumab.

Retinoids
Retinoic acid is an example of a retinoid.

Antimetabolites
Antimetabolites are cell cycle-specific antineoplastic agents.

80.
Of the following terms, which is used to refer to the period of time during which mourning a loss
takes place?

Bereavement
Bereavement is the period of time during which mourning a loss takes place.

Grief
Grief is the personal feelings that accompany an anticipated or actual loss

Mourning
Mourning is the individual, family, group and cultural expressions of grief and associated
behaviors

Hospice
Hospice is a coordinated program of interdisciplinary care and services provided primarily in the
home to terminally ill patients and their families.

81.
Which of the following "awareness contexts" is characterized by the patient, the family, and the
health care professionals being aware that the patient is dying but all pretend otherwise?

Mutual pretense awareness


In mutual pretense awareness, the patient, the family and the health care professionals are aware
that the patient is dying but all pretend otherwise.

Closed awareness
In closed awareness, the patient is unaware of his terminality in a context where others are aware.

Suspected awareness
In suspected awareness, the patient suspects what others know and attempts to find it out.

Open awareness
In open awareness, all are aware that the patient is dying and are able to openly acknowledge that
reality.

82.
For individuals known to be dying by virtue of age and/or diagnoses, which of the following
signs indicate approaching death:

Increased restlessness
As the oxygen supply to the brain decreases, the patient may become restless.

Increased wakefulness
As the body weakens, the patient will sleep more and begin to detach from the environment.

Increased eating
For many patients, refusal of food is an indication that they are ready to die.

Increased urinary output


Based upon decreased intake, urinary output generally decreases in amount and frequency.

83.
Which of the following terms best describes a living will?

Medical directive
The living will is a type of advance medical directive in which the individual of sound mind
documents treatment preferences.

Proxy directive
A proxy directive is the appointment and authorization of another individual to make medical
decisions on behalf of the person who created an advance directive when he/she is no loner able
to speak for him/herself.

Health care power of attorney


Health care power of attorney is a legal document that enables the signer to designate another
individual to make health care decisions on his/her behalf when he/she is unable to do so.
Durable power of attorney for health
A durable power of attorney for health care is a legal document that enables the signer to
designate another individual to make health care decisions on his/her behalf when he/she is
unable to do so.

84.
A malignant tumor

gains access to the blood and lymphatic channels.


By this mechanism, the tumor metastasizes to other areas of the body.

demonstrates cells that are well-differentiated.


Cells of malignant tumors are undifferentiated.

is usually slow growing.


Malignant tumors demonstrate variable rate of growth; however, the more anaplastic the tumor,
the faster its growth.

grows by expansion.
A malignant tumor grows at the periphery and sends out processes that infiltrate and destroy
surrounding tissues.

85.
Which of the following classes of antineoplastic agents is cell--cycle-specific?

Antimetabolites (5-FU)
Antimetabolites are cell--cycle-specific (S phase).

Antitumor antibiotics (bleomycin)


Antitumor antibiotics are cell-cycle nonspecific.

Alkylating agents (cisplatin)


Alkylating agents are cell-cycle nonspecific.

Nitrosureas (carmustine)
Nitrosureas are cell-cycle nonspecific.

86.
Regarding the surgical patient, which of the following terms refers to the period of time that
constitutes the surgical experience?

Perioperative phase
Perioperative period includes the preoperative, intraoperative, and postoperative phases.
Preoperative phase
Preoperative phase is the period of time from when the decision for surgical intervention is made
to when the patient is transferred to the operating room table.

Intraoperative phase
Intraoperataive phase is the period of time from when the patient is transferred to the operating
room table to when he or she is admitted to the postanesthesia care unit.

Postoperative phase
Postoperative phase is the period of time that begins with the admission of the patient to the
postanesthesia care unit and ends after a follow-up evaluation in the clinical setting or home.

87.
When the indication for surgery is without delay, the nurse recognizes that the surgery will be
classified as

emergency.
Emergency surgery means that the patient requires immediate attention and the disorder may be
life-threatening.

urgent.
Urgent surgery means that the patient requires prompt attention within 24-30 hours.

required.
Required surgery means that the patient needs to have surgery, and it should be planned within a
few weeks or months.

elective.
Elective surgery means that there is an indication for surgery, but failure to have surgery will not
be catastrophic.

88.
When a person with a history of chronic alcoholism is admitted to the hospital for surgery, the
nurse anticipates that the patient may show signs of alcohol withdrawal delirium during which
time period?

Up to 72 hours after alcohol withdrawal


Alcohol withdrawal delirium is associated with a significant mortality rate when it occurs
postoperatively.

Immediately upon admission


Onset of symptoms depends upon time of last consumption of alcohol.
Upon awakening in the post-anesthesia care unit
Onset of symptoms depends upon time of last consumption of alcohol.

Up to 24 hours after alcohol withdrawal


Twenty-four hours is too short a time frame to consider alcohol withdrawal delirium no longer a
threat to a chronic alcoholic.

89.
Which of the following categories of medications may result in seizure activity if withdrawn
suddenly?

Tranquilizers
Abrupt withdrawal of tranquilizers may result in anxiety, tension, and even seizures if withdrawn
suddenly.

Adrenal corticosteroids
Abrupt withdrawal of steroids may precipitate cardiovascular collapse.

Antidepressants
Monoamine oxidase inhibitors increase the hypotensive effects of anesthetics.

Diuretics
Thiazide diuretics may cause excessive respiratory depression during anesthesia due to an
associated electrolyte imbalance.

90.
When the patient is encouraged to concentrate on a pleasant experience or restful scene, the
cognitive coping strategy being employed by the nurse is

imagery.
Imagery has proven effective for oncology patients.

optimistic self-recitation.
Optimistic self-recitation is practiced when the patient is encouraged to recite optimistic thoughts
such as “I know all will go well.”

distraction.
Distraction is employed when the patient is encouraged to think of an enjoyable story or recite a
favorite poem.

progressive muscular relaxation.


Progressive muscular relaxation requires contracting and relaxing muscle groups and is a physical
coping strategy as opposed to cognitive.
91.
According to the American Society of Anesthesiology Physical Status Classification System, a
patient with severe systemic disease that is not incapacitating is noted to have physical status
classification

P3
Classification P3 patients are those who have compensated heart failure, cirrhosis, or poorly
controlled diabetes, for example.

P4
Classification P4 patients have an incapacitating systemic disease that is a constant threat to life.

P1
Classification P1 refers to a normal healthy patient

P2
Classification P2 reflects a patient with mild systemic disease

92.
Which stage of anesthesia is termed surgical anesthesia?

III
With proper administration of the anesthetic, this stage may be maintained for hours.

I
Stage I is beginning anesthesia, as the patient breathes in the anesthetic mixture and experiences
warmth, dizziness, and a feeling of detachment.

II
Stage II is the excitement stage, which may be characterized by struggling, singing, laughing, or
crying.

IV
Stage IV is a stage of medullary depression and is reached when too much anesthesia has been
administered.

93.
Fentanyl (Sublimaze) is categorized as which type of intravenous anesthetic agent?

Neuroleptanalgesic
Fentanyl is 75-100 times more potent than morphine and has about 25% of the duration of
morphine (IV).
Tranquilizer
Examples of tranquilizers include midazolam (Versed) and diazepam (Valium).

Opioid
Opioids include morphine and meperidine hydrochloride (Demerol).

Dissociative agent
Ketamine is a dissociative agent.

94
Which of the following manifestations is often the earliest sign of malignant hyperthermia?

Tachycardia (heart rate above 150 beats per minute)


Tachycardia is often the earliest sign of malignant hyperthermia.

Hypotension
Hypotension is a later sign of malignant hyperthermia.

Elevated temperature
The rise in temperature is actually a late sign that develops rapidly.

Oliguria
Scant urinary output is a later sign of malignant hyperthermia.

95.
Which of the following terms is used to refer to protrusion of abdominal organs through the
surgical incision?

Evisceration
Evisceration is a surgical emergency.

Hernia
A hernia is a weakness in the abdominal wall.

Dehiscence
Dehiscence refers to partial or complete separation of wound edges.

Erythema
Erythema refers to redness of tissue.

96.
When the method of wound healing is one in which wound edges are not surgically approximated
and integumentary continuity is restored by granulations, the wound healing is termed
second intention healing.
When wounds dehisce, they will be allowed to heal by secondary intention.

primary intention healing.


Primary or first intention healing is the method of healing in which wound edges are surgically
approximated and integumentary continuity is restored without granulating.

first intention healing.


Primary or first intention healing is the method of healing in which wound edges are surgically
approximated and integumentary continuity is restored without granulating.

third intention healing.


Third intention healing is a method of healing in which surgical approximation of wound edges is
delayed and integumentary continuity is restored by bringing apposing granulations together.

97.
The nurse recognizes which of the following signs as typical of the patient in shock?

Rapid, weak, thready pulse


Pulse increases as the body tries to compensate.

Flushed face
Pallor is an indicator of shock.

Warm, dry skin


Skin is generally cool and moist in shock.

Increased urine output


Usually, a low blood pressure and concentrated urine are observed in the patient in shock.

98.
When the nurse observes that the postoperative patient demonstrates a constant low level of
oxygen saturation, although the patient's breathing appears normal, the nurse identifies that the
patient may be suffering which type of hypoxemia?

Subacute
Supplemental oxygen may be indicated.

Hypoxic
Hypoxic hypoxemia results from inadequate breathing.

Episodic
Episodic hypoxemia develops suddenly, and the patient may be at risk for myocardial ischemia,
cerebral dysfunction, and cardiac arrest.

Anemic
Anemic hypoxemia results from blood loss during surgery.

99.
When the surgeon performs an appendectomy, the nurse recognizes that the surgical category will
be identified as

clean contaminated.
Clean-contaminated cases are those with a potential, limited source for infection, the exposure to
which, to a large extent, can be controlled.

clean.
Clean cases are those with no apparent source of potential infection.

contaminated.
Contaminated cases are those that contain an open and obvious source of potential infection.

dirty.
A traumatic wound with foreign bodies, fecal contamination, or purulent drainage would be
considered a dirty case.

100.
Which of the following terms is used to describe inability to breathe easily except in an upright
position?

Orthopnea
Patients with orthopnea are placed in a high Fowler's position to facilitate breathing.

Dyspnea
Dyspnea refers to labored breathing or shortness of breath.

Hemoptysis
Hemoptysis refers to expectoration of blood from the respiratory tract.

Hypoxemia
Hypoxemia refers to low oxygen levels in the blood.

101.
In relation to the structure of the larynx, the cricoid cartilage is

the only complete cartilaginous ring in the larynx.


The cricoid cartilage is located below the thyroid cartilage.

used in vocal cord movement with the thyroid cartilage.


The arytenoid cartilages are used in vocal cord movement with the thyroid cartilage.

the largest of the cartilage structures.


The thyroid cartilage is the largest of the cartilage structures; part of it forms the Adam's apple.

the valve flap of cartilage that covers the opening to the larynx during swallowing.
The epiglottis is the valve flap of cartilage that covers the opening to the larynx during
swallowing.

102.
Which respiratory volume is the maximum volume of air that can be inhaled after maximal
expiration?

Inspiratory reserve volume


Inspiratory reserve volume is normally 3000 mL.

Tidal volume
Tidal volume is the volume of air inhaled and exhaled with each breath.

Expiratory reserve volume


Expiratory reserve volume is the maximum volume of air that can be exhaled forcibly after a
normal exhalation.

Residual volume
Residual volume is the volume of air remaining in the lungs after a maximum exhalation.

103.
The individual who demonstrates displacement of the sternum is described as having a

pigeon chest.
Pigeon chest may occur with rickets, Marfan's syndrome, or severe kyphoscoliosis.

barrel chest.
A barrel chest is seen in patients with emphysema as a result of over-inflation of the lungs.

funnel chest.
A funnel chest occurs when there is a depression in the lower portion of the sternum.

kyphoscoliosis.
Kyphoscoliosis is characterized by elevation of the scapula and a corresponding S-shaped spine.
104.
When the nurse auscultates chest sounds that are harsh and cracking, sounding like two pieces of
leather being rubbed together, she records her finding as

pleural friction rub.


A pleural friction rub is heard secondary to inflammation and loss of lubricating pleural fluid.

crackles.
Crackles are soft, high-pitched, discontinuous popping sounds that occur during inspiration.

sonorous wheezes.
Sonorous wheezes are deep, low-pitched rumbling sounds heard primarily during expiration.

sibilant wheezes.
Sibilant wheezes are continuous, musical, high-pitched, whistle-like sounds heard during
inspiration and expiration.

105.
Which of the following terms is used to describe hemorrhage from the nose?

Epistaxis
Epistaxis is due to rupture of tiny, distended vessels in the mucous membrane of any area of the
nose.

Xerostomia
Xerostomia refers to dryness of the mouth.

Rhinorrhea
Rhinorrhea refers to drainage of a large amount of fluid from the nose.

Dysphagia
Dysphagia refers to difficulties in swallowing.

106.
The herpes simplex virus (HSV-1), which produces a cold sore (fever blister), has an incubation
period of

2-12 days.
HSV-1 is transmitted primarily by direct contact with infected secretions.

0-3 months.
The time period 0-3 months exceeds the incubation period.
20-30 days.
The time period 20-30 days exceeds the incubation period.

3-6 months.
The time period 3-6 months exceeds the incubation period.

107.
Another term for clergyman's sore throat is

chronic granular pharyngitis.


In clergyman's sore throat, the pharynx is characterized by numerous swollen lymph follicles.

aphonia.
Aphonia refers to the inability to use one's voice.

atrophic pharyngitis.
Atrophic pharyngitis is characterized by a membrane that is thin, white, glistening, and at times
wrinkled.

hypertrophic pharyngitis.
Hypertrophic pharyngitis is characterized by general thickening and congestion of the pharyngeal
mucous membrane.

108.
Which type of sleep apnea is characterized by lack of airflow due to pharyngeal occlusion?

Obstructive
Obstructive sleep apnea occurs usually in men, especially those who are older and overweight.

Simple
Types of sleep apnea do not include a simple characterization.

Mixed
Mixed sleep apnea is a combination of central and obstructive apnea with one apneic episode.

Central
In central sleep apnea, the patient demonstrates simultaneous cessation of both airflow and
respiratory movements.

109.
When the patient who has undergone laryngectomy suffers wound breakdown, the nurse monitors
him very carefully because he is identified as being at high risk for
carotid artery hemorrhage.
The carotid artery lies close to the stoma and may rupture from erosion if the wound does not
heal properly.

pulmonary embolism.
Pulmonary embolism is associated with immobility.

dehydration.
Dehydration may lead to poor wound healing and breakdown.

pneumonia.
Pneumonia is a risk for any postoperative patient.

110.
Which of the following terms refers to lung tissue that has become more solid in nature due to a
collapse of alveoli or infectious process?

Consolidation
Consolidation occurs during an infectious process such as pneumonia.

Atelectasis
Atelectasis refers to collapse or airless condition of the alveoli caused by hypoventilation,
obstruction to the airways, or compression.

Bronchiectasis
Bronchiectasis refers to chronic dilation of a bronchi or bronchi in which the dilated airway
becomes saccular and a medium for chronic infection.

Empyema
Empyema refers to accumulation of purulent material in the pleural space.

111.
Which of the following community-acquired pneumonias demonstrates the highest occurrence
during summer and fall?

Legionnaires' disease
Legionnaires' disease accounts for 15% of community-acquired pneumon
Posted by StuffedNurse (co-author) at 8:32 PM
Wednesday, January 23, 2008

StuffedNurse : NP1 practice exam

NURSING PRACTICE I – Foundation of PROFESSIONAL Nursing Practice

SITUATIONAL

Situation 1 – Mr. Ibarra is assigned to the triage area and while on duty, he assesses the condition
of Mrs. Simon who came in with asthma. She has difficulty breathing and her respiratory rate is
40 per minute. Mr. Ibarra is asked to inject the client epinephrine 0.3 mg subcutaneously.

1. The indication for epinephrine injection for Mrs. Simon is to:

A. Reduce anaphylaxis
B. Relieve hypersensitivity to allergen
C. Relieve respiratory distress due to bronchial spasm
D. Restore client’s cardiac rhythm

2. When preparing the epinephrine injection from an ampule, the nurse initially:

A. Taps the ampule at the top to allow fluid to flow to the base of the ampule
B. Checks expiration date of the medication ampule
C. Removes needle cap of syringe and pulls plunger to expel air
D. Breaks the neck of the ampule with a gauze wrapped around it
3. Mrs. Simon is obese. When administering a subcutaneous injection to an obese patient, it is
best for the nurse to:

A. Inject needle at a 15 degree angle over the stretched skin of the client
B. Pinch skin at the injection site and use airlock technique
C. Pull skin of patient down to administer the drug in a Z track
D. Spread skin or pinch at the injection site and inject needle at a 45-90 degree angle

4. When preparing for a subcutaneous injection, the proper size of syringe and needle would be:

A. Syringe 3-5 ml and needle gauge 21 to 23


B. Tuberculin syringe 1 ml with needle gauge 26 or 27
C. Syringe 2 ml and needle gauge 22
D. Syringe 1-3 ml and needle gauge 25 to 27

5. The rationale for giving medications through the subcutaneous route is:

A. There are many alternative sites for subcutaneous injection


B. Absorption time of the medicine is slower
C. There are less pain receptors in this area
D. The medication can be injected while the client is in any position

Situation 2 – The use of massage and meditation to help decrease stress and pain have been
strongly recommended based on documented testimonials.

6. Martha wants to do a study on this topic: “Effects of massage and meditation on stress and
pain”. The type of research that best suits this topic is:

A. Applied research
B. Qualitative research
C. Basic research
D. Quantitative research

7. The type of research design that does not manipulate independent variable is:

A. Experimental design
B. Quasi-experimental design
C. Non-experimental design
D. Quantitative design

8. This research topic has the potential to contribute to nursing because it seeks to

A. include new modalities of care


B. resolve a clinical problem
C. clarify an ambiguous modality of care
D. enhance client care

9. Martha does review of related literature for the purpose of

A. determine statistical treatment of data research


B. gathering data about what is already known or unknown about the problem
C. to identify if problem can be replicated
D. answering the research question

10. Client’s rights should be protected when doing research using human subjects. Martha
identifies these rights as follows EXCEPT:

A. right of self-determination
B. right to compensation
C. right of privacy
D. right not to be harmed

Situation 3 – Richard has a nursing diagnosis of ineffective airway clearance related to excessive
secretions and is at risk for infection because of retained secretions. Part of Nurse Mario’s nursing
care plan is to loosen and remove excessive secretions in the airway.

11. Mario listens to Richard’s bilateral sounds and finds that congestion is in the upper lobes of
the lungs. The appropriate position to drain the anterior and posterior apical segments of the lungs
when Mario does percussion would be:

A. Client lying on his back then flat on his abdomen on Trendelenburg position
B. Client seated upright in bed or on a chair then leaning forward in sitting position then flat on
his back and on his abdomen
C. Client lying flat on his back and then flat on his abdomen
D. Client lying on his right then left side on Trendelenburg position

12. When documenting outcome of Richard’s treatment Mario should include the following in his
recording EXCEPT:

A. Color, amount and consistency of sputum


B. Character of breath sounds and respiratory rate before and after procedure
C. Amount of fluid intake of client before and after the procedure
D. Significant changes in vital signs

13. When assessing Richard for chest percussion or chest vibration and postural drainage, Mario
would focus on the following EXCEPT:

A. Amount of food and fluid taken during the last meal before treatment
B. Respiratory rate, breath sounds and location of congestion
C. Teaching the client’s relatives to perform the procedure
D. Doctor’s order regarding position restrictions and client’s tolerance for lying flat
14. Mario prepares Richard for postural drainage and percussion. Which of the following is a
special consideration when doing the procedure?

A. Respiratory rate of 16 to 20 per minute


B. Client can tolerate sitting and lying positions
C. Client has no signs of infection
D. Time of last food and fluid intake of the client

15. The purpose of chest percussion and vibration is to loosen secretions in the lungs. The
difference between the procedures is:

A. Percussion uses only one hand while vibration uses both hands
B. Percussion delivers cushioned blows to the chest with cupped palms while vibration gently
shakes secretion loose on the exhalation cycle
C. In both percussion and vibration the hands are on top of each other and hand action is in tune
with client’s breath rhythm
D. Percussion slaps the chest to loosen secretions while vibration shakes the secretions along with
the inhalation of air

Situation 4 – A 61 year old man, Mr. Regalado, is admitted to the private ward for observation
after complaints of severe chest pain. You are assigned to take care of the client.

16. When doing an initial assessment, the best way for you to identify the client’s priority
problem is to:

A. Interview the client for chief complaints and other symptoms


B. Talk to the relatives to gather data about history of illness
C. Do auscultation to check for chest congestion
D. Do a physical examination while asking the client relevant questions

17. Upon establishing Mr. Regalado’s nursing needs, the next nursing approach would be to:

A. Introduce the client to the ward staff to put the client and family at ease
B. Give client and relatives a brief tour of the physical set up the unit
C. Take his vital signs for a baseline assessment
D. Establish priority needs and implement appropriate interventions

18. Mr. Regalado says he has “trouble going to sleep”. In order to plan your nursing intervention
you will:

A. Observe his sleeping patterns in the next few days


B. Ask him what he means by this statement
C. Check his physical environment to decrease noise level
D. Take his blood pressure before sleeping and upon waking up
19. Mr. Regalado’s lower extremities are swollen and shiny. He has pitting pedal edema. When
taking care of Mr. Regalado, which of the following interventions would be the most appropriate
immediate nursing approach?

A. Moisturize lower extremities to prevent skin irritation


B. Measure fluid intake and output to decrease edema
C. Elevate lower extremities for postural drainage
D. Provide the client a list of food low in sodium

20. Mr. Regalado will be discharged from your unit within the hour. Nursing actions when
preparing a client for discharge include all EXCEPT:

A. Making a final physical assessment before client leaves the hospital


B. Giving instructions about his medication regimen
C. Walking the client to the hospital exit to ensure his safety
D. Proper recording of pertinent data

Situation 5 – Nancy, mother of 2 young kids, 36 years old, had a mammogram and was told that
she has breast cysts and that she may need surgery. This causes her anxiety as shown by increase
in her pulse and respiratory rate, sweating and feelings of tension.

21. Considering her level of anxiety, the nurse can best assist Nancy by:

A. Giving her activities to divert her attention


B. Giving detailed explanations about the treatments she will undergo
C. Preparing her and her family in case surgery is not successful
D. Giving her clear but brief information at the level of her understanding

22. Nancy blames God for her situation. She is easily provoked to tears and wants to be left alone,
refusing to eat or talk to her family. A religious person before, she now refuses to pray or go to
church stating that God has abandoned her. The nurse understands that Nancy is grieving for her
self and is in the stage of:

A. bargaining
B. denial
C. anger
D. acceptance

23. The nurse visits Nancy and prods her to eat her food. Nancy replies “what’s the use? My time
is running out.” The nurse’s best response would be:

A. “The doctor ordered full diet for you so that you will be strong for surgery”
B. “I understand how you feel but you have to try for your children’s sake”
C. “Have you told your doctor how you feel? Are you changing your mind about your surgery?”
D. “You sound like you are giving up.”
24. The nurse feels sad about Nancy’s illness and tells her head nurse during the end of shift
endorsement that “it’s unfair for Nancy to have cancer when she is still so young and with two
kids”. The best response of the head nurse would be:

A. Advise the nurse to “be strong and learn to control her feelings”
B. Assign the nurse to another client to avoid sympathy for the client
C. Reassure the nurse that the client has hope if she goes through all treatments prescribed for her
D. Ask the other nurses what they feel about the patient to find out if they share the same feelings

25. Realizing that she feels angry about Nancy’s condition, the nurse learns that being self-aware
is a conscious process that she should do in any situation like this because:

A. This is a necessary part of the nurse – client relationship process


B. The nurse is a role model for the client and should be strong
C. How the nurse thinks and feels affect her actions towards her client and her work
D. The nurse has to be therapeutic at all times and should not be affected

Situation 6 – Mrs. Seva, 52 years old, asks you about possible problems regarding her elimination
now that she is in the menopausal stage.

26. Instruction on health promotion regarding urinary elimination is important. Which would you
include?

A. Hold urine as long as she can before emptying the bladder to strengthen her sphincter muscles
B. If burning sensation is experienced while voiding, drink pineapple juice
C. After urination, wipe from anal area up towards the pubis
D. Tell client to empty the bladder at each voiding

27. Mrs. Seva also tells the nurse that she is often constipated. Because she is aging, what
physical changes predispose her to constipation?

A. inhibition of the parasympathetic reflex


B. weakness of sphincter muscles of anus
C. loss of tone of the smooth muscles of the colon
D. decreased ability to absorb fluids in the lower intestines

28. The nurse understands that one of these factors contributes to constipation:

A. excessive exercise
B. high fiber diet
C. no regular time for defecation daily
D. prolonged use of laxatives

29. Mrs. Seva talks about fear of being incontinent due to a prior experience of dribbling urine
when laughing or sneezing and when she has a full bladder. Your most appropriate instruction
would be to:
A. tell client to drink less fluids to avoid accidents
B. instruct client to start wearing thin adult diapers
C. ask the client to bring change of underwear “just in case”
D. teach client pelvic exercise to strengthen perineal muscles

30. Mrs. Seva asked for instructions for skin care for her mother who has urinary incontinence
and is almost always in bed. Your instruction would focus on prevention of skin irritation and
breakdown by:

A. Using thick diapers to absorb urine well


B. Drying the skin with baby powder to prevent or mask the smell of ammonia
C. Thorough washing, rising and drying of skin area that get wet with urine
D. Making sure that linen are smooth and dry at all times

Situation 7 – Using Maslow’s need theory, Airway, Breathing and Circulation are the
physiological needs vital to life. The nurse’s knowledge and ability to identify and immediately
intervene to meet these needs is important to save lives.

31. Which of these clients has a problem with the transport of oxygen from the lungs to the
tissues:

A. Carol with tumor in the brain


B. Theresa with anemia
C. Sonnyboy with a fracture in the femur
D. Brigitte with diarrhea

32. You noted from the lab exams in the chart of M. Santos that he has reduced oxygen in the
blood. This condition is called:

A. Cyanosis
B. Hypoxia
C. Hypoxemia
D. Anemia

33. You will do nasopharyngeal suctioning on Mr. Abad. Your guide for the length of insertion of
the tubing for an adult would be:

A. tip of the nose to the base of the neck


B. the distance from the tip of the nose to the middle of the neck
C. the distance from the tip of the nose to the tip of the ear lobe
D. eight to ten inches

34. While doing nasopharyngeal suctioning on Mr. Abad, the nurse can avoid trauma to the area
by:
A. Apply suction for at least 20-30 seconds each time to ensure that all secretions are removed
B. Using gloves to prevent introduction of pathogens to the respiratory system
C. Applying no suction while inserting the catheter
D. Rotating catheter as it is inserter with gentle suction

35. Myrna has difficulty breathing when on her back and must sit upright in bed to breath
effectively and comfortably. The nurse documents this condition as:

A. Apnea
B. Orthopnea
C. Dyspnea
D. Tachypnea

Situation 8 – You are assigned to screen for hypertension. Your task is to take blood pressure
readings and you are informed about avoiding the common mistakes in BP taking that lead to
false or inaccurate blood pressure readings.

36. When taking blood pressure reading the cuff should be:

A. deflated fully then immediately start second reading for same client
B. deflated quickly after inflating up to 180 mmHg
C. large enough to wrap around upper arm of the adult client 1 cm above brachial artery
D. inflated to 30 mmHg above the estimated systolic BP based on palpation of radial or bronchial
artery

37. Chronic Obstructive Pulmonary Disease (COPD) in one of the leading causes of death world
wide and is a preventable disease. The primary cause of COPD is

A. tobacco hack
B. bronchitis
C. asthma
D. cigarette smoking

38. In your health education class for clients with diabetes you teach them the areas for control of
Diabetes which include all EXCEPT

A. regular physical activity


B. thorough knowledge of foot care
C. prevention nutrition
D. proper nutrition

39. You teach your clients the difference between, Type I (IDDM) and Type II (NDDM) diabetes.
Which of the following is true?

A. both types diabetes mellitus clients are all prone to developing ketosis
B. Type II (NIDDM) is more common and is also preventable compared to Type I (IDDM)
diabetes which is genetic in etiology
C. Type I (IIDM) is characterized by fasting hyperglycemia
D. Type II (NIDDM) is characterized by abnormal immune response

40. Lifestyle-related diseases in general share areas common risk factors. These are the following
except:

A. physical activity
B. smoking
C. genetics
D. nutrition

Situation 9 – Nurse Rivera witnesses a vehicular accident near the hospital where she works. She
decides to get involved and help the victims of the accident

41. Her priority nursing action would be to:

A. Assess damage to property


B. Assist in the police investigation since she is a witness
C. Report the incident immediately to the local police authorities
D. Assess the extent of injuries incurred by the victims of the accident

42. Priority attention should be given to which of these clients?

A. Linda who shows severe anxiety due to trauma of the accident


B. Ryan who has chest injury, is pale and with difficulty breathing
C. Noel who has lacerations on the arms with mild bleeding
D. Andy whose left ankle swelled and has some abrasions

43. In the emergency room, Nurse Rivera is assigned to attend to the client with lacerations on the
arms. While assessing the extent of the wound the nurse observes that the wound is now starting
to bleed profusely. The most immediate nursing action would be to:

A. Apply antiseptic to prevent infection


B. Clean the wound vigorously of contaminants
C. Control and reduce bleeding of the wound
D. Bandage the wound and elevate the arm

44. The nurse applies dressing on the bleeding site. This intervention is done to:

A. Reduce the need to change dressing frequently


B. Allow the pus to surface faster
C. Protect the wound from microorganisms in the air
D. Promote hemostasis

45. After the treatment, the client is sent home and asked to come back for follow-up care. Your
responsibilities when the client is to be discharged include the following EXCEPT:

A. Encouraging the client to go to the outpatient clinic for follow up care


B. Accurate recording of treatment done and instructions given to client
C. Instructing the client to see you after discharge for further assistance
D. Providing instructions regarding wound care

Situation 10 – While working in the clinic, a new client, Geline, 35 years old, arrives for her
doctor’s appointment. As the clinic nurse, you are to assist the client fill up forms, gather data and
make an assessment.

46. The purpose of your initial nursing interview is to:

A. Record pertinent information in the client’s chart for health team to read
B. Assist the client find solutions to he her health concerns
C. Understand her lifestyle, health needs and possible problems to develop a plan of care
D. Make nursing diagnoses for identified health problems

47. While interviewing Geline, she starts to moan and doubles up in pain. She tells you that this
pain occurs about an hour after taking black coffee without breakfast for a few weeks now. You
will record this as follows:

A. Claims to have abdominal pains after intake of coffee unrelieved by analgesics


B. After drinking coffee, the client experienced severe abdominal pain
C. Client complained of intermittent abdominal pain an hour after drinking coffee
D. Client reported abdominal pain an hour after drinking black coffee for three weeks now.

48. Geline tells you that she drinks black coffee frequently within the day to “have energy and be
wide awake” and she eats nothing for breakfast and eats strictly vegetable salads for lunch and
dinner to lose weight. She has lost weight during the past two weeks. In planning a healthy
balanced diet with Geline, you will:

A. Start her off with a cleansing diet to free her body of toxins then change to a vegetarian diet
and drink plenty of fluids
B. Plan a high protein diet, low carbohydrate diet for her considering her favorite food.
C. Instruct her to attend classes in nutrition to find food rich in complex carbohydrates to
maintain daily high energy level.
D. Discuss with her the importance of eating a variety of food from major food groups with
plenty of fluids.

49. Geline tells you that she drinks 4-5 cups of black coffee and diet cola drinks. She also smokes
up to a pack of cigarettes daily. She confesses that she is in her 2nd month of pregnancy but does
not want to become fat that is why she limits her food intake. You warn or caution her about
which of the following?

A. Caffeine products affect the central nervous system and may cause the mother to have a
“nervous breakdown”
B. Malnutrition and its possible effects on growth and development problems in the unborn fetus
C. Caffeine causes a stimulant effect on both mother and the baby
D. Studies show conclusively that caffeine causes mental retardation

50. Your health education plan for Geline stresses proper diet for a pregnant woman and the
prevention of non-communicable diseases that are influenced by her lifestyle. These include the
following EXCEPT:

A. Cardiovascular diseases
B. Cancer
C. Diabetes Mellitus
D. Osteoporosis

answer key np1

note: guys pacensya na dont have time yet to post the rationales, super haba most of them, maybe
later this month or maybe i'll post ung mga sources na lang na books so you can read them on
your own.(^^,)

answers na lang muna for now po..

-diannemaydee

NURSING PRACTICE I – Foundation of PROFESSIONAL Nursing Practice

SITUATIONAL

Situation 1 – Mr. Ibarra is assigned to the triage area and while on duty, he assesses the condition
of Mrs. Simon who came in with asthma. She has difficulty breathing and her respiratory rate is
40 per minute. Mr. Ibarra is asked to inject the client epinephrine 0.3 mg subcutaneously.

1. The indication for epinephrine injection for Mrs. Simon is to:

Relieve respiratory distress due to bronchial spasm

2. When preparing the epinephrine injection from an ampule, the nurse initially:

Checks expiration date of the medication ampule

3. Mrs. Simon is obese. When administering a subcutaneous injection to an obese patient, it is


best for the nurse to:

Spread skin or pinch at the injection site and inject needle at a 45-90 degree angle

4. When preparing for a subcutaneous injection, the proper size of syringe and needle would be:

Syringe 1-3 ml and needle gauge 25 to 27


5. The rationale for giving medications through the subcutaneous route is:

Absorption time of the medicine is slower

Situation 2 – The use of massage and meditation to help decrease stress and pain have been
strongly recommended based on documented testimonials.

6. Martha wants to do a study on this topic: “Effects of massage and meditation on stress and
pain”. The type of research that best suits this topic is:

Qualitative research

7. The type of research design that does not manipulate independent variable is:

Non-experimental design

8. This research topic has the potential to contribute to nursing because it seeks to

enhance client care

9. Martha does review of related literature for the purpose of

A. athering data about what is already known or unknown about the problem

10. Client’s rights should be protected when doing research using human subjects. Martha
identifies these rights as follows EXCEPT:

right to compensation

Situation 3 – Richard has a nursing diagnosis of ineffective airway clearance related to excessive
secretions and is at risk for infection because of retained secretions. Part of Nurse Mario’s nursing
care plan is to loosen and remove excessive secretions in the airway.

11. Mario listens to Richard’s bilateral sounds and finds that congestion is in the upper lobes of
the lungs. The appropriate position to drain the anterior and posterior apical segments of the lungs
when Mario does percussion would be:

Client seated upright in bed or on a chair then leaning forward in sitting position then flat on his
back and on his abdomen

12. When documenting outcome of Richard’s treatment Mario should include the following in his
recording EXCEPT:

Amount of fluid intake of client before and after the procedure


13. When assessing Richard for chest percussion or chest vibration and postural drainage, Mario
would focus on the following EXCEPT:

Teaching the client’s relatives to perform the procedure

14. Mario prepares Richard for postural drainage and percussion. Which of the following is a
special consideration when doing the procedure?

Client can tolerate sitting and lying positions

15. The purpose of chest percussion and vibration is to loosen secretions in the lungs. The
difference between the procedures is:

Percussion delivers cushioned blows to the chest with cupped palms while vibration gently
shakes secretion loose on the exhalation cycle

Situation 4 – A 61 year old man, Mr. Regalado, is admitted to the private ward for observation
after complaints of severe chest pain. You are assigned to take care of the client.

16. When doing an initial assessment, the best way for you to identify the client’s priority
problem is to:

Do a physical examination while asking the client relevant questions

17. Upon establishing Mr. Regalado’s nursing needs, the next nursing approach would be to:

Establish priority needs and implement appropriate interventions

18. Mr. Regalado says he has “trouble going to sleep”. In order to plan your nursing intervention
you will:

Ask him what he means by this statement

19. Mr. Regalado’s lower extremities are swollen and shiny. He has pitting pedal edema. When
taking care of Mr. Regalado, which of the following interventions would be the most appropriate
immediate nursing approach?

Elevate lower extremities for postural drainage

20. Mr. Regalado will be discharged from your unit within the hour. Nursing actions when
preparing a client for discharge include all EXCEPT:

Walking the client to the hospital exit to ensure his safety

Situation 5 – Nancy, mother of 2 young kids, 36 years old, had a mammogram and was told that
she has breast cysts and that she may need surgery. This causes her anxiety as shown by increase
in her pulse and respiratory rate, sweating and feelings of tension.

21. Considering her level of anxiety, the nurse can best assist Nancy by:

Giving her clear but brief information at the level of her understanding

22. Nancy blames God for her situation. She is easily provoked to tears and wants to be left alone,
refusing to eat or talk to her family. A religious person before, she now refuses to pray or go to
church stating that God has abandoned her. The nurse understands that Nancy is grieving for her
self and is in the stage of:

anger

23. The nurse visits Nancy and prods her to eat her food. Nancy replies “what’s the use? My time
is running out.” The nurse’s best response would be:

“You sound like you are giving up.”

24. The nurse feels sad about Nancy’s illness and tells her head nurse during the end of shift
endorsement that “it’s unfair for Nancy to have cancer when she is still so young and with two
kids”. The best response of the head nurse would be:

Advise the nurse to “be strong and learn to control her feelings”

25. Realizing that she feels angry about Nancy’s condition, the nurse learns that being self-aware
is a conscious process that she should do in any situation like this because:

How the nurse thinks and feels affect her actions towards her client and her work

Situation 6 – Mrs. Seva, 52 years old, asks you about possible problems regarding her elimination
now that she is in the menopausal stage.

26. Instruction on health promotion regarding urinary elimination is important. Which would you
include?

Tell client to empty the bladder at each voiding

27. Mrs. Seva also tells the nurse that she is often constipated. Because she is aging, what
physical changes predispose her to constipation?

loss of tone of the smooth muscles of the colon

28. The nurse understands that one of these factors contributes to constipation:

prolonged use of laxatives


29. Mrs. Seva talks about fear of being incontinent due to a prior experience of dribbling urine
when laughing or sneezing and when she has a full bladder. Your most appropriate instruction
would be to:

teach client pelvic exercise to strengthen perineal muscles

30. Mrs. Seva asked for instructions for skin care for her mother who has urinary incontinence
and is almost always in bed. Your instruction would focus on prevention of skin irritation and
breakdown by:

Thorough washing, rising and drying of skin area that get wet with urine

Situation 7 – Using Maslow’s need theory, Airway, Breathing and Circulation are the
physiological needs vital to life. The nurse’s knowledge and ability to identify and immediately
intervene to meet these needs is important to save lives.

31. Which of these clients has a problem with the transport of oxygen from the lungs to the
tissues:

Theresa with anemia

32. You noted from the lab exams in the chart of M. Santos that he has reduced oxygen in the
blood. This condition is called:

Hypoxemia

33. You will do nasopharyngeal suctioning on Mr. Abad. Your guide for the length of insertion of
the tubing for an adult would be:

the distance from the tip of the nose to the tip of the ear lobe

34. While doing nasopharyngeal suctioning on Mr. Abad, the nurse can avoid trauma to the area
by:
Applying no suction while inserting the catheter

35. Myrna has difficulty breathing when on her back and must sit upright in bed to breath
effectively and comfortably. The nurse documents this condition as:

Orthopnea

Situation 8 – You are assigned to screen for hypertension. Your task is to take blood pressure
readings and you are informed about avoiding the common mistakes in BP taking that lead to
false or inaccurate blood pressure readings.

36. When taking blood pressure reading the cuff should be:
deflated fully then immediately start second reading for same client

37. Chronic Obstructive Pulmonary Disease (COPD) in one of the leading causes of death world
wide and is a preventable disease. The primary cause of COPD is

cigarette smoking

38. In your health education class for clients with diabetes you teach them the areas for control of
Diabetes which include all EXCEPT

prevention nutrition

39. You teach your clients the difference between, Type I (IDDM) and Type II (NDDM) diabetes.
Which of the following is true?

Type II (NIDDM) is more common and is also preventable compared to Type I (IDDM) diabetes
which is genetic in etiology

40. Lifestyle-related diseases in general share areas common risk factors. These are the following
except:

genetics

Situation 9 – Nurse Rivera witnesses a vehicular accident near the hospital where she works. She
decides to get involved and help the victims of the accident

41. Her priority nursing action would be to:

Assess the extent of injuries incurred by the victims of the accident

42. Priority attention should be given to which of these clients?

Ryan who has chest injury, is pale and with difficulty breathing

43. In the emergency room, Nurse Rivera is assigned to attend to the client with lacerations on the
arms. While assessing the extent of the wound the nurse observes that the wound is now starting
to bleed profusely. The most immediate nursing action would be to:

Bandage the wound and elevate the arm

44. The nurse applies dressing on the bleeding site. This intervention is done to:

Promote hemostasis

45. After the treatment, the client is sent home and asked to come back for follow-up care. Your
responsibilities when the client is to be discharged include the following EXCEPT:

Instructing the client to see you after discharge for further assistance

Situation 10 – While working in the clinic, a new client, Geline, 35 years old, arrives for her
doctor’s appointment. As the clinic nurse, you are to assist the client fill up forms, gather data and
make an assessment.

46. The purpose of your initial nursing interview is to:

Understand her lifestyle, health needs and possible problems to develop a plan of care

47. While interviewing Geline, she starts to moan and doubles up in pain. She tells you that this
pain occurs about an hour after taking black coffee without breakfast for a few weeks now. You
will record this as follows:

Client reported abdominal pain an hour after drinking black coffee for three weeks now.

48. Geline tells you that she drinks black coffee frequently within the day to “have energy and be
wide awake” and she eats nothing for breakfast and eats strictly vegetable salads for lunch and
dinner to lose weight. She has lost weight during the past two weeks. In planning a healthy
balanced diet with Geline, you will:

Discuss with her the importance of eating a variety of food from major food groups with plenty of
fluids.

49. Geline tells you that she drinks 4-5 cups of black coffee and diet cola drinks. She also smokes
up to a pack of cigarettes daily. She confesses that she is in her 2nd month of pregnancy but does
not want to become fat that is why she limits her food intake. You warn or caution her about
which of the following?

Malnutrition and its possible effects on growth and development problems in the unborn fetus

50. Your health education plan for Geline stresses proper diet for a pregnant woman and the
prevention of non-communicable diseases that are influenced by her lifestyle. These include the
following EXCEPT:

Cancer
Wednesday, January 23, 2008

StuffedNurse : NP2 practice exam

NURSING PRACTICE II – Community Health Nursing and Care of the


Mother and Child

SITUATIONAL

Situation 1 – Nurse Minette is an Independent Nurse Practitioner


following-up referred clients in their respective homes. Here she
handles a case of POSTPARTIAL MOTHER AND FAMILY focusing on
HOME CARE.

1. Nurse Minette needs to schedule a first home visit to OB client


Leah. When is a first home-care visit typically made?

A. Within 4 days after discharge


B. Within 24 hours after discharge
C. Within 1 hour after discharge
D. Within 1 week of discharge

2. Leah is developing constipation from being on bed rest. What


measures would you suggest she take to help prevent this?

A. Eat more frequent small meals instead of three large one daily
B. Walk for at least half an hour daily to stimulate peristalsis
C. Drink more milk, increased calcium intake prevents constipation
D. Drink eight full glasses of fluid such as water daily

3. If you were Minette, which of the following actions would alert you
that a new mother is entering a postpartal taking-hold phase?
A. She urges the baby to stay awake so that she can breast-feed him
or her
B. She tells you she was in a lot of pain all during labor
C. She says that she has not selected a name for the baby as yet
D. She sleeps as if exhausted from the effort of labor

4. At 6-week postpartum visit what should this postpartal mother’s


fundic height be?

A. Inverted and palpable at the cervix


B. Six fingerbreadths below umbilicus
C. No longer palpable on her abdomen
D. One centimeter above the symphysis pubis

5. This postpartal mother wants to loose the weight she gained in


pregnancy, so she is reluctant to increase her caloric intake for
breast-feeding. By how much should a lactating mother increase her
caloric intake during the first 6 months after birth?

A. 350 kcal/day
B. 500 kcal/day
C. 200 kcal/day
D. 1000 kcal/day

Situation 2 – As the CPE is applicable for all professional nurse, the


professional growth and development of Nurses with specialties shall
be addressed by a Specialty Certification Council. The following
questions apply to these special groups of nurses.

6. Which of the following serves as the legal basis and statute


authority for the Board of Nursing to promulgate measures to effect
the creation of a Specialty Certification Council and promulgate
professional development programs for this group of nurse-
professionals?
A. R.A. 7610
B. P.D. 223
C. R.A. 9173
D. R.A. 7164
7. By force of law, therefore, the PRC-Board of Nursing released
Resolution No. 14 Series of 1999 entitled: “Adoption of a Nursing
Specialty Certification Program and Creation of Nursing Specialty
Certification Council.” This rule-making power is called:

A. Quasi-Judicial Power
B. Regulatory Power
C. Quasi-Legislative Power
D. Executive/Promulgating Power

8. Under the PRC-Board of Nursing Resolution promulgating the


adoption of a Nursing Specialty Certification Program and Council,
which two (2) of the following serves as the strongest for its
enforcement?

(a) Advances made in Science and Technology have provided the


climate for specialization in almost all aspects of human endeavor;
and
(b) As necessary consequence, there has emerged a new concept
known as globalization which seeks to remove barriers in trade,
industry and services imposed by the national laws of countries all
over the world; and
(c) Awareness of this development should impel the nursing sector
to prepare our people in the services sector to meet the above
challenge; and
(d) Current trends of specialization in nursing practice recognized by
the International Council of Nurses (ICN) of which the Philippines is
a member for the benefit of the Filipino in terms of deepening and
refining nursing practice and enhancing the quality of nursing care.

A. b & c are strong justifications


B. a & b are strong justifications
C. a & c are strong justifications
D. a & d are strong justifications

9. Which of the following IS NOT a correct statement as regards


Specialty Certification?

A. The Board of Nursing intended to create the Nursing Specialty


Certification Program as a means of perpetuating the creation of an
elite force of Filipino Nurse Professionals.
B. The Board of Nursing shall oversee the administration of the NSCP
through the various Nursing Specialty Boards which will eventually
be created
C. The Board of Nursing at the time exercised their powers under
R.A. 7164 in order to adopt the creation of the Nursing Specialty
Certification Council and Program
D. The Board of Nursing consulted nursing leaders of national
nursing associations and other concerned nursing groups which later
decided to ask a special group of nurses of the program for nursing
specialty Certification.

10. The NSCC was created for the purpose of implementing the
Nursing Specialty policy under the direct supervision and
stewardship of the Board of Nursing. Who shall comprise the NSCC?

A. A Chairperson who is the current President of the APO; a member


from the Academe; and the last member coming from the Regulatory
Board
B. The chairperson and members of the Regulatory Board ipso facto
acts as the CPE Council
C. A Chairperson, chosen from among the Regulatory Board
members; a Vice Chairperson appointed by the BON at-large; two
other members also chosen at-large; and one representing the
consumer group;
D. A Chairperson who is the President of the Association from the
Academe; a member from the Regulatory Board; and the last member
coming from the APO
No answer..maybe a bonus question…

Situation 3 –Nurse Anna is a new BSN graduate and has just passed
her Licensure Examination for Nurses in the Philippines. She has
likewise been hired as a new Community Health Nurse in one of the
Rural Health Units in their City, which of the following conditions
may be acceptable TRUTHS applied to Community Health Nursing
Practice.

11. Which of the following is the primary focus of community health


nursing practice?

A. Cure of illnesses
B. Prevention of illnesses
C. Rehabilitation back to health
D. Promotion of health

12. In community health nursing, which of the following is our unit of


service as nurses?

A. The community
B. The extended members of every family
C. The individual members of the Barangay
D. The Family

13. A very important part of the Community Health Nursing


Assessment Process includes:

A. the application of professional judgment in estimating importance


of facts to family and community
B. evaluation structures and qualifications of health center team
C. coordination with other sectors in relation to health concerns
D. carrying out nursing procedures as per plan of action

14. In community health nursing it is important to take into account


the family health data coupled with an equally important need to
perform ocular inspection of the area as activities which are powerful
elements of:

A. evaluation
B. assessment
C. implementation
D. planning

15. The initial step in PLANNING process in order to engage in any


nursing project or activities at the community level involves

A. goal-setting
B. monitoring
C. evaluation of data
D. provision of data

Situation 4 – Please continue responding as a professional nurse in


these other health situations through the following questions.

16. Transmission of HIV from an infected individual to another


person occurs:

A. Most frequently in nurses with needlesticks


B. Only if there is a large viral load in the blood
C. Most commonly as a result of sexual contact
D. In all infants born to women with HIV infection

17. The medical record of a client reveals a condition in which the


fetus cannot pass through the maternal pelvis. The nurse interprets
this as:

A. Contracted pelvis
B. Maternal disproportion
C. Cervical insufficiency
D. Fetopelvic disproportion
18. The nurse would anticipate a cesarean birth for a client who has
which infection present at the onset of labor?

A. Herpes-simplex virus
B. Human papilloma virus
C. Hepatitis
D. Toxoplasmosis

19. After a vaginal examination, the nurse determines that the


client’s fetus is in an occiput posterior position. The nurse would
anticipate that the client will have:

A. A precipitous birth
B. Intense back pain
C. Frequent leg cramps
D. Nausea and vomiting

20. The rationales for using a prostaglandin gel for a client prior to
the induction of labor is to:

A. Soften and efface the cervix


B. Numb cervical pain receptors
C. Prevent cervical lacerations
D. Stimulate uterine contractions

Situation 5 – Nurse Lorena is a Family Planning and Infertility Nurse


Specialist and currently attends to FAMILY PLANNING CLIENTS AND
INFERTILE COUPLES. The following conditions pertain to meeting the
nursing needs of this particular population group.

21. Dina, 17 years old, asks you how a tubal ligation prevents
pregnancy. Which would be the best answer?

A. Prostaglandins released from the cut fallopian tubes can kill sperm
B. Sperm can not enter the uterus because the cervical entrance is
blocked.
C. Sperm can no longer reach the ova, because the fallopian tubes
are blocked
D. The ovary no longer releases ova as there is no where for them to
go.

22. The Dators are a couple undergoing testing for infertility.


Infertility is said to exist when:

A. a woman has no uterus


B. a woman has no children
C. a couple has been trying to conceive for 1 year
D. a couple has wanted a child for 6 months

23. Another client named Lilia is diagnosed as having endometriosis.


This condition interferes with fertility because:

A. endometrial implants can block the fallopian tubes


B. the uterine cervix becomes inflamed and swollen
C. the ovaries stop producing adequate estrogen
D. pressure on the pituitary leads to decreased FSH levels

24. Lilia is scheduled to have a hysterosalphingogram. Which of the


following instructions would you give her regarding this procedure?

A. She will not be able to conceive for 3 months after the procedure
B. The sonogram of the uterus will reveal any tumors present
C. Many women experience mild bleeding as an after effect
D. She may feel some cramping when the dye is inserted

25. Lilia’s cousin on the other hand, knowing nurse Lorena’s


specialization asks what artificial insemination by donor entails.
Which would be your best answer if you were Nurse Lorena?

A. Donor sperm are introduced vaginally into the uterus or cervix


B. Donor sperm are injected intra-abdominally into each ovary
C. Artificial sperm are injected vaginally to test tubal patency
D. The husband’s sperm is administered intravenously weekly

Situation 6 – There are other important basic knowledge in the


performance of our task as Community Health Nurse in relation to
IMMUNIZATION, these include:

26. The correct temperature to store vaccines in a refrigerator is:

A. between -4 deg C and +8 deg C


B. between 2 deg C and +8 deg C
C. between -8 deg C and 0 deg C
D. between -8 deg C and +4 deg C

27. Which of the following vaccines is not done by intramuscular


(IM) injection?

A. Measles vaccine
B. DPT
C. Hepa-B vaccine
D. Tetanus toxoids

28. This vaccine content is derived from RNA recombinants.

A. Measles
B. Tetanus toxoids
C. Hepatitis B vaccines
D. DPT

29. This is the vaccine needed before a child reaches one (1) year in
order for him/her to qualify as a :fully immunized child”.

A. DPT
B. Measles
C. Hepatitis B
D. BCG
30. Which of the following dose of tetanus toxoid is given to the
mother to protect her infant from neonatal tetanus and likewise
provide 10 years protection for the mother?

A. Tetanus toxoid 3
B. Tetanus toxoid 2
C. Tetanus toxoid 1
D. Tetanus toxoid 4

Situation 7 – Records contain those comprehensive descriptions of


patient’s health conditions and needs and at the same serve as
evidences of every nurse’s accountability in the care giving process.
Nursing records normally differ from institution to institution
nonetheless they follow similar patterns of meeting needs for
specific types of information. The following pertains to
documentation/records management.

31. This special form is used when the patient is admitted to the unit.
The nurse completes the information in this record particularly
his/her basic personal data, current illness, previous health history,
health history of the family, emotional profile, environmental history
as well as physical assessment together with nursing diagnosis on
admission. What do you call this record?

A. Nursing Kardex
B. Nursing Health History and Assessment Worksheet
C. Medicine and Treatment Record
D. Discharge Summary

32. These are sheets/forms which provide an efficient and time


saving way to record information that must be obtained repeatedly at
regular and/or short intervals of time. This does not replace the
progress notes; instead this record of information on vital signs,
intake and output, treatment, postoperative care, post partum care,
and diabetic regimen, etc. This is used whenever specific
measurements or observations are needed to be documented
repeatedly. What is this?

A. Nursing Kardex
B. Graphic Flow Sheets
C. Discharge Summary
D. Medicine and Treatment Record

33. These records show all medications and treatment provided on a


repeated basis. What do you call this record?

A. Nursing Health History and Assessment Worksheet


B. Discharge Summary
C. Nursing Kardex
D. Medicine and Treatment Record

34. This flip-over card is usually kept in a portable file at the Nurse’s
Station. It has 2-parts: the activity and treatment section and a
nursing care plan section. This carries information about basic
demographic data, primary medical diagnosis, current orders of the
physician to be carried out by the nurse, written nursing care plan,
nursing orders, scheduled tests and procedures, safety precautions
in patient care and factors related to daily living activities. This
record is used in the charge-of-shift reports or during the bedside
rounds or walking rounds. What record is this?

A. Discharge Summary
B. Medicine and Treatment Record
C. Nursing Health History and Assessment Worksheet
D. Nursing Kardex

35. Most nurses regard this conventional recording of the date, time,
and mode by which the patient leaves a healthcare unit but this
record includes importantly, directs of planning for discharge that
starts soon after the person is admitted to a healthcare institution. It
is accepted that collaboration or multidisciplinary involvement (of all
members of the health team) in discharge results in comprehensive
care. What do you call this?

A. Discharge Summary
B. Nursing Kardex
C. Medicine and Treatment Record
D. Nursing Health History and Assessment Worksheet

Situation 8 – As Filipino Professional Nurses we must be


knowledgeable about the Code of Ethics for Filipino Nurse and
practice these by heart. The next questions pertain to this Code of
Ethics.

36. Which of the following is TRUE about the Code of Ethics of


Filipino Nurses?

A. The Philippine Nurses Association for being the accredited


professional organization was given the privilege to formulate a Code
of Ethics for Nurses which the Board of Nursing promulgated
B. Code for Nurses was first formulated in 1982 published in the
Proceedings of the Third Annual Convention of the PNA House of
Delegates
C. The present code utilized the Code of Good Governance for the
Professions in the Philippines
D. Certificates of Registration of registered nurses may be revoked or
suspended for violations of any provisions of the Code of Ethics.

37. Based on the Code of Ethics for Filipino Nurses, what is regarded
as the hallmark of nursing responsibility and accountability?

A. Human rights of clients, regardless of creed and gender


B. The privilege of being a registered professional nurses
C. Health, being a fundamental right of every individual
D. Accurate documentation of actions and outcomes

38. Which of the following nurses behavior is regarded as a violation


of the Code of Ethics of Filipino Nurses?

A. A nurse withholding harmful information to the family members of


a patient
B. A nurse declining commission sent by a doctor for her referral
C. A nurse endorsing a person running for congress.
D. Nurse Reviewers and/or nurse review center managers who pays
a considerable amount of cash for reviewees who would memorize
items from the licensure exams and submit these to them after the
examination.

39. A nurse should be cognizant that professional programs for


specialty certification by the Board of Nursing accredited through
the:

Professional Regulation Commission


Nursing Specialty Certification Council
Association of Deans of Philippine Colleges of Nursing
Philippine Nurse Association

40. Mr. Santos, R.N. works in a nursing home, and he knows that one
of his duties is to be an advocate for his patients. Mr. Santos knows a
primary duty of an advocate is to;

A. act as the patient’s legal representative


B. complete all nursing responsibilities on time
C. safeguard the well being of every patient
D. maintain the patient’s right to privacy

Situation 9 – Nurse Joanna works as an OB-Gyne Nurse and attends


to several HIGH-RISK PREGNANCIES: Particular women with
preexisting or Newly Acquired illness. The following conditions apply

41. Bernadette is a 22-year old woman. Which condition would make


her more prone than others to developing a Candida infection during
pregnancy?
A. Her husband plays golf 6 days a week
B. She was over 35 when she became pregnant
C. She usually drinks tomato juice for breakfast
D. She has developed gestational diabetes

42. Bernadette develops a deep vein thrombosis following an auto


accident and is prescribed heparin sub-Q. What should Joanna
educate her about in regard to this?

A. Some infants will be born with allergic symptoms to heparin


B. Her infant will be born with scattered petechiae on his trunk
C. Heparin can cause darkened skin in newborns
D. Heparin does not cross placenta and so does not affect a fetus

43. The cousin of Bernadette with sickle-cell anemia alerted Joanna


that she may need further instruction on prenatal care. What
statement signifies this fact?

A. I’ve stopped jogging so I don’t risk becoming dehydrated.


B. I take an iron pill every day to help grow new red blood cells
C. I am careful to drink at least eight glasses of fluid every day
D. I understand why folic acid is important for red cell formation

44. Bernadette routinely takes acetylsalicylic acid (aspirin) for


arthritis. Why should she limit or discontinue this toward the end of
pregnancy?

A. Aspirin can lead to deep vein thrombosis following birth


B. Newborns develop a red rash from salicylate toxicity
C. Newbors develop withdrawal headaches from salicylates
D. Salicylates can lead to increased maternal bleeding at childbirth

45. Bernadette received a laceration on her leg from her automobile


accident. Why are lacerations of lower extremities potentially more
serious in pregnant women than others?
A. Lacerations can provoke allergic responses because of
gonadothropic hormone
B. Increased bleeding can occur from uterine pressure on leg veins
C. A woman is less able to keep the laceration clean because of her
fatigue
D. Healing is limited during pregnancy, so these will not heal until
after birth.

Situation 10 – Still in your self-managed Child Health Nursing Clinic,


you encounter these cases pertaining to the CARE OF CHILDREN
WITH PULMONARY AFFECTIONS.

46. Josie brought her 3 months old child to your clinic because of
cough and colds. Which of the following is your primary action?

A. Give cotrimoxazole tablet or syrup


B. Assess the patient using the chart on management of children with
cough
C. Refer to the doctor
D. Teach the mother how to count her child’s breathing

47. In responding to the care concerns of children with sever


disease, referral to the hospital is of the essence especially if the
child manifests which of the following?

A. Wheezing
B. Stop feeding well
C. Fast breathing
D. Difficulty to awaken

48. Which of the following is the most important responsibility of a


nurse in the prevention of unnecessary deaths from pneumonia and
other severe diseases?

A. Giving antibiotics
B. Taking of the temperature of the sick child
C. Provision of Careful Assessment
D. Weighing of the sick child

49. You were able to identify factors that lead to respiratory


problems in the community where your health facility serve. Your
primary role therefore in order to reduce morbidity due to pneumonia
is to:

A. Teach mothers how to recognize early signs and symptoms of


pneumonia
B. Make home visits to sick children
C. Refer cases to hospitals
D. Seek assistance and mobilize the BHWs to have a meeting with
mothers

50. Which of the following is the principal focus of the CARI program
of the Department of Health?

A. Enhancement of health team capabilities


B. Teach mothers how to detect signs and where to refer
C. Mortality reduction through early detection
D. Teach other community health workers how to assess patients.

answer key np

note: guys, i'll post the rationales later this month na lang ha, super
hectic lang talaga my sched (^^,)

NURSING PRACTICE II – Community Health Nursing and Care of the


Mother and Child

SITUATIONAL

Situation 1 – Nurse Minette is an Independent Nurse Practitioner


following-up referred clients in their respective homes. Here she
handles a case of POSTPARTIAL MOTHER AND FAMILY focusing on
HOME CARE.

1. Nurse Minette needs to schedule a first home visit to OB client


Leah. When is a first home-care visit typically made?

Within 24 hours after discharge

2. Leah is developing constipation from being on bed rest. What


measures would you suggest she take to help prevent this?

Drink eight full glasses of fluid such as water daily

3. If you were Minette, which of the following actions would alert you
that a new mother is entering a postpartal taking-hold phase?

She urges the baby to stay awake so that she can breast-feed him or
her

4. At 6-week postpartum visit what should this postpartal mother’s


fundic height be?

No longer palpable on her abdomen

5. This postpartal mother wants to loose the weight she gained in


pregnancy, so she is reluctant to increase her caloric intake for
breast-feeding. By how much should a lactating mother increase her
caloric intake during the first 6 months after birth?

500 kcal/day

Situation 2 – As the CPE is applicable for all professional nurse, the


professional growth and development of Nurses with specialties shall
be addressed by a Specialty Certification Council. The following
questions apply to these special groups of nurses.
6. Which of the following serves as the legal basis and statute
authority for the Board of Nursing to promulgate measures to effect
the creation of a Specialty Certification Council and promulgate
professional development programs for this group of nurse-
professionals?

R.A. 7164
7. By force of law, therefore, the PRC-Board of Nursing released
Resolution No. 14 Series of 1999 entitled: “Adoption of a Nursing
Specialty Certification Program and Creation of Nursing Specialty
Certification Council.” This rule-making power is called:

Quasi-Legislative Power

8. Under the PRC-Board of Nursing Resolution promulgating the


adoption of a Nursing Specialty Certification Program and Council,
which two (2) of the following serves as the strongest for its
enforcement?

(a) Advances made in Science and Technology have provided the


climate for specialization in almost all aspects of human endeavor;
and
(b) As necessary consequence, there has emerged a new concept
known as globalization which seeks to remove barriers in trade,
industry and services imposed by the national laws of countries all
over the world; and
(c) Awareness of this development should impel the nursing sector
to prepare our people in the services sector to meet the above
challenge; and
(d) Current trends of specialization in nursing practice recognized by
the International Council of Nurses (ICN) of which the Philippines is
a member for the benefit of the Filipino in terms of deepening and
refining nursing practice and enhancing the quality of nursing care.

a & b are strong justifications


9. Which of the following IS NOT a correct statement as regards
Specialty Certification?

The Board of Nursing intended to create the Nursing Specialty


Certification Program as a means of perpetuating the creation of an
elite force of Filipino Nurse Professionals.

10. The NSCC was created for the purpose of implementing the
Nursing Specialty policy under the direct supervision and
stewardship of the Board of Nursing. Who shall comprise the NSCC?

A. A Chairperson who is the current President of the APO; a member


from the Academe; and the last member coming from the Regulatory
Board
B. The chairperson and members of the Regulatory Board ipso facto
acts as the CPE Council
C. A Chairperson, chosen from among the Regulatory Board
members; a Vice Chairperson appointed by the BON at-large; two
other members also chosen at-large; and one representing the
consumer group;
D. A Chairperson who is the President of the Association from the
Academe; a member from the Regulatory Board; and the last member
coming from the APO

No answer..maybe a bonus question…

Situation 3 –Nurse Anna is a new BSN graduate and has just passed
her Licensure Examination for Nurses in the Philippines. She has
likewise been hired as a new Community Health Nurse in one of the
Rural Health Units in their City, which of the following conditions
may be acceptable TRUTHS applied to Community Health Nursing
Practice.

11. Which of the following is the primary focus of community health


nursing practice?
Promotion of health

12. In community health nursing, which of the following is our unit of


service as nurses?

The Family

13. A very important part of the Community Health Nursing


Assessment Process includes:

the application of professional judgment in estimating importance of


facts to family and community

14. In community health nursing it is important to take into account


the family health data coupled with an equally important need to
perform ocular inspection of the area as activities which are powerful
elements of:

assessment

15. The initial step in PLANNING process in order to engage in any


nursing project or activities at the community level involves

goal-setting

Situation 4 – Please continue responding as a professional nurse in


these other health situations through the following questions.

16. Transmission of HIV from an infected individual to another


person occurs:

Most commonly as a result of sexual contact

17. The medical record of a client reveals a condition in which the


fetus cannot pass through the maternal pelvis. The nurse interprets
this as:
A. Fetopelvic disproportion

18. The nurse would anticipate a cesarean birth for a client who has
which infection present at the onset of labor?

Herpes-simplex virus

19. After a vaginal examination, the nurse determines that the


client’s fetus is in an occiput posterior position. The nurse would
anticipate that the client will have:

Intense back pain

20. The rationales for using a prostaglandin gel for a client prior to
the induction of labor is to:

Soften and efface the cervix

Situation 5 – Nurse Lorena is a Family Planning and Infertility Nurse


Specialist and currently attends to FAMILY PLANNING CLIENTS AND
INFERTILE COUPLES. The following conditions pertain to meeting the
nursing needs of this particular population group.

21. Dina, 17 years old, asks you how a tubal ligation prevents
pregnancy. Which would be the best answer?

Sperm can no longer reach the ova, because the fallopian tubes are
blocked

22. The Dators are a couple undergoing testing for infertility.


Infertility is said to exist when:

a couple has been trying to conceive for 1 year

23. Another client named Lilia is diagnosed as having endometriosis.


This condition interferes with fertility because:

endometrial implants can block the fallopian tubes

24. Lilia is scheduled to have a hysterosalphingogram. Which of the


following instructions would you give her regarding this procedure?

She may feel some cramping when the dye is inserted

25. Lilia’s cousin on the other hand, knowing nurse Lorena’s


specialization asks what artificial insemination by donor entails.
Which would be your best answer if you were Nurse Lorena?

Donor sperm are introduced vaginally into the uterus or cervix

Situation 6 – There are other important basic knowledge in the


performance of our task as Community Health Nurse in relation to
IMMUNIZATION, these include:

26. The correct temperature to store vaccines in a refrigerator is:

between 2 deg C and +8 deg C

27. Which of the following vaccines is not done by intramuscular


(IM) injection?

Measles vaccine

28. This vaccine content is derived from RNA recombinants.

Hepatitis B vaccines

29. This is the vaccine needed before a child reaches one (1) year in
order for him/her to qualify as a :fully immunized child”.

Measles
30. Which of the following dose of tetanus toxoid is given to the
mother to protect her infant from neonatal tetanus and likewise
provide 10 years protection for the mother?

Tetanus toxoid 4

Situation 7 – Records contain those comprehensive descriptions of


patient’s health conditions and needs and at the same serve as
evidences of every nurse’s accountability in the care giving process.
Nursing records normally differ from institution to institution
nonetheless they follow similar patterns of meeting needs for
specific types of information. The following pertains to
documentation/records management.

31. This special form is used when the patient is admitted to the unit.
The nurse completes the information in this record particularly
his/her basic personal data, current illness, previous health history,
health history of the family, emotional profile, environmental history
as well as physical assessment together with nursing diagnosis on
admission. What do you call this record?

Nursing Health History and Assessment Worksheet

32. These are sheets/forms which provide an efficient and time


saving way to record information that must be obtained repeatedly at
regular and/or short intervals of time. This does not replace the
progress notes; instead this record of information on vital signs,
intake and output, treatment, postoperative care, post partum care,
and diabetic regimen, etc. This is used whenever specific
measurements or observations are needed to be documented
repeatedly. What is this?

Graphic Flow Sheets

33. These records show all medications and treatment provided on a


repeated basis. What do you call this record?

Medicine and Treatment Record

34. This flip-over card is usually kept in a portable file at the Nurse’s
Station. It has 2-parts: the activity and treatment section and a
nursing care plan section. This carries information about basic
demographic data, primary medical diagnosis, current orders of the
physician to be carried out by the nurse, written nursing care plan,
nursing orders, scheduled tests and procedures, safety precautions
in patient care and factors related to daily living activities. This
record is used in the charge-of-shift reports or during the bedside
rounds or walking rounds. What record is this?

Nursing Kardex

35. Most nurses regard this conventional recording of the date, time,
and mode by which the patient leaves a healthcare unit but this
record includes importantly, directs of planning for discharge that
starts soon after the person is admitted to a healthcare institution. It
is accepted that collaboration or multidisciplinary involvement (of all
members of the health team) in discharge results in comprehensive
care. What do you call this?

Discharge Summary

Situation 8 – As Filipino Professional Nurses we must be


knowledgeable about the Code of Ethics for Filipino Nurse and
practice these by heart. The next questions pertain to this Code of
Ethics.

36. Which of the following is TRUE about the Code of Ethics of


Filipino Nurses?

The present code utilized the Code of Good Governance for the
Professions in the Philippines
37. Based on the Code of Ethics for Filipino Nurses, what is regarded
as the hallmark of nursing responsibility and accountability?

Accurate documentation of actions and outcomes

38. Which of the following nurses behavior is regarded as a violation


of the Code of Ethics of Filipino Nurses?

Nurse Reviewers and/or nurse review center managers who pays a


considerable amount of cash for reviewees who would memorize
items from the licensure exams and submit these to them after the
examination.

39. A nurse should be cognizant that professional programs for


specialty certification by the Board of Nursing accredited through
the:

Nursing Specialty Certification Council

40. Mr. Santos, R.N. works in a nursing home, and he knows that one
of his duties is to be an advocate for his patients. Mr. Santos knows a
primary duty of an advocate is to;

maintain the patient’s right to privacy

Situation 9 – Nurse Joanna works as an OB-Gyne Nurse and attends


to several HIGH-RISK PREGNANCIES: Particular women with
preexisting or Newly Acquired illness. The following conditions apply

41. Bernadette is a 22-year old woman. Which condition would make


her more prone than others to developing a Candida infection during
pregnancy?

She has developed gestational diabetes


42. Bernadette develops a deep vein thrombosis following an auto
accident and is prescribed heparin sub-Q. What should Joanna
educate her about in regard to this?

Heparin does not cross placenta and so does not affect a fetus

43. The cousin of Bernadette with sickle-cell anemia alerted Joanna


that she may need further instruction on prenatal care. What
statement signifies this fact?

I take an iron pill every day to help grow new red blood cells

44. Bernadette routinely takes acetylsalicylic acid (aspirin) for


arthritis. Why should she limit or discontinue this toward the end of
pregnancy?

Salicylates can lead to increased maternal bleeding at childbirth

45. Bernadette received a laceration on her leg from her automobile


accident. Why are lacerations of lower extremities potentially more
serious in pregnant women than others?

Increased bleeding can occur from uterine pressure on leg veins

Situation 10 – Still in your self-managed Child Health Nursing Clinic,


you encounter these cases pertaining to the CARE OF CHILDREN
WITH PULMONARY AFFECTIONS.

46. Josie brought her 3 months old child to your clinic because of
cough and colds. Which of the following is your primary action?

Assess the patient using the chart on management of children with


cough
A. Refer to the doctor
B. Teach the mother how to count her child’s breathing
47. In responding to the care concerns of children with sever
disease, referral to the hospital is of the essence especially if the
child manifests which of the following?

Difficulty to awaken

48. Which of the following is the most important responsibility of a


nurse in the prevention of unnecessary deaths from pneumonia and
other severe diseases?

Provision of Careful Assessment

49. You were able to identify factors that lead to respiratory


problems in the community where your health facility serve. Your
primary role therefore in order to reduce morbidity due to pneumonia
is to:

Teach mothers how to recognize early signs and symptoms of


pneumonia

50. Which of the following is the principal focus of the CARI program
of the Department of Health?

Mortality reduction through early detection

Wednesday, January 23, 2008

StuffedNurse: NP3 practice exam

NURSING PRACTICE III – Care of Clients with Physiologic and Psychosocial Alterations (Part
A)
SITUATIONAL

Situation 1 – Concerted work efforts among members of the surgical team is essential to the
success of the surgical procedure.

1. The sterile nurse or sterile personnel touch only sterile supplies and instruments. When there is
a need for sterile supply which is not in the sterile field, who hands out these items by opening its
outer cover?

A. Circulating Nurse
B. Anaesthesiologist
C. Surgeon
D. Nursing Aide

2. The OR team performs distinct roles for one surgical procedure to be accomplished within a
prescribed time frame and deliver a standard patient outcome. While the surgeon performs the
surgical procedure, who monitors the status of the client like urine output, blood loss?

A. Scrub Nurse
B. Surgeon
C. Anaesthesiologist
D. Circulating Nurse

3. Surgery schedules are communicated to the OR usually a day prior to the procedure by the
nurse of the floor or ward where the patient is confined. For orthopedic cases, what department is
usually informed to be present in the OR?

A. Rehabilitation department
B. Laboratory department
C. Maintenance department
D. Radiology department

4. Minimally invasive surgery is very much into technology. Aside from the usual surgical team,
who else has to be present when a client undergoes laparoscopic surgery?

A. Information technician
B. Biomedical technician
C. Electrician
D. Laboratory technician

5. In massive blood loss, prompt replacement of compatible blood is crucial. What department
needs to be alerted to coordinate closely with the patient’s family for immediate blood component
therapy?

A. Security Division
B. Chaiplaincy
C. Social Service Section
D. Pathology department

Situation 2 – You are assigned in the Orthopedic Ward where clients are complaining of pain in
varying degrees upon movement of body parts.

6. Troy is a one day post open reduction and internal fixation (ORIF) of the left hip and is in pain.
Which of the following observation would prompt you to call the doctor?

A. Dressing is intact but partially soiled


B. Left foot is cold to touch and pedal pulse is absent
C. Left leg in limited functional anatomic position
D. BP 114/78, pulse of 82 beats/minute

7. There is an order of Demerol 50 mg I.M. now and every 6 hours p r n. You injected Demerol at
5 pm. The next dose of Demerol 50 mg I.M. is given:

A. When the client asks for the next dose


B. When the patient is in severe pain
C. At 11 pm
D. At 12 pm

8. You continuously evaluate the client’s adaptation to pain. Which of the following behaviors
indicate appropriate adaptation?

A. The client reports pain reduction and decreased activity


B. The client denies existence of pain
C. The client can distract himself during pain episodes
D. The client reports independence from watchers

9. Pain in ortho cases may not be mainly due to the surgery. There might be other factors such as
cultural or psychological that influence pain. How can you alter these factors as the nurse?

A. Explain all the possible interventions that may cause the client to worry
B. Establish trusting relationship by giving his medication on time
C. Stay with the client during pain episodes
D. Promote client’s sense of control and participation in control by listening to his concerns

10. In some hip surgeries, an epidural catheter for Fentanyl epidural analgesia is given. What is
your nursing priority care in such a case?

A. Instruct client to observe strict bed rest


B. Check for epidural catheter drainage
C. Administer analgesia through epidural catheter as prescribed
D. Assess respiratory rate carefully
Situation 3 – Records are vital tools in any institution and should be properly maintained for
specific use and time.

11. The patient’s medical record can work as a double edged sword. When can the medical record
become the doctor’s/nurse’s worst enemy?

A. When the record is voluminous


B. When a medical record is subpoenaed in court
C. When it is missing
D. When the medical record is inaccurate, incomplete, and inadequate

12. Disposal of medical records in government hospitals/institutions must be done in close


coordination with what agency?

A. Department of Interior and Local Government (DILG)


B. Metro Manila Development Authority (MMDA)
C. Records Management Archives Office (RMAO)
D. Department of Health (DOH)

13. In the hospital, when you need the medical record of a discharged patient for research you
will request permission through:

A. Doctor in charge
B. The hospital director
C. The nursing service
D. Medical records section

14. You readmitted a client who was in another department a month ago. Since you will need the
previous chart, from whom do you request the old chart?

A. Central supply section


B. Previous doctor’s clinic
C. Department where the patient was previously admitted
D. Medical records section

15. Records Management and Archives Office of the DOH is responsible for implementing its
policies on record disposal. You know that your institution is covered by this policy if:

A. Your hospital is considered tertiary


B. Your hospital is in Metro Manila
C. It obtained permit to operate from DOH
D. Your hospital is PhilHealth accredited

Situation 4 – In the OR, there are safety protocols that should be followed. The OR nurse should
be well versed with all these to safeguard the safety and quality of patient delivery outcome.
16. Which of the following should be given highest priority when receiving patient in the OR?

A. Assess level of consciousness


B. Verify patient identification and informed consent
C. Assess vital signs
D. Check for jewelry, gown, manicure, and dentures

17. Surgeries like I and D (incision and drainage) and debridement are relatively short procedures
but considered ‘dirty cases’. When are these procedures best scheduled?

A. Last case
B. In between cases
C. According to availability of anaesthesiologist
D. According to the surgeon’s preference

18. OR nurses should be aware that maintaining the client’s safety is the overall goal of nursing
care during the intraoperative phase. As the circulating nurse, you make certain that throughout
the procedure…

A. the surgeon greets his client before induction of anesthesia


B. the surgeon and anesthesiologist are in tandem
C. strap made of strong non-abrasive materials are fastened securely around the joints of the
knees and ankles and around the 2 hands around an arm board.
D. Client is monitored throughout the surgery by the assistant anesthesiologist

19. Another nursing check that should not be missed before the induction of general anesthesia is:

A. check for presence underwear


B. check for presence dentures
C. check patient’s ID
D. check baseline vital signs

20. Some lifetime habits and hobbies affect postoperative respiratory function. If your client
smokes 3 packs of cigarettes a day for the past 10 years, you will anticipate increased risk for:

A. perioperative anxiety and stress


B. delayed coagulation time
C. delayed wound healing
D. postoperative respiratory function

Situation 5 – Nurses hold a variety of roles when providing care to a perioperative patient.

21. Which of the following role would be the responsibility of the scrub nurse?

A. Assess the readiness of the client prior to surgery


B. Ensure that the airway is adequate
C. Account for the number of sponges, needles, supplies, used during the surgical procedure.
D. Evaluate the type of anesthesia appropriate for the surgical client

22. As a perioperative nurse, how can you best meet the safety need of the client after
administering preoperative narcotic?

A. Put side rails up and ask the client not to get out of bed
B. Send the client to OR with the family
C. Allow client to get up to go to the comfort room
D. Obtain consent form

23. It is the responsibility of the pre-op nurse to do skin prep for patients undergoing surgery. If
hair at the operative site is not shaved, what should be done to make suturing easy and lessen
chance of incision infection?

A. Draped
B. Pulled
C. Clipped
D. Shampooed

24. It is also the nurse’s function to determine when infection is developing in the surgical
incision. The perioperative nurse should observe for what signs of impending infection?

A. Localized heat and redness


B. Serosanguinous exudates and skin blanching
C. Separation of the incision
D. Blood clots and scar tissue are visible

25. Which of the following nursing interventions is done when examining the incision wound and
changing the dressing?

A. Observe the dressing and type and odor of drainage if any


B. Get patient’s consent
C. Wash hands
D. Request the client to expose the incision wound

Situation 6 – Carlo, 16 years old, comes to the ER with acute asthmatic attack. RR is 46/min and
he appears to be in acute respiratory distress.

26. Which of the following nursing actions should be initiated first?

A. Promote emotional support


B. Administer oxygen at 6L/min
C. Suction the client every 30 min
D. Administer bronchodilator by nebulizer
27. Aminophylline was ordered for acute asthmatic attack. The mother asked the nurse, what is its
indication, the nurse will say:

A. Relax smooth muscles of the bronchial airway


B. Promote expectoration
C. Prevent thickening of secretions
D. Suppress cough

28. You will give health instructions to Carlo, a case of bronchial asthma. The health instruction
will include the following, EXCEPT:

A. Avoid emotional stress and extreme temperature


B. Avoid pollution like smoking
C. Avoid pollens, dust, seafood
D. Practice respiratory isolation

29. The asthmatic client asked you what breathing techniques he can best practice when asthmatic
attack starts. What will be the best position?

A. Sit in high-Fowler’s position with extended legs


B. Sit-up with shoulders back
C. Push on abdomen during exhalation
D. Lean forward 30-40 degrees with each exhalation

30. As a nurse, you are always alerted to monitor status asthmaticus who will likely and initially
manifest symptoms of:

A. metabolic alkalosis
B. respiratory acidosis
C. respiratory alkalosis
D. metabolic acidosis

Situation 7 – Joint Commission on Accreditation of Hospital Organization (JCAHO) patient


safety goals and requirements include the care and efficient use of technology in the OR and
elsewhere in the healthcare facility.

31. As the head nurse in the OR, how can you improve the effectiveness of clinical alarm
systems?

A. Limit suppliers to a few so that quality is maintained


B. Implement a regular inventory of supplies and equipment
C. Adherence to manufacturer’s recommendation
D. Implement a regular maintenance and testing of alarm systems

32. Overdosage of medication or anesthetic can happen even with the aid of technology like
infusion pumps, sphygmomanometer and similar devices/machines. As a staff, how can you
improve the safety of using infusion pumps?

A. Check the functionality of the pump before use


B. Select your brand of infusion pump like you do with your cellphone
C. Allow the technician to set the infusion pump before use
D. Verify the flow rate against your computation

33. JCAHOs universal protocol for surgical and invasive procedures to prevent wrong site, wrong
person, and wrong procedure/surgery includes the following, EXCEPT:

A. Mark the operative site if possible


B. Conduct pre-procedure verification process
C. Take a video of the entire intra-operative procedure
D. Conduct ‘time out’ immediately before starting the procedure

34. You identified a potential risk of pre-and postoperative clients. To reduce the risk of patient
harm resulting from fall, you can implement the following, EXCEPT:

A. Assess potential risk of fall associated with the patient’s medication regimen
B. Take action to address any identified risks through Incident Report (IR)
C. Allow client to walk with relative to the OR
D. Assess and periodically reassess individual client’s risk for falling

35. As a nurse, you know you can improve on accuracy of patient’s identification by 2 patient
identifiers, EXCEPT:

A. identify the client by his/her wrist tag and verify with family members
B. identify client by his/her wrist tag and call his/her by name
C. call the client by his/her case and bed number
D. call the patient by his/her name and bed number

Situation 8 – Team efforts is best demonstrated in the OR.

36. If you are the nurse in charge for scheduling surgical cases, what important information do
you need to ask the surgeon?

A. Who is your internist


B. Who is your assistant and anesthesiologist, and what is your preferred time and type of
surgery?
C. Who are your anesthesiologist, internist, and assistant
D. Who is your anesthesiologist

37. In the OR, the nursing tandem for every surgery is:

A. Instrument technician and circulating nurse


B. Nurse anesthetist, nurse assistant, and instrument technician
C. Scrub nurse and nurse anesthetist
D. Scrub and circulating nurses

38. While team effort is needed in the OR for efficient and quality patient care delivery, we
should limit the number of people in the room for infection control. Who comprise this team?

A. Surgeon, anesthesiologist, scrub nurse, radiologist, orderly


B. Surgeon, assistants, scrub nurse, circulating nurse, anesthesiologist
C. Surgeon, assistant surgeon, anesthesiologist, scrub nurse, pathologist
D. Surgeon, assistant surgeon, anesthesiologist, intern, scrub nurse

39. When surgery is on-going, who coordinates the activities outside, including the family?

A. Orderly/clerk
B. Nurse Supervisor
C. Circulating Nurse
D. Anesthesiologist

40. The breakdown in teamwork is often times a failure in:

A. Electricity
B. Inadequate supply
C. Leg work
D. Communication

Situation 9 – Colostomy is a surgically created anus. It can be temporary or permanent,


depending on the disease condition.

41. Skin care around the stoma is critical. Which of the following is not indicated as a skin care
barriers?

A. Apply liberal amount of mineral oil to the area


B. Use karaya paste and rings around the stoma
C. Clean the area daily with soap and water before applying bag
D. Apply talcum powder twice a day

42. What health instruction will enhance regulation of a colostomy (defecation) of clients?

A. Irrigate after lunch everyday


B. Eat fruits and vegetables in all three meals
C. Eat balanced meals at regular intervals
D. Restrict exercise to walking only

43. After ileostomy, which of the following condition is NOT expected?

A. Increased weight
B. Irritation of skin around the stoma
C. Liquid stool
D. Establishment of regular bowel movement

44. The following are appropriate nursing interventions during colostomy irrigation, EXCEPT:

A. Increase the irrigating solution flow rate when abdominal cramps is felt
B. Insert 2-4 inches of an adequately lubricated catheter to the stoma
C. Position client in semi-Fowler
D. Hang the solution 18 inches above the stoma

45. What sensation is used as a gauge so that patients with ileostomy can determine how often
their pouch should be drained?

A. Sensation of taste
B. Sensation of pressure
C. Sensation of smell
D. Urge to defecate

Situation 10 – As a beginner in research, you are aware that sampling is an essential elements of
the research process.

46. What does a sample group represent?

A. Control group
B. Study subjects
C. General population
D. Universe

47. What is the most important characteristic of a sample?

A. Randomization
B. Appropriate location
C. Appropriate number
D. Representativeness

48. Random sampling ensures that each subject has:

A. Been selected systematically


B. An equal chance of selection
C. Been selected based on set criteria
D. Characteristics that match other samples

49. Which of the following methods allows the use of any group of research subject?

A. Purposive
B. Convenience
C. Snow-ball
D. Quota

50. You decided to include 5 barangays in your municipality and chose a sampling method that
would get representative samples from each barangay. What should be the appropriate method
ofor you to use in this care?

A. Cluster sampling
B. Random sampling
C. Startified ampling
D. Systematic sampling

answer key np3

note: later na po ung rationales! (^^,)

NURSING PRACTICE III – Care of Clients with Physiologic and Psychosocial Alterations (Part
A)

SITUATIONAL

Situation 1 – Concerted work efforts among members of the surgical team is essential to the
success of the surgical procedure.

1. The sterile nurse or sterile personnel touch only sterile supplies and instruments. When there is
a need for sterile supply which is not in the sterile field, who hands out these items by opening its
outer cover?

Circulating Nurse

2. The OR team performs distinct roles for one surgical procedure to be accomplished within a
prescribed time frame and deliver a standard patient outcome. While the surgeon performs the
surgical procedure, who monitors the status of the client like urine output, blood loss?

Anaesthesiologist

3. Surgery schedules are communicated to the OR usually a day prior to the procedure by the
nurse of the floor or ward where the patient is confined. For orthopedic cases, what department is
usually informed to be present in the OR?

Radiology department

4. Minimally invasive surgery is very much into technology. Aside from the usual surgical team,
who else has to be present when a client undergoes laparoscopic surgery?
Biomedical technician

5. In massive blood loss, prompt replacement of compatible blood is crucial. What department
needs to be alerted to coordinate closely with the patient’s family for immediate blood component
therapy?

Social Service Section

Situation 2 – You are assigned in the Orthopedic Ward where clients are complaining of pain in
varying degrees upon movement of body parts.

6. Troy is a one day post open reduction and internal fixation (ORIF) of the left hip and is in pain.
Which of the following observation would prompt you to call the doctor?

Left foot is cold to touch and pedal pulse is absent

7. There is an order of Demerol 50 mg I.M. now and every 6 hours p r n. You injected Demerol at
5 pm. The next dose of Demerol 50 mg I.M. is given:

At 11 pm

8. You continuously evaluate the client’s adaptation to pain. Which of the following behaviors
indicate appropriate adaptation?

The client can distract himself during pain episodes

9. Pain in ortho cases may not be mainly due to the surgery. There might be other factors such as
cultural or psychological that influence pain. How can you alter these factors as the nurse?

Promote client’s sense of control and participation in control by listening to his concerns

10. In some hip surgeries, an epidural catheter for Fentanyl epidural analgesia is given. What is
your nursing priority care in such a case?

Assess respiratory rate carefully

Situation 3 – Records are vital tools in any institution and should be properly maintained for
specific use and time.

11. The patient’s medical record can work as a double edged sword. When can the medical record
become the doctor’s/nurse’s worst enemy?

When the medical record is inaccurate, incomplete, and inadequate


12. Disposal of medical records in government hospitals/institutions must be done in close
coordination with what agency?

Department of Health (DOH)

13. In the hospital, when you need the medical record of a discharged patient for research you
will request permission through:

Medical records section

14. You readmitted a client who was in another department a month ago. Since you will need the
previous chart, from whom do you request the old chart?

Medical records section

15. Records Management and Archives Office of the DOH is responsible for implementing its
policies on record disposal. You know that your institution is covered by this policy if:

It obtained permit to operate from DOH

Situation 4 – In the OR, there are safety protocols that should be followed. The OR nurse should
be well versed with all these to safeguard the safety and quality of patient delivery outcome.

16. Which of the following should be given highest priority when receiving patient in the OR?

Verify patient identification and informed consent

17. Surgeries like I and D (incision and drainage) and debridement are relatively short procedures
but considered ‘dirty cases’. When are these procedures best scheduled?

Last case

18. OR nurses should be aware that maintaining the client’s safety is the overall goal of nursing
care during the intraoperative phase. As the circulating nurse, you make certain that throughout
the procedure…

strap made of strong non-abrasive materials are fastened securely around the joints of the knees
and ankles and around the 2 hands around an arm board.

19. Another nursing check that should not be missed before the induction of general anesthesia is:

check baseline vital signs

20. Some lifetime habits and hobbies affect postoperative respiratory function. If your client
smokes 3 packs of cigarettes a day for the past 10 years, you will anticipate increased risk for:
delayed wound healing

Situation 5 – Nurses hold a variety of roles when providing care to a perioperative patient.

21. Which of the following role would be the responsibility of the scrub nurse?

Account for the number of sponges, needles, supplies, used during the surgical procedure.

22. As a perioperative nurse, how can you best meet the safety need of the client after
administering preoperative narcotic?

Put side rails up and ask the client not to get out of bed

23. It is the responsibility of the pre-op nurse to do skin prep for patients undergoing surgery. If
hair at the operative site is not shaved, what should be done to make suturing easy and lessen
chance of incision infection?

Clipped

24. It is also the nurse’s function to determine when infection is developing in the surgical
incision. The perioperative nurse should observe for what signs of impending infection?

Localized heat and redness

25. Which of the following nursing interventions is done when examining the incision wound and
changing the dressing?

Observe the dressing and type and odor of drainage if any

Situation 6 – Carlo, 16 years old, comes to the ER with acute asthmatic attack. RR is 46/min and
he appears to be in acute respiratory distress.

26. Which of the following nursing actions should be initiated first?

Administer bronchodilator by nebulizer

27. Aminophylline was ordered for acute asthmatic attack. The mother asked the nurse, what is its
indication, the nurse will say:

Relax smooth muscles of the bronchial airway

28. You will give health instructions to Carlo, a case of bronchial asthma. The health instruction
will include the following, EXCEPT:

Practice respiratory isolation


29. The asthmatic client asked you what breathing techniques he can best practice when asthmatic
attack starts. What will be the best position?

Lean forward 30-40 degrees with each exhalation

30. As a nurse, you are always alerted to monitor status asthmaticus who will likely and initially
manifest symptoms of:

respiratory acidosis

Situation 7 – Joint Commission on Accreditation of Hospital Organization (JCAHO) patient


safety goals and requirements include the care and efficient use of technology in the OR and
elsewhere in the healthcare facility.

31. As the head nurse in the OR, how can you improve the effectiveness of clinical alarm
systems?

Implement a regular maintenance and testing of alarm systems

32. Overdosage of medication or anesthetic can happen even with the aid of technology like
infusion pumps, sphygmomanometer and similar devices/machines. As a staff, how can you
improve the safety of using infusion pumps?

Check the functionality of the pump before use

33. JCAHOs universal protocol for surgical and invasive procedures to prevent wrong site, wrong
person, and wrong procedure/surgery includes the following, EXCEPT:

Take a video of the entire intra-operative procedure

34. You identified a potential risk of pre-and postoperative clients. To reduce the risk of patient
harm resulting from fall, you can implement the following, EXCEPT:

Allow client to walk with relative to the OR

35. As a nurse, you know you can improve on accuracy of patient’s identification by 2 patient
identifiers, EXCEPT:

call the client by his/her case and bed number

Situation 8 – Team efforts is best demonstrated in the OR.

36. If you are the nurse in charge for scheduling surgical cases, what important information do
you need to ask the surgeon?
Who is your assistant and anesthesiologist, and what is your preferred time and type of surgery?

37. In the OR, the nursing tandem for every surgery is:

Scrub and circulating nurses

38. While team effort is needed in the OR for efficient and quality patient care delivery, we
should limit the number of people in the room for infection control. Who comprise this team?

Surgeon, assistants, scrub nurse, circulating nurse, anesthesiologist

39. When surgery is on-going, who coordinates the activities outside, including the family?

Circulating Nurse

40. The breakdown in teamwork is often times a failure in:

Communication

Situation 9 – Colostomy is a surgically created anus. It can be temporary or permanent,


depending on the disease condition.

41. Skin care around the stoma is critical. Which of the following is not indicated as a skin care
barriers?

Apply liberal amount of mineral oil to the area

42. What health instruction will enhance regulation of a colostomy (defecation) of clients?

Eat balanced meals at regular intervals

43. After ileostomy, which of the following condition is NOT expected?

Increased weight

44. The following are appropriate nursing interventions during colostomy irrigation, EXCEPT:

Increase the irrigating solution flow rate when abdominal cramps is felt

45. What sensation is used as a gauge so that patients with ileostomy can determine how often
their pouch should be drained?

Sensation of pressure

Situation 10 – As a beginner in research, you are aware that sampling is an essential elements of
the research process.

46. What does a sample group represent?

General population

47. What is the most important characteristic of a sample?

Representativeness

48. Random sampling ensures that each subject has:

An equal chance of selection

49. Which of the following methods allows the use of any group of research subject?

Convenience

50. You decided to include 5 barangays in your municipality and chose a sampling method that
would get representative samples from each barangay. What should be the appropriate method
ofor you to use in this care?

Cluster sampling
Wednesday, January 23, 2008

StuffedNurse: NP4 practice exam

NURSING PRACTICE IV – Care of Clients with Physiologic and


Psychosocial Alterations (Part B)

MULTIPLE CHOICE
Situation 1 – Because of the serious consequences of severe burns, management requires a
multidisciplinary approach. You have important responsibilities as a nurse.

1. While Sergio was lighting a barbecue grill with a lighter fluid, his shirt burns into flames. The
most effective way to extinguish the flames with as little further damage as possible is to:

A. log roll on the grass/ground


B. slap the flames with his hands
C. remove the burning clothes
D. pour cold liquid over the flames

2. Once the flames are extinguished, it is most important to:

A. cover Sergio with a warm blanket


B. give him sips of water
C. calculate the extent of his burns
D. assess the Sergio’s breathing

3. Sergio is brought to Emergency Room after the barbecue grill accident. Based on the
assessment of the physician, Sergio sustained superficial partial thickness burns on his trunk,
right upper extremities and right lower extremities. His wife asks what that means? Your most
accurate response would be:

A. Structures beneath the skin are damage


B. Dermis is partially damaged
C. Epidermis and dermis are both damaged
D. Epidermis is damaged

4. During the first 24 hours after the thermal injury, you should asses Sergio for:

A. hypokalemia and hypernatremia


B. hypokalemia and hyponatremia
C. hyperkalemia and hyponatremia
D. hyperkalemia and hypernatremia

5. Teddy, who sustained deep partial thickness and full thickness burns of the face, whole anterior
chest and both upper extremities two days ago begins to exhibit extreme restlessness. You
recognize that this most likely indicates that Teddy is developing:

A. Cerebral hypoxia C. metabolic acidosis


B. Hypervolemia D. Renal failure
Situation 2 – You are now working as a staff nurse in a general hospital. You have to be prepared
to handle situations with ethico-legal and moral implifications.

6. You are in night duty in surgical ward. One of your patients Martin is a prisoner who sustained
an abdominal gunshot wound. He is being guarded by policeman from the local police unit.
During your rounds you heard a commotion. You saw the policeman trying to hit Martin. You
asked why he was trying to hit Martin. He denied the matter. Which among the following
activities will you do first?

A. Write an accident report


B. Call security officer and report the incident
C. Call your nurse supervisor and report the incident
D. Call the physician on duty

7. You are on morning duty in the medical ward. You have 10 patients assigned to you. During
your endorsement rounds, you found out that one of your patients was not in bed. The patient
next to him informed you that he went home without notifying the nurses. Which among the
following will you do first?

A. Make an incident report


B. Call security to report the incident
C. Wait for 2 hours before reporting
D. Report the incident to your supervisor

8. You are on duty in the medical ward. You were asked to check the narcotics cabinet. You found
out that what is on record does not tally with the drugs used. Which among the following will you
do first?

A. Write an incident report and refer the matter to the nursing director
B. Keep your findings to yourself
C. Report the matter to your supervisor
D. Find out from the endorsement any patient who might have been given narcotics

9. You are on duty in the medical ward. The mother of your patient who is also a nurse, came
running to the nurses station and informed you that Fiolo went into cardiopulmonary arrest.

A. Start basic life support measures


B. Call for the Code
C. Bring the crash cart to the room
D. Go to see Fiolo and assess for airway patency and breathing problems
10. You are admitting Jorge to the ward and you found out that he is positive for HIV. Which
among the following will you do first?

A. Take note of it and plan to endorse this to next shift


B. Keep this matter to yourself
C. Write an incident report
D. Report the matter to your head nurse

Situation 3 - Colorectal cancer can affect old and younger people. Surgical procedures and other
modes of treatment are done to ensure quality of life. You are assigned in the cancer institute to
care of patients with this type of cancer.

11. Larry, 55 years old, who is suspected of having colorectal cancer, is admitted to the CI. After
taking the history and vital signs the physician does which test as a screening test for colorectal
cancer.

A. Barium enema
B. Carcinoembryonic antigen
C. Annual digital rectal examination
D. Proctosigmoidoscopy

12. To confirm his impression of colorectal cancer, Larry will require which diagnostic study?

A. Carcinoembryonic antigen
B. Proctosigmoidoscopy
C. Stool hematologic test
D. Abdominal computed tomography (CT) test

13. The following are risk factors for colorectal cancer, EXCEPT:

A. Inflammatory bowels
B. High fat, high fiber diet
C. Smoking
D. Genetic factors-familial adenomatous polyposis

14. Symptoms associated with cancer of the colon include:

A. constipation, ascites and mucus in the stool


B. diarrhea, heart burn and eructation
C. blood in the stools, anemia, and “pencil shaped” stools
D. anorexia, hematemesis, and increased peristalasis

15. Several days prior to bowel surgery, Larry may be given sulfasuxidine and neomycin
primarily to:
A. promote rest of the bowel by minimizing peristalsis
B. reduce the bacterial content of the colon
C. empty the bowel of solid waste
D. soften the stool by retaining water in the colon

Situation 4 – ENTEROSTOMAL THERAPY is now considered a specialty in nursing. You are


participating in the OSTOMY CARE CLASS.

16. You plan to teach Fermin how to irrigate the colostomy when:

A. The perineal wound heals And Fermin can sit comfortably on the commode
B. Fermin can lie on the side comfortably, about the 3rd postoperative day
C. The abdominal incision is closed and contamination is no longer a danger
D. The stools starts to become formed, around the 7th postoperative day

17. When preparing to teach Fermin how to irrigate colostomy, you should plan to do the
procedure:

A. When Fermin would have normal bowel movement


B. At least 2 hours before visiting hours
C. Prior to breakfast and morning care
D. After Fermin accepts alteration in body image

18. When observing a return demonstration of a colostomy irrigation, you know that more
teaching is required if Fermin:

A. Lubricates the tip of the catheter prior to inserting into the stoma
B. Hangs the irrigating bag on the bathroom door cloth hook during fluid insertion
C. Discontinues the insertion of fluid after only 500 ml of fluid has been instilled
D. Clamps of the flow of fluid when felling uncomfortable

19. You are aware that teaching about colostomy care is understood when Fermin states, “I will
contact my physician and report:

A. If I have any difficulty inserting the irrigating tub into the stoma.”
B. If I noticed a loss of sensation to touch in the stoma tissue.”
C. The expulsion of flatus while the irrigating fluid is running out.”
D. When mucus is passed from the stoma between the irrigations.”

20. You would know after teaching Fermin that dietary instruction for him is effective when he
states, “It is important that I eat:

A. Soft food that are easily digested and absorbed by my large intestines.”
B. Bland food so that my intestines do not become irritated.”
C. Food low in fiber so that there is less stool.”
D. Everything that I ate before the operation, while avoiding foods that cause gas.”

Situation 5 – Ensuring safety is one of your most important responsibilities. You will need to
provide instructions and information to your clients to prevent complications.

21. Randy has chest tubes attached to a pleural drainage system. When caring for him you should:

A. empty the drainage system at the end of the shift


B. clamp the chest tube when suctioning
C. palpate the surrounding areas for crepitus
D. change the dressing daily using aseptic techniques

22. Fanny, came in from PACU after pelvic surgery. As Fanny’s nurse you know that the sign that
would be indicative of a developing thrombophlebitis would be:

A. a tender, painful area on the leg


B. a pitting edema of the ankle
C. a reddened area at the ankle
D. pruritus on the calf and ankle

23. To prevent recurrent attacks on Terry who has acute glumerulonephritis, you should instruct
her to:

A. seek early treatment for respiratory infections


B. take showers instead of tub bath
C. continue to take the same restrictions on fluid intake
D. avoid situations that involve physical activity

24. Herbert had a laryngectomy and he is now for discharge. He verbalized his concern regarding
his laryngectomy tube being dislodged. What should you teach him first?

A. Recognize that prompt closure of the tracheal opening may occur


B. Keep calm because there is no immediate emergency
C. Reinsert another tubing immediately
D. Notify the physician at once

25. When caring for Larry after an exploratory chest surgery and pneumonectomy, your priority
would be to maintain:

A. supplementary oxygen
B. ventilation exchange
C. chest tube drainage
D. blood replacement

Situation 6 – Infection can cause debilitating consequences when host resistance is compromised
and virulence of microorganisms and environmental factors are favorable. Infection control is one
important responsibility of the nurse to ensure quality of care.

26. Honrad, who has been complaining of anorexia and feeling tired, develops jaundice, after a
workup he is diagnosed of having Hepatitis A. his wife asks you about gamma globulin for
herself and her household help. Your most appropriate response would be:

A. “Don’t worry your husband’s type of hepatitis is no longer communicable”


B. “Gamma globulin provides passive immunity for hepatitis B”
C. “You should contact your physician immediately about getting gammaglobulin.”
D. “A vaccine has been developed for this type of hepatitis”

27. Voltaire develops a nosocomial respiratory tract infection. He ask you what that means? Your
best response would be:

A. “You acquired the infection after you have been admitted to the hospital.”
B. “This is a highly contagious infection requiring complete isolation.”
C. “The infection you had prior to hospitalization flared up.”
D. “As a result of medical treatment, you have acquired a secondary infection.’

28. As a nurse you know that one of the complications that you have to watch out for when caring
for Omar who is receiving total parenteral nutrition is:

A. stomatitis
B. hepatitis
C. dysrhythmia
D. infection

29. A solution used to treat Pseudomonas wound infection is:

A. Dakin’s solution
B. Half-strength hydrogen peroxide
C. Acetic acid
D. Betadine

30. Which of the following is the most reliable in diagnosing a wound infection?

A. Culture and sensitivity


B. Purulent drainage from a wound
C. WBC count of 20,000/μL
D. Gram stain testing
Situation 7 – As a nurse you need to anticipate the occurrence of complications of stroke so that
life threatening situations can be prevented.

31. Wendy is admitted to the hospital with signs and symptoms of stroke. Her Glasgow Coma
Scale is 6 on admission. A central venous catheter was inserted an I.V. infusion was started. As a
nurse assigned to Wendy what will be your priority goal?

A. Prevent skin breakdown


B. Preserve muscle function
C. Promote urinary elimination
D. Maintain a patent airway

32. Knowing that for a comatose patient hearing is the last sense to be lost, as Judy’s nurse, what
should you do?

A. Tell her family that probably she can’t hear them


B. Talk loudly so that Wendy can hear you
C. Tell her family who are in the room not to talk
D. Speak softly then hold her hands gently

33. Which among the following interventions should you consider as the highest priority when
caring for June who has hemiparesis secondary to stroke?

A. Place June on an upright lateral position


B. Perform range of motion exercises
C. Apply antiembolic stockings
D. Use hand rolls or pillows for support

34. Ivy, age 40, was admitted to the hospital with a severe headache, stiff neck and photophobia.
She was diagnosed with a subarachnoid hemorrhage secondary to ruptured aneurysm. While
waiting for surgery, you can provide a therapeutic environment by doing which of the following?

A. honoring her request for a television


B. placing her bed near the window
C. dimming the light in her room
D. allowing the family unrestricted visiting privileges

35. When performing a neurologic assessment on Walter, you find that his pupils are fixed and
dilated. This indicated that he:

A. probably has meningitis


B. is going to be blind because of trauma
C. is permanently paralyzed
D. has received a significant brain injury

Situation 8 – With the improvement in life expectancies and the emphasis in the quality of life it
is important to provide quality care to our older patients. There are frequently encountered
situations and issues relevant to the older patients.

36. Hypoxia may occur in the older patients because of which of the following physiologic
changes associated with aging.

A. Ineffective airway clearance


B. Decreased alveolar surfaced area
C. Decreased anterior-posterior chest diameter
D. Hyperventilation

37. The older patient is at higher risk for incontinence because of:

A. dilated urethra
B. increased glomerular filtration rate
C. diuretic use
D. decreased bladder capacity

38. Merle, age 86, is complaining of dizziness when she stands up. This may indicate:

A. dementia
B. a visual problem
C. functional decline
D. drug toxicity

39. Cardiac ischemia in an older patient usually produces:

A. ST-T wave changes


B. Very high creatinine kinase level
C. Chest pain radiating to the left arm
D. Acute confusion

40. The most dependable sign of infection in the older patient is:

A. change in mental status


B. fever
C. pain
D. decreased breath sounds with crackles
Situation 9 – A “disaster” is a large-scale emergency—even a small emergency left unmanaged
may turn into a disaster. Disaster preparedness is crucial and is everybody’s business. There are
agencies that are in charge of ensuring prompt response. Comprehensive Emergency
Management (CEM) is an integrated approach to the management of emergency programs and
activities for all four emergency phases (mitigation, preparedness, response, and recovery), for all
types of emergencies and disasters (natural, man-made, and attack) and for all levels of
government and the private sector.

41. Which of the four phases of emergency management is defined as “sustained action that
reduces or eliminates long-term risk to people and property from natural hazards and their
effects.”?

A. Recovery
B. Mitigation
C. Response
D. Preparedness

42. You are a community health nurse collaborating with the Red Cross and working with disaster
relief following a typhoon which flooded and devastated the whole province. Finding safe
housing for survivors, organizing support for the family, organizing counseling debriefing
sessions and securing physical care are the services you are involved with. To which type of
prevention are these activities included:

A. Tertiary prevention
B. Primary prevention
C. Aggregate care prevention
D. Secondary prevention

43. During the disaster you see a victim with a green tag, you know that the person:

A. has injuries that are significant and require medical care but can wait hours with threat to life
or limb
B. has injuries that are life threatening but survival is good with minimal intervention
C. indicates injuries that are extensive and chances of survival are unlikely even with definitive
care
D. has injuries that are minor and treatment can be delayed from hours to days

44. The term given to a category of triage that refers to life threatening or potentially life
threatening injury or illness requiring immediate treatment:

A. Immediate
B. Emergent
C. Non-acute
D. Urgent
45. Which of the following terms refer to a process by which the individual receives education
about recognition of stress reaction and management strategies for handling stress which may be
instituted after a disaster?

A. Clinical incident stress management


B. Follow-up
C. Defriefing
D. Defusion

Situation 10 – As a member of the health and nursing team you have a crucial role to play in
ensuring that all the members participate actively is the various tasks agreed upon.

46. While eating his meal, Matthew accidentally dislodges his IV lines and bleeds. Blood oozes
on the surface of the over-bed table. It is most appropriate that you instruct the housekeeper to
clean the table with:

A. Acetone
B. Alcohol
C. Ammonia
D. Bleach

47. You are a member of the infection control team of the hospital. Based on a feedback during
the meeting of the committee there is an increased incidence of pseudomonas infection in the
Burn Unit (3 out of 10 patients had positive blood and wound culture). What is your priority
activity?

A. Establish policies for surveillance and monitoring


B. Do data gathering about the possible sources of infection (observation, chart review,
interview).
C. Assign point persons who can implement policies.
D. Meet with the nursing group working in the burn unit and discuss problem with them.

48. Part of your responsibility as a member of the diabetes core group is to get referrals from the
various wards regarding diabetic patients needing diabetes education. Prior to discharge today, 4
patients are referred to you. How would you start prioritizing your activities?

A. Bring your diabetes teaching kit and start your session taking into consideration their distance
from your office
B. Contact the nurse in-charge and find out from her the reason for the referral
C. Determine their learning needs then prioritize
D. Involve the whole family in the teaching class

49. You have been designated as a member of the task force to plan activities for the Cancer
Consciousness Week. Your committee has 4 months to plan and implement the plan. You are
assigned to contact the various cancer support groups in your hospital. What will be your priority
activity?

A. Find out if there is a budget for this activity


B. Clarify objectives of the activity with the task force before contacting the support groups
C. Determine the VIPs and Celebrities who will be invited
D. Find out how many support groups there are in the hospital and get the contact number of their
president

50. You are invited to participate in the medical mission activity of your alumni association. In
the planning stage everybody is expected to identify what they can do during the medical mission
and what resources are needed. You thought it is also your chance to share what you can do for
others. What will be your most important role where you can demonstrate the impact of nursing
in health?

A. Conduct health education on healthy life style


B. Be a triage nurse
C. Take the initial history and document findings
D. Act as a coordinator

np4 practice exam answer key

note: i may not be able to post the rationales, its start of the review season and i am super busy,
plus there are people na nangpipirate ng materials...so so so bad

NURSING PRACTICE IV – Care of Clients with Physiologic and


Psychosocial Alterations (Part B)

MULTIPLE CHOICE

Situation 1 – Because of the serious consequences of severe burns, management requires a


multidisciplinary approach. You have important responsibilities as a nurse.

1. While Sergio was lighting a barbecue grill with a lighter fluid, his shirt burns into flames. The
most effective way to extinguish the flames with as little further damage as possible is to:

log roll on the grass/ground

2. Once the flames are extinguished, it is most important to:

assess the Sergio’s breathing

3. Sergio is brought to Emergency Room after the barbecue grill accident. Based on the
assessment of the physician, Sergio sustained superficial partial thickness burns on his trunk,
right upper extremities and right lower extremities. His wife asks what that means? Your most
accurate response would be:

Epidermis and dermis are both damaged

4. During the first 24 hours after the thermal injury, you should asses Sergio for:

hyperkalemia and hyponatremia

5. Teddy, who sustained deep partial thickness and full thickness burns of the face, whole anterior
chest and both upper extremities two days ago begins to exhibit extreme restlessness. You
recognize that this most likely indicates that Teddy is developing:

metabolic acidosis

Situation 2 – You are now working as a staff nurse in a general hospital. You have to be prepared
to handle situations with ethico-legal and moral implifications.

6. You are in night duty in surgical ward. One of your patients Martin is a prisoner who sustained
an abdominal gunshot wound. He is being guarded by policeman from the local police unit.
During your rounds you heard a commotion. You saw the policeman trying to hit Martin. You
asked why he was trying to hit Martin. He denied the matter. Which among the following
activities will you do first?

Call your nurse supervisor and report the incident

7. You are on morning duty in the medical ward. You have 10 patients assigned to you. During
your endorsement rounds, you found out that one of your patients was not in bed. The patient
next to him informed you that he went home without notifying the nurses. Which among the
following will you do first?

Report the incident to your supervisor

8. You are on duty in the medical ward. You were asked to check the narcotics cabinet. You found
out that what is on record does not tally with the drugs used. Which among the following will you
do first?

Report the matter to your supervisor

9. You are on duty in the medical ward. The mother of your patient who is also a nurse, came
running to the nurses station and informed you that Fiolo went into cardiopulmonary arrest.
Bring the crash cart to the room

10. You are admitting Jorge to the ward and you found out that he is positive for HIV. Which
among the following will you do first?

Report the matter to your head nurse

Situation 3 - Colorectal cancer can affect old and younger people. Surgical procedures and other
modes of treatment are done to ensure quality of life. You are assigned in the cancer institute to
care of patients with this type of cancer.

11. Larry, 55 years old, who is suspected of having colorectal cancer, is admitted to the CI. After
taking the history and vital signs the physician does which test as a screening test for colorectal
cancer.

Annual digital rectal examination

12. To confirm his impression of colorectal cancer, Larry will require which diagnostic study?

Proctosigmoidoscopy

13. The following are risk factors for colorectal cancer, EXCEPT:

Smoking

14. Symptoms associated with cancer of the colon include:

blood in the stools, anemia, and “pencil shaped” stools

15. Several days prior to bowel surgery, Larry may be given sulfasuxidine and neomycin
primarily to:

reduce the bacterial content of the colon

Situation 4 – ENTEROSTOMAL THERAPY is now considered a specialty in nursing. You are


participating in the OSTOMY CARE CLASS.

16. You plan to teach Fermin how to irrigate the colostomy when:

The stools starts to become formed, around the 7th postoperative day

17. When preparing to teach Fermin how to irrigate colostomy, you should plan to do the
procedure:

When Fermin would have normal bowel movement

18. When observing a return demonstration of a colostomy irrigation, you know that more
teaching is required if Fermin:

Hangs the irrigating bag on the bathroom door cloth hook during fluid insertion

19. You are aware that teaching about colostomy care is understood when Fermin states, “I will
contact my physician and report:

If I have any difficulty inserting the irrigating tub into the stoma.”

20. You would know after teaching Fermin that dietary instruction for him is effective when he
states, “It is important that I eat:

A. Everything that I ate before the operation, while avoiding foods that cause gas.”

Situation 5 – Ensuring safety is one of your most important responsibilities. You will need to
provide instructions and information to your clients to prevent complications.

21. Randy has chest tubes attached to a pleural drainage system. When caring for him you should:

palpate the surrounding areas for crepitus

22. Fanny, came in from PACU after pelvic surgery. As Fanny’s nurse you know that the sign that
would be indicative of a developing thrombophlebitis would be:

a tender, painful area on the leg

23. To prevent recurrent attacks on Terry who has acute glumerulonephritis, you should instruct
her to:

seek early treatment for respiratory infections

24. Herbert had a laryngectomy and he is now for discharge. He verbalized his concern regarding
his laryngectomy tube being dislodged. What should you teach him first?

Notify the physician at once

25. When caring for Larry after an exploratory chest surgery and pneumonectomy, your priority
would be to maintain:
ventilation exchange

Situation 6 – Infection can cause debilitating consequences when host resistance is compromised
and virulence of microorganisms and environmental factors are favorable. Infection control is one
important responsibility of the nurse to ensure quality of care.

26. Honrad, who has been complaining of anorexia and feeling tired, develops jaundice, after a
workup he is diagnosed of having Hepatitis A. his wife asks you about gamma globulin for
herself and her household help. Your most appropriate response would be:

“You should contact your physician immediately about getting gammaglobulin.”

27. Voltaire develops a nosocomial respiratory tract infection. He ask you what that means? Your
best response would be:

“You acquired the infection after you have been admitted to the hospital.”

28. As a nurse you know that one of the complications that you have to watch out for when caring
for Omar who is receiving total parenteral nutrition is:

infection

29. A solution used to treat Pseudomonas wound infection is:

Dakin’s solution

30. Which of the following is the most reliable in diagnosing a wound infection?

WBC count of 20,000/μL

Situation 7 – As a nurse you need to anticipate the occurrence of complications of stroke so that
life threatening situations can be prevented.

31. Wendy is admitted to the hospital with signs and symptoms of stroke. Her Glasgow Coma
Scale is 6 on admission. A central venous catheter was inserted an I.V. infusion was started. As a
nurse assigned to Wendy what will be your priority goal?

Maintain a patent airway

32. Knowing that for a comatose patient hearing is the last sense to be lost, as Judy’s nurse, what
should you do?

Speak softly then hold her hands gently


33. Which among the following interventions should you consider as the highest priority when
caring for June who has hemiparesis secondary to stroke?

Perform range of motion exercises

34. Ivy, age 40, was admitted to the hospital with a severe headache, stiff neck and photophobia.
She was diagnosed with a subarachnoid hemorrhage secondary to ruptured aneurysm. While
waiting for surgery, you can provide a therapeutic environment by doing which of the following?

dimming the light in her room

35. When performing a neurologic assessment on Walter, you find that his pupils are fixed and
dilated. This indicated that he:

has received a significant brain injury

Situation 8 – With the improvement in life expectancies and the emphasis in the quality of life it
is important to provide quality care to our older patients. There are frequently encountered
situations and issues relevant to the older patients.

36. Hypoxia may occur in the older patients because of which of the following physiologic
changes associated with aging.

Decreased alveolar surfaced area

37. The older patient is at higher risk for incontinence because of:

decreased bladder capacity

38. Merle, age 86, is complaining of dizziness when she stands up. This may indicate:

functional decline

39. Cardiac ischemia in an older patient usually produces:

Acute confusion

40. The most dependable sign of infection in the older patient is:

change in mental status


Situation 9 – A “disaster” is a large-scale emergency—even a small emergency left unmanaged
may turn into a disaster. Disaster preparedness is crucial and is everybody’s business. There are
agencies that are in charge of ensuring prompt response. Comprehensive Emergency
Management (CEM) is an integrated approach to the management of emergency programs and
activities for all four emergency phases (mitigation, preparedness, response, and recovery), for all
types of emergencies and disasters (natural, man-made, and attack) and for all levels of
government and the private sector.

41. Which of the four phases of emergency management is defined as “sustained action that
reduces or eliminates long-term risk to people and property from natural hazards and their
effects.”?

Mitigation

42. You are a community health nurse collaborating with the Red Cross and working with disaster
relief following a typhoon which flooded and devastated the whole province. Finding safe
housing for survivors, organizing support for the family, organizing counseling debriefing
sessions and securing physical care are the services you are involved with. To which type of
prevention are these activities included:

Tertiary prevention

43. During the disaster you see a victim with a green tag, you know that the person:

has injuries that are minor and treatment can be delayed from hours to days

44. The term given to a category of triage that refers to life threatening or potentially life
threatening injury or illness requiring immediate treatment:

Emergent

45. Which of the following terms refer to a process by which the individual receives education
about recognition of stress reaction and management strategies for handling stress which may be
instituted after a disaster?

Defusion

Situation 10 – As a member of the health and nursing team you have a crucial role to play in
ensuring that all the members participate actively is the various tasks agreed upon.

46. While eating his meal, Matthew accidentally dislodges his IV lines and bleeds. Blood oozes
on the surface of the over-bed table. It is most appropriate that you instruct the housekeeper to
clean the table with:
Bleach

47. You are a member of the infection control team of the hospital. Based on a feedback during
the meeting of the committee there is an increased incidence of pseudomonas infection in the
Burn Unit (3 out of 10 patients had positive blood and wound culture). What is your priority
activity?

Do data gathering about the possible sources of infection (observation, chart review, interview).

48. Part of your responsibility as a member of the diabetes core group is to get referrals from the
various wards regarding diabetic patients needing diabetes education. Prior to discharge today, 4
patients are referred to you. How would you start prioritizing your activities?

Determine their learning needs then prioritize

49. You have been designated as a member of the task force to plan activities for the Cancer
Consciousness Week. Your committee has 4 months to plan and implement the plan. You are
assigned to contact the various cancer support groups in your hospital. What will be your priority
activity?

Clarify objectives of the activity with the task force before contacting the support groups

50. You are invited to participate in the medical mission activity of your alumni association. In
the planning stage everybody is expected to identify what they can do during the medical mission
and what resources are needed. You thought it is also your chance to share what you can do for
others. What will be your most important role where you can demonstrate the impact of nursing
in health?

Conduct health education on healthy life style


Wednesday, January 23, 2008
StuffedNurse: NP5 practice exam

NURSING PRACTICE V- Care of Clients with physiologic and


Psychosocial
Alterations (Part C).

SITUATIONAL

Situation 1 – Jimmy developed this goal for hospitalization. “To get a


handle on my nervousness.” The nurse is going to collaborate with
him to reach his goal. Jimmy was admitted to the hospital because
he called his therapist that he planned to asphyxiate himself with
exhaust from his car but frightened instead. He realized he needed
help.

1. The nurse recognized that Jimmy had conceptualized his problem


and the next priority goal in the care plan is:

A. help the client find meaning in his experience


B. help the client to plan alternatives
C. help the client cope with the present problem
D. help the client to communicate

2. The nurse is guided that Jimmy is aware of his concerns of the


“here and now” when he crossed out which item from this “list of
what to know”.

A. anxiety laden unconscious conflicts


B. subjective idea of the range of mild to severe anxiety
C. early signs of anxiety
D. physiologic indices of anxiety

3. While Jimmy was discussing the signs and symptoms of anxiety


with his nurse, he recognized that complete disruption of the ability
to perceive occurs in:

A. panic state of anxiety


B. severe anxiety
C. moderate anxiety
D. mild anxiety

4. Jimmy initiates independence and takes an active part in his self


care with the following EXCEPT:

A. agreeing to contact the staff when he is anxious


B. becoming aware of the conscious feeling
C. assessing need for medication and medicating himself
D. writing out a list of behaviors that he identified as anxious

5. The nurse notes effectiveness of interventions in using subjective


and objective data in the:

A. initial plans or orders


B. database
C. problem list
D. progress notes

Situation 2 – A research study was undertaken in order to identify


and analyze a disabled boy’s coping reaction pattern during stress.

6. This study which is an in depth study of one boy is a:

A. case study
B. longitudinal study
C. cross-sectional study
D. evaluative study

7. The process recording was the principal tool for data collection.
Which of the following is NOT a part of a process recording?
A. Non verbal narrative account
B. Analysis and interpretation
C. Audio-visual recording
D. Verbal narrative account

8. Which of these does NOT happen in a descriptive study?

A. Exploration of relationship between two or more phenomena.


B. Explanation of relationship between two or more phenomena.
C. Investigation of a phenomenon in real life context.
D. Manipulation of variable

9. The investigator also provided the nursing care of the subject. The
investigator is referred to as a/an:

A. Participant-observer
B. Observer researcher
C. Caregiver
D. Advocate

10. To ensure reliability of the study, the investigator’s analysis and


interpretations were:

A. subjected to statistical treatment


B. correlated with a list of coping behaviors
C. subjected to an inter-observer agreement
D. scored and compared standard criteria

Situation 3 – During the morning endorsement, the outgoing nurse


informed the nursing staff that Regina, 35 years old, was given
Flurazepam (Dalmane) 15mg at 10:00pm because she had trouble
going to sleep. Before approaching Regina, the nurse read the
observation of the night nurse.

11. Which of the following approaches of the nurse validates the data
gathered?

A. “I learned that you were up till ten last night, tell me what
happened
before you were finally able to sleep and how was your sleep?”
B. “Hmm.. You look like you had a very sound sleep. That pill you
were
given last night is effective isn’t it?”
C. “Regina, did you sleep well?”
D. “Regina, how are you?”

12. Regina is a high school teacher. Which of these information


LEAST communicate attention and care for her needs for information
about her medicine?

A. Guided by a medication teaching plan, go over with her the


purpose, indications and special instructions, about the medication
and provide her a checklist
B. Provide a drug literature.
C. Have an informal conversation about the medication and its effects
D. Ask her what time she would like to watch the informative video
about the medication.

13. The nurse engages Regina in the process of mutual inquiry to


provide an opportunity for Regina to:

A. face emerging problems realistically


B. conceptualize her problem
C. cope with her present problem
D. perceive her participation in an experience

!4. Which of these responses indicate that Regina needs further


discussion regarding special instructions?

A. “I have to take this medicine judiciously”


B. “I know I will stop taking the medicine when there is advice from
the doctor for me to discontinue.”
C. “I will inform you and the doctor any untoward reactions I have.”
D. “I like taking this sleeping pill. It solves my problem of insomnia. I
wish I can take it for life.”

15. Regina commits to herself that she understood and will observe
all the medicine precautions by:

A. affixing her signature to the teaching plan that she has understood
the nurse
B. committing what she learned to her memory
C. verbally agreeing with the nurse
D. relying on her husband to remember the precautions

Situation 4 – The nurse-patient relationship is a modality through


which the nurse meets the client’s needs.

16. The nurse’s most unique tool in working with the emotionally ill
client is his/her

A. theoretical knowledge
B. personality make up
C. emotional reactions
D. communication skills

17. The psychiatric nurse who is alert to both the physical and
emotional needs of clients is working from the philosophical
framework that states:

A. All behavior is meaningful, communicating a message or a need.


B. Human beings are systems of interdependent and interrelated
parts.
C. Each individual has the potential for growth and change in the
direction of positive mental health.
D. There is a basic similarity among all human beings.
18. One way to increase objectivity in dealing with one’s fears and
anxieties is through the process of:

A. observation
B. intervention
C. validation
D. collaboration

19. All of the following responses are non therapeutic. Which is the
MOST direct violation of the concept, congruence of behavior?

A. Responding in a punitive manner to the client.


B. Rejecting the client as a unique human being
C. Tolerating all behavior in the client.
D. Communicating ambivalent messages to the client.

20. The mentally ill person responds positively to the nurse who is
warm and caring. This is a demonstration of the nurse’s role as:

A. counselor
B. mother surrogate
C. therapist
D. socializing agent

Situation 5 – The nurse engages the client in a nurse-patient


interaction.

21. The best time to inform the client about terminating the nurse-
patient relationship is:

A. when the client asks how long the relationship would be


B. during the working phase
C. towards the end of the relationship
D. at the start of the relationship

22. The client says, “I want to tell you something but can you
promise that you will keep this a secret?” A therapeutic response of
the nurse is:

A. “Yes, our interaction is confidential provided the information you


tell me is not detrimental to your safety.”
B. “Of course yes, this is just between you and me. Promise!”
C. “Yes, it is my principle to uphold my client’s rights.”
D. “Yes, you have the right to invoke confidentiality of our
interaction.”

23. When the nurse respects the client’s self-disclosure, this is a


gauge for the nurse’s:

A. trustworthiness
B. loyalty
C. integrity
D. professionalism

24. Rapport has been established in the nurse-client relationship.


The client asks to visit the nurse after his discharge. The appropriate
response of the nurse would be:

A. “The best time to talk is during the nurse-client interaction time. I


am committed to have this time available for us while you are at the
hospital and ends after your discharge.”
B. “Yes, If you keep it confidential, this is part of privileged
communication.”
C. “I am committed for your care.”
D. “I am sorry, though I would want to, it is against hospital policy.”

25. The client has not been visited by relatives for months. He gives a
telephone number and requests the nurse to call. An appropriate
action of the nurse would be:

A. Inform the attending physician about the request of the client.


B. Assist the client to bring his concern to the attention of the social
worker.
C. “Here (gives her mobile phone). You may call this number now”.
D. Ask the client what is the purpose of contacting his relatives.

Situation 6 – Camila, 25 years old, was reported to be gradually


withdrawing and isolating herself from friends and family members.
She became neglectful of her personal hygiene. She was observed to
be talking irrelevantly and incoherently. She was diagnosed as
schizophrenia disorder.

26. The past history of Camila would most probably reveal that her
premorbid personality is:

A. schizoid
B. extrovert
C. ambivert
D. cycloid

27. Camila refuses to relate with to others because she:

A. is irritable
B. feels superior of others
C. anticipates rejection
D. is depressed

28. Which of the following disturbances in interpersonal relationships


MOST often predispose to the development of schizophrenia?

A. Lack of participation in peer groups


B. Faulty family atmosphere and interaction
C. Extreme rebellion towards authority figures
D. Solo parenting

29. Camila’s indifference toward the environment is a compensatory


behavior to overcome:
A. Guilt feelings
B. Ambivalence
C. Narcissistic behavior
D. Insecurity feelings

30. Schizophrenia is a/an:

A. anxiety disorder
B. neurosis
C. psychosis
D. personality disorder

Situation 7 – Salome, 80 year old widow, has been observed to be


irritable, demanding and speaking louder than usual. She would
prefer to be alone and take her meals by herself, minimize receiving
visitors at home and no longer bothers to answer telephone calls
because of deterioration of hearing. She was brought by her
daughter to the Geriatric clinic for assessment and treatment.

31. The nurse counsels Salome’s daughter that Salome’s becoming


very loud and tendency to become aggressive is a/an:

A. beginning indifference to the world around her


B. attempt to maintain authoritative role
C. overcompensation for hearing loss
D. behavior indicative of unresolved repressed conflict of the past

32. A nursing diagnosis for Salome is:

A. sensory deprivation
B. social isolation
C. cognitive impairment
D. ego despair

33. The nurse will assist Salome and her daughter to plan a goal
which is for Salome to:
A. adjust to the loss of sensory and perceptual function
B. participate in conversation and other social situations
C. accept the steady loss of hearing that occurs with aging
D. increase her self-esteem to maintain her authoritative role

34. The daughter understood, the following ways to assist Salome


meet her needs and avoiding which of the following:

A. Using short simple sentences


B. Speaking distinctly and slowly
C. Speaking at eye level and having the client’s attention
D. Allowing her to take her meals alone

35. Salome was fitted a hearing aid. She understood the proper use
and wear of this device when she says that the battery should be
functional, the device is turned on and adjusted to a:

A. therapeutic level
B. comfortable level
C. prescribed level
D. audible level

Situation 8 – For more than a month now, Cecilia is persistently


feeling restless, worried and feeling as if something dreadful is going
to happen. She fears being alone in places and situations where she
thinks that no one might come to rescue her just in case something
happens to her.

36. Cecilia is demonstrating:

A. acrophobia
B. claustrophobia
C. agoraphobia
D. xenophobia
37. Cecilia’s problem is that she always sees and thinks negative
things hence she is always fearful. Phobia is a symptom described as:

A. organic
B. psychosomatic
C. psychotic
D. neurotic

38. Cecilia has a lot of irrational thoughts: The goal of therapy is to


modify her:

A. communication
B. cognition
C. observation
D. perception

39. Cognitive therapy is indicated for Cecilia when she is already able
to handle anxiety reactions. Which of the following should the nurse
implement?

A. assist her in recognizing irrational beliefs and thoughts


B. help find meaning in her behavior
C. provide positive reinforcement for acceptable behavior
D. Administer anxiolytic drug

40. After discharge, which of these behaviors indicate a positive


result of being able to overcome her phobia?

A. She reads a book in the public library


B. She drives alone along the long expressway.
C. She watches television with the family in the recreation room
D. She joins an art therapy group

Situation 9 – It is the first day of clinical experience of nursing


students at the Psychiatry Ward. During the orientation, the nurse
emphasizes that the team members including nursing students are
legally responsible to safeguard patient’s records from loss or
destruction or from people not authorized to read it.

41. It is unethical to tell one’s friends and family members data


about patients because doing so is a violation of patients’ rights to:

A. Informed consent
B. Confidentiality
C. Least restrictive environment
D. Civil liberty

42. The nurse must see to it that the written consent of mentally ill
patients must be taken from:

A. Doctor
B. Social worker
C. Parents or legal guardian
D. Law enforcement authorities

43. In an extreme situation and when no other resident or intern is


available, should a nurse receive telephone orders, the order has to
be correctly written and signed by the physician within:

A. 24 hours
B. 36 hours
C. 48 hours
D. 12 hours

44. The following are SOAP (Subjective – Objective – Analysis – Plan)


statements on a problem: Anxiety about diagnosis. What is the
objective data?

A. Relate patient’s feelings to physician; initiate and encourage her


to verbalize her fears; give emotional support by spending more time
with patient; continue to make necessary explanations regarding
diagnostic tests.
B. Has periods of crying; frequently verbalizes fear of what
diagnostic tests will reveal
C. Anxiety due to unknown
D. “I’m so worried about what else they’ll find wrong with me.”

45. Nursing care plans provide very meaningful data for the patient
profile and initial plan because the focus is on the:

A. Summary of chronological notations made by individual health


team members
B. Identification of patient’s responses to medical diagnoses and
treatment
C. Patient’s responses to health and illness as a total person in
interaction with the environment
D. Step by step procedures for the management of common problems

Situation 10 – Marie is 5½ years old and described by the mother as


bedwetting at night.

46. Which of the following is the MOST common physiological cause


of night bed wetting?

A. deep sleep factors


B. abnormal bladder development
C. infections
D. familial and genetic factors

47. All of the following, EXCEPT one compromise the concepts of


behavior therapy program.

A. reward and punishment


B. extinction
C. learning
D. placebo as a form of treatment

48. To help Marie who bed wets at night practice acceptable and
appropriate behavior, it is important for the parents to be consistent
with the following approaches EXCEPT:

A. discipline with a kind attitude


B. matter of fact in handling the behavior
C. sympathize for the child
D. be loving yet firm

49. A therapeutic verbal approach that communicates strong


disapproval is:

A. “You are supposed to get up and go in the toilet when you feel you
have to go and did not. The next time you bed wet, I’ll tell your
friends and hang your sheets out the window for them to see.”
B. “You are supposed to get up and go in the toilet when you feel you
have to go and did not. I expect you to from now on without fail.”
C. “If you bed wet, you will change your bed linen and wash the
sheets.”
D. “If you don’t make an effort to control your bedwetting, I’d be
upset and disappointed.”

50. During your conference, the parent inquires how to motivate


Marie to be dry in the morning. Your response which is an immediate
intervention would be:

A. Give a star each time she wakes up dry and every set of five stars,
give a prize.
B. Tokens make her materialistic at an early age. Give praise and
hugs occasionally.
C. What does your child want that you can give every time he/she
wakes up dry in the morning?
D. Promise him/her a long awaited vacation after school is over.

NP5 practice exam answer key

note: rationales will be for classroom discussion to avoid piracy


NURSING PRACTICE V- Care of Clients with physiologic and
Psychosocial
Alterations (Part C).

SITUATIONAL

Situation 1 – Jimmy developed this goal for hospitalization. “To get a


handle on my nervousness.” The nurse is going to collaborate with
him to reach his goal. Jimmy was admitted to the hospital because
he called his therapist that he planned to asphyxiate himself with
exhaust from his car but frightened instead. He realized he needed
help.

1. The nurse recognized that Jimmy had conceptualized his problem


and the next priority goal in the care plan is:

help the client to plan alternatives

2. The nurse is guided that Jimmy is aware of his concerns of the


“here and now” when he crossed out which item from this “list of
what to know”.

subjective idea of the range of mild to severe anxiety

3. While Jimmy was discussing the signs and symptoms of anxiety


with his nurse, he recognized that complete disruption of the ability
to perceive occurs in:

panic state of anxiety

4. Jimmy initiates independence and takes an active part in his self


care with the following EXCEPT:
agreeing to contact the staff when he is anxious

5. The nurse notes effectiveness of interventions in using subjective


and objective data in the:

progress notes

Situation 2 – A research study was undertaken in order to identify


and analyze a disabled boy’s coping reaction pattern during stress.

6. This study which is an in depth study of one boy is a:

case study

7. The process recording was the principal tool for data collection.
Which of the following is NOT a part of a process recording?

Audio-visual recording

8. Which of these does NOT happen in a descriptive study?

Manipulation of variable

9. The investigator also provided the nursing care of the subject. The
investigator is referred to as a/an:

Observer researcher

10. To ensure reliability of the study, the investigator’s analysis and


interpretations were:

subjected to an inter-observer agreement

Situation 3 – During the morning endorsement, the outgoing nurse


informed the nursing staff that Regina, 35 years old, was given
Flurazepam (Dalmane) 15mg at 10:00pm because she had trouble
going to sleep. Before approaching Regina, the nurse read the
observation of the night nurse.

11. Which of the following approaches of the nurse validates the data
gathered?

“I learned that you were up till ten last night, tell me what happened
before you were finally able to sleep and how was your sleep?”

12. Regina is a high school teacher. Which of these information


LEAST communicate attention and care for her needs for information
about her medicine?

Provide a drug literature.

13. The nurse engages Regina in the process of mutual inquiry to


provide an opportunity for Regina to:

perceive her participation in an experience

!4. Which of these responses indicate that Regina needs further


discussion regarding special instructions?

“I like taking this sleeping pill. It solves my problem of insomnia. I


wish I can take it for life.”

15. Regina commits to herself that she understood and will observe
all the medicine precautions by:

committing what she learned to her memory

Situation 4 – The nurse-patient relationship is a modality through


which the nurse meets the client’s needs.
16. The nurse’s most unique tool in working with the emotionally ill
client is his/her

communication skills

17. The psychiatric nurse who is alert to both the physical and
emotional needs of clients is working from the philosophical
framework that states:

All behavior is meaningful, communicating a message or a need.

18. One way to increase objectivity in dealing with one’s fears and
anxieties is through the process of:

validation

19. All of the following responses are non therapeutic. Which is the
MOST direct violation of the concept, congruence of behavior?

Communicating ambivalent messages to the client.

20. The mentally ill person responds positively to the nurse who is
warm and caring. This is a demonstration of the nurse’s role as:

mother surrogate

Situation 5 – The nurse engages the client in a nurse-patient


interaction.

21. The best time to inform the client about terminating the nurse-
patient relationship is:

at the start of the relationship


22. The client says, “I want to tell you something but can you
promise that you will keep this a secret?” A therapeutic response of
the nurse is:

“Yes, our interaction is confidential provided the information you tell


me is not detrimental to your safety.”

23. When the nurse respects the client’s self-disclosure, this is a


gauge for the nurse’s:

trustworthiness

24. Rapport has been established in the nurse-client relationship.


The client asks to visit the nurse after his discharge. The appropriate
response of the nurse would be:

“The best time to talk is during the nurse-client interaction time. I


am committed to have this time available for us while you are at the
hospital and ends after your discharge.”

25. The client has not been visited by relatives for months. He gives a
telephone number and requests the nurse to call. An appropriate
action of the nurse would be:

Ask the client what is the purpose of contacting his relatives.

Situation 6 – Camila, 25 years old, was reported to be gradually


withdrawing and isolating herself from friends and family members.
She became neglectful of her personal hygiene. She was observed to
be talking irrelevantly and incoherently. She was diagnosed as
schizophrenia disorder.

26. The past history of Camila would most probably reveal that her
premorbid personality is:
schizoid

27. Camila refuses to relate with to others because she:

anticipates rejection

28. Which of the following disturbances in interpersonal relationships


MOST often predispose to the development of schizophrenia?

Faulty family atmosphere and interaction

29. Camila’s indifference toward the environment is a compensatory


behavior to overcome:

Insecurity feelings

30. Schizophrenia is a/an:

psychosis

Situation 7 – Salome, 80 year old widow, has been observed to be


irritable, demanding and speaking louder than usual. She would
prefer to be alone and take her meals by herself, minimize receiving
visitors at home and no longer bothers to answer telephone calls
because of deterioration of hearing. She was brought by her
daughter to the Geriatric clinic for assessment and treatment.

31. The nurse counsels Salome’s daughter that Salome’s becoming


very loud and tendency to become aggressive is a/an:

overcompensation for hearing loss

32. A nursing diagnosis for Salome is:

social isolation
33. The nurse will assist Salome and her daughter to plan a goal
which is for Salome to:

adjust to the loss of sensory and perceptual function

34. The daughter understood, the following ways to assist Salome


meet her needs and avoiding which of the following:

Allowing her to take her meals alone

35. Salome was fitted a hearing aid. She understood the proper use
and wear of this device when she says that the battery should be
functional, the device is turned on and adjusted to a:

prescribed level

Situation 8 – For more than a month now, Cecilia is persistently


feeling restless, worried and feeling as if something dreadful is going
to happen. She fears being alone in places and situations where she
thinks that no one might come to rescue her just in case something
happens to her.

36. Cecilia is demonstrating:

agoraphobia

37. Cecilia’s problem is that she always sees and thinks negative
things hence she is always fearful. Phobia is a symptom described as:

neurotic

38. Cecilia has a lot of irrational thoughts: The goal of therapy is to


modify her:

cognition
39. Cognitive therapy is indicated for Cecilia when she is already able
to handle anxiety reactions. Which of the following should the nurse
implement?

provide positive reinforcement for acceptable behavior

40. After discharge, which of these behaviors indicate a positive


result of being able to overcome her phobia?

She drives alone along the long expressway.

Situation 9 – It is the first day of clinical experience of nursing


students at the Psychiatry Ward. During the orientation, the nurse
emphasizes that the team members including nursing students are
legally responsible to safeguard patient’s records from loss or
destruction or from people not authorized to read it.

41. It is unethical to tell one’s friends and family members data


about patients because doing so is a violation of patients’ rights to:

Confidentiality

42. The nurse must see to it that the written consent of mentally ill
patients must be taken from:

Parents or legal guardian

43. In an extreme situation and when no other resident or intern is


available, should a nurse receive telephone orders, the order has to
be correctly written and signed by the physician within:

24 hours

44. The following are SOAP (Subjective – Objective – Analysis – Plan)


statements on a problem: Anxiety about diagnosis. What is the
objective data?
Has periods of crying; frequently verbalizes fear of what diagnostic
tests will reveal

45. Nursing care plans provide very meaningful data for the patient
profile and initial plan because the focus is on the:

Identification of patient’s responses to medical diagnoses and


treatment

Situation 10 – Marie is 5½ years old and described by the mother as


bedwetting at night.

46. Which of the following is the MOST common physiological cause


of night bed wetting?

familial and genetic factors

47. All of the following, EXCEPT one compromise the concepts of


behavior therapy program.

placebo as a form of treatment

48. To help Marie who bed wets at night practice acceptable and
appropriate behavior, it is important for the parents to be consistent
with the following approaches EXCEPT:

49. A therapeutic verbal approach that communicates strong


disapproval is:

“If you don’t make an effort to control your bedwetting, I’d be upset
and disappointed.”

50. During your conference, the parent inquires how to motivate


Marie to be dry in the morning. Your response which is an immediate
intervention would be:

What does your child want that you can give every time he/she
wakes up dry in the morning?

Potrebbero piacerti anche